You are on page 1of 105

2

INDEX
01-11-2019 ............................................................................................................................................................ 3

02-11-2019............................................................................................................................................................ 7

04-11-2019 ......................................................................................................................................................... 12

05-11-2019 .......................................................................................................................................................... 15

06-11-2019 ......................................................................................................................................................... 19

07-11-2019 .......................................................................................................................................................... 23

08-11-2019 ......................................................................................................................................................... 27

09-11-2019 ......................................................................................................................................................... 31

11-11-2019 ........................................................................................................................................................... 35

12-11-2019 ......................................................................................................................................................... 40

13-11-2019 .......................................................................................................................................................... 43

14-11-2019 .......................................................................................................................................................... 47

15-11-2019 ..........................................................................................................................................................50

16-11-2019 .......................................................................................................................................................... 52

18-11-2019 .......................................................................................................................................................... 55

19-11-2019 .......................................................................................................................................................... 59

20-11-2019..........................................................................................................................................................64

21-11-2019 ......................................................................................................................................................... 68

22-11-2019 .......................................................................................................................................................... 73

23-11-2019 .......................................................................................................................................................... 76

25-11-2019 .......................................................................................................................................................... 81

26-11-2019 ......................................................................................................................................................... 86

27-11-2019 .......................................................................................................................................................... 91

28-11-2019.......................................................................................................................................................... 95

29-11-2019 ..........................................................................................................................................................99

30-11-2019........................................................................................................................................................ 102

www.shankariasacademy.com | www.iasparliament.com
3

01-11-2019 d. 1, 2 and 3
1) “Pegasus” sometimes seen in the news
recently is a/an? 4) Consider the following statements with
a. Spyware respect to UNESCO Creative Cities Network
(UCCN)
b. Exo-planet
1. The network brings together cities that
c. Indian Satellite base development on creativity in five
d. Volcanic Mountain creative fields.
2. No cities in India except Mumbai and
Hyderabad were listed under the
2) Consider the following statements network.
1. The bird is also called as “Hargila”. Which of the statement(s) given above is/are
2. They are listed as Endangered under the correct?
IUCN Red List. a. 1 only
3. They build colonies on tall trees during b. 2 only
their breeding season in winter.
c. Both 1 and 2
4. They are currently distributed only in
Assam and Bihar in India and a few d. Neither 1 nor 2
other locations in Cambodia.
Identify the species that correctly matches with 5) Consider the following statements
the above description:
1. Article 3 of the Indian Constitution
a. Great Indian Bustard deals with the formation of or changes
b. Jerdon’s Courser in the existing states of the Union of
India.
c. Greater adjutant storks
2. Jammu & Kashmir is the first State of
d. Siberian Crane the Indian Union to be reduced to a
Union Territory (UT).

3) Consider the following statements with Which of the statement(s) given above is/are
respect to National Health Profile (NHP) correct?

1. It was prepared annually by the a. 1 only


National Health Authority (NHA). b. 2 only
2. According to the NHP, 2019, India has c. Both 1 and 2
registered an improved sex ratio and a
decline in birth and death rates. d. Neither 1 nor 2

3. According to the NHP, 2019, Non-


communicable diseases dominate over 6) Arrange the following in decending order
communicable diseases in the total with respect to its weightage in the ‘Index of
disease burden of the country. Eight Core Industries’
Which of the statements given above are 1. Coal production
correct?
2. Crude Oil production
a. 1 and 2 only
3. Petroleum Refinery production
b. 1 and 3 only
4. Electricity generation
c. 2 and 3 only
5. Steel production

www.shankariasacademy.com | www.iasparliament.com
4

Choose the correct answer 9) Consider the following statements with


respect to Gogona music
a. 3-4-5-1-2
1. It is a traditional instrument made of a
b. 2-4-1-5-3
single piece of seasoned bamboo or
c. 4-3-5-2-1 horn, played during Bihu performances
in Assam.
d. 3-5-4-1-2
2. The Ramdhan Gogona is performed by
women whereas Lahori Gogona is
7) Consider the following statements with performed by men.
respect to Mekong river Which of the statement(s) given above is/are
1. It orginates in Myanmar and flows correct?
through Laos, Thailand, Cambodia and a. 1 only
Vietnam.
b. 2 only
2. The Golden Triangle area, formed by the
confluence of the Mekong River and the c. Both 1 and 2
Ruak River, is a natural boundary
d. Neither 1 nor 2
between Laos, Myanmar and Thailand.
Which of the statement(s) given above is/are
correct? 10) The place ‘Gottiprolu’ which was in news
recently is located in?
a. 1 only
a. Karnataka
b. 2 only
b. Odisha
c. Both 1 and 2
c. Telangana
d. Neither 1 nor 2
d. Andhra Pradesh

8) Consider the following statements with


respect to The Right To Information ( RTI Answers
Amendment) Act, 2019
1. a
1. Under the Rules notified, the terms of
service of the CIC, including her/his pay  It has been reported that the popular
and perks, will be the same as that of the messaging platform WhatsApp was used to spy
Cabinet Secretary. on journalists and human rights activists in
India.
2. The CIC and the ICs have a fixed tenure
of three years.  The surveillance was carried out using
a spyware tool called Pegasus, which has
Which of the statement(s) given above is/are been developed by an Israeli firm, the NSO
correct? Group.
a. 1 only
 Pegasus works by sending an exploit link, and
b. 2 only if the target user clicks on the link, the
malware or the code that allows the
c. Both 1 and 2 surveillance is installed on the user’s phone.
d. Neither 1 nor 2
 A presumably newer version of the malware
does not even require a target user to click a
link.
 Once Pegasus is installed, the attacker has
complete access to the target user’s phone.

www.shankariasacademy.com | www.iasparliament.com
5

2. c 4. d
 All the other options except Greater  UNESCO has recently announced
adjutant storks are listed under Critically that Mumbai and Hyderabad have been
Endangered category of IUCN Red List. included in its network of 'Creative
Cities' on the occasion of World Cities Day
 Greater adjutant storks are 2019.
called ‘Hargila’, meaning bone swallower, in
Assamese.  The network covers seven creative fields:
crafts and folk arts, media arts, film, design,
 They build colonies on tall trees during their gastronomy, literature and music.
breeding season in winter.
 Mumbai has been designated as Creative
 Earlier, widely distributed throughout City of Films and Hyderabad a Creative
northern and eastern India and many City of Gastronomy.
countries of south and south-east Asia, the
Greater Adjutant Stork is currently distributed  Till now, Varanasi (music), Chennai
only in Assam and Bihar in India and a few (music) and Jaipur (crafts & folk
other locations in Cambodia. art) were in the network.
3. c  By joining the UNESCO Creative Cities
Network (UCCN), the cities pledge to share
 National Health Profile (NHP) is an
their best practices and develop partnerships
annual publication of the Central Bureau of
involving public and private sectors as well as
Health Intelligence (CBHI).
civil society to strengthen creation, production,
 As per the NHP, sex ratio (number of females distribution and dissemination of cultural
per 1,000 males) in the country has improved activities, goods and services.
from 933 in 2001 to 943 in 2011. 5. c
 In rural areas the sex ratio has increased from  Article 2 relates to the admission or
946 to 949. establishment of new states that are not part of
 The report also showed that the estimated the Union of India.
birth rate, death rate and natural growth rate  Article 3, on the other hand, relates to the
are declining. formation of or changes in the existing states
 As per the report, the total fertility rate of the Union of India.
(average number of children that will be born  In other words, Article 3 deals with the
to a woman during her lifetime) in 12 States internal re-adjustment inter se of the
has fallen below two children per woman and territories of the constituent states of the
nine States have reached replacement levels of Union of India.
2.1 and above.
 The Indian Union now has 28 States and
 It was also observed that non-communicable nine UTs.
diseases dominated over the communicable in
the total disease burden of the country.  Jammu & Kashmir became the first State of
the Indian Union to be reduced to a Union
Territory (UT), with former IAS officer Girish
Murmu being sworn in as its first Lieutenant
Governor (LG).
6. a
 Core sector output falls to -5.2% in September,
worst in 14 years.
 The Index of Eight Core Industries is a
monthly production index released
by CSO and is considered to be the lead

www.shankariasacademy.com | www.iasparliament.com
6

indicator of the monthly industrial  The Irrawaddy dolphin exists in small


performance in India. isolated populations around Southeast Asia
in Mekong River.
 Its weightage in the index
 Some populations are close to extinction such
1. Petroleum Refinery production (weight:
as the those in the Mekong River and
28.04 %)
Malampaya Sound in the Philippines.
2. Electricity generation (weight:
19.85 %)
3. Steel production (weight: 17.92
%)
4. Coal production (weight:
10.33 %)
5. Crude Oil production (weight:
8.98 %)
6. The Natural Gas production (weight: 88
%)
7. Cement production (weight: 5.37
%)
8. Fertilizers production (weight: 2.63
%)
7. b 8. c
 Mekong River is the 7th longest in Asia, and  The RTI (Amendment) Bill received the assent
the 12th longest in the world. of the President on the 1st August, 2019 and
became an Act of Parliament.
 It orginates from the Tibetian
Plateau and runs through China, Myanmar,  The Right To Information (Amendment)
Laos, Thailand, Cambodia and Vietnam. Act, 2019 is an amendment to the Right to
Information Act, 2005.
 In China, the river has another name
as Lancang River. It origins from Yushu  Recently, government notified the new RTI
Tibetan Autonomous Prefecture in Qinghai Rules under the Act.
Province.
 The Act to allow the government to fix the
 The Golden Triangle area, formed by the terms and conditions of service of the Chief
confluence of the Mekong River and the Ruak Information Commissioner (CIC) and
River, is a natural boundary between, Information Commissioners (ICs) of the both
Centre and the states.
1. Laos (to the east of the Mekong River),
 Under the recently notified Rules,
2. Myanmar (to the north of the Ruak River) and
1. the terms of service of the CIC, including
3. Thailand (to the west of the Mekong River).
her/his pay and perks, will be the same as
 The Golden Triangle area is the largest tourist that of the Cabinet Secretary, which is
trap in northern Thailand. significantly lower than that of the CEC.
 The Mekong River Delta in southern 2. And those of the ICs are similar to that of a
Vietnam offers the fertile lands on which the Secretary to the Government of India.
Vietnamese people grow half of their nation’s  The CIC was earlier on a par with the CEC,
agricultural product. who occupies position 9A in the table (along
with the Comptroller and Auditor General of
India, and Chairman, UPSC).

www.shankariasacademy.com | www.iasparliament.com
7

 After being downgraded to Cabinet Secretary 10. d


rank, would mean a downgradation in the
 Recently, excavation by the Archaeological
‘Table of Precedence’.
Survey of India (ASI) in Gottiprolu has
 The CIC and the ICs now have a fixed unearthed a trade centre of early historic
tenure of just three years (earlier a fixed period.
term of five years).
 Gottiprolu is a village located in Naidupet
9. a Mandal of Sri Potti Sriramulu Nellore
district, Andhra Pradesh.
 Gogona is a traditional musical instrument
which is played during Bihu performances in  The site of Gottiprolu lies on the right bank of
Assam. a distributary of river Swarnamukhi.
 It is made of a single piece of seasoned bamboo  Excavation by ASI indicates Gottiprolu as
or horn that has a bifurcation on one end. a trade centre of early historic period.
 The ASI also found a 2000-year-old brick
structure and a Vishnu sculpture at the site.
 The proximity of the site to the seacoast
suggests that the site could have served as a
strategic settlement involved in maritime
trade.
 The site also had a series of broken
terracotta pipes fitted into one another,
revealing about the civic amenities maintained
by the occupants.
 Other major retrieved antiquities unearthed
are copper and lead coins, iron spear
 Apart from being played during Bihu head, stone celts, terracotta beads, ear
performances, Gogona is also used by the studs in semi precious stones and
Mongoloid and Kirat tribes in their folk music. hopscotches.
 The characteristic feature of the Bihu dance is
the use of traditional instruments such as
Baanhi, Dhol, Gogona, Taal, Toka and Xutuli. 02-11-2019
 These instruments lend a unique musical 1) Consider the following statements with
flavour when rendering the songs for the respect to Chavang Kut Festival
dance. 1. It is the post-harvest festival of the
 According to the size, Gogona is known by Garo, Khasi and Jaintia Tribes.
different names such as Lahori Gogona, 2. The festival celebrated this year also
Ramdhan Gogona, and Xaliki Gogona. marks the centenary of Anglo-Kuki war
fought against the British in 1917-1919.
 The Ramdhan and Lahori Gogona are used
more commonly. Which of the statement(s) given above
is/are incorrect?
 The Ramdhan Gogona is performed
by men whereas Lahori Gogona is a. 1 only
performed by women.
b. 2 only
 A female Bihu perfomer is referred to c. Both 1 and 2
as Bihuwati.
d. Neither 1 nor 2

www.shankariasacademy.com | www.iasparliament.com
8

2) Consider the following statements with Which of the statement(s) given above is/are
respect to Centralized Public Grievance correct?
Redress and Monitoring System (CPGRAMS)
a. 1 only
1. It is an online web-enabled system
b. 2 only
developed by National Informatics
Centre (NIC). c. Both 1 and 2
2. It primarily aims to enable submission d. Neither 1 nor 2
of grievances by the aggrieved citizens
from anywhere and anytime to
Ministries/Departments/Organizations/ 5) “Prison Statistics India 2017” was recently
State Governments. released by which of the following?
Which of the statement(s) given above is/are 1. It was released by the Crime and
correct? Criminal Tracking Network & Systems
a. 1 only (CCTNS).

b. 2 only 2. According to the report, Kerala has the


lowest occupancy rate in prisons
c. Both 1 and 2 compared to other large States in the
country.
d. Neither 1 nor 2
Which of the statement(s) given above is/are
correct?
3) Which of the following is/are referred
a. 1 only
as White Goods?
b. 2 only
1. Radio
c. Both 1 and 2
2. Television
d. Neither 1 nor 2
3. Refrigerator
4. Washing Machine
6) Consider the following related to Dwarf
Select the correct answer using the codes given
planets
below:
1. It is in orbit around the Sun.
a. 1 and 2 only
2. It has not cleared the neighborhood
b. 3 and 4 only
around its orbit.
c. 2, 3 and 4 only
3. It can be a satellite
d. 1, 2, 3 and 4
Which of the following are the criteria to
classify a celestial body as Dwarf planet?
4) Consider the following statements with a. 1 and 2 only
respect to Armed Forces Special Powers Act
b. 2 and 3 only
(AFSPA)
c. 1 and 3 only
1. The Act is in force in Jammu & Kashmir
since 1990. d. All of the above
2. Ministry of Home Affairs is the
designated authority for notifying the
AFSPA in the Union Territories of 7) Consider the following statements with
Jammu and Kashmir (J&K) and respect to Electoral Bonds
Ladakh. 1. Registered political parties that secured
not less than 10% of votes polled in
former Lok Sabha or legislative

www.shankariasacademy.com | www.iasparliament.com
9

assembly elections are eligible to receive Which of the statement(s) given above is/are
electoral bonds. correct?
2. Electoral bonds will be valid for 15 a. 1 only
days only from the date of purchase.
b. 2 only
Which of the statement(s) given above is/are
c. Both 1 and 2
correct?
d. Neither 1 nor 2
a. 1 only
b. 2 only
c. Both 1 and 2 Answers
d. Neither 1 nor 2 1. a
 Chavang Kut, the post-harvest festival of
the Kuki-Chin-Mizo communities is being
8) Consider the following statements with celebrated across North-Eastern states with
respect to states flag in India traditional gaiety and enthusiasm.
1. There is no explicit mention in the  The festival also marks the centenary
Constitution of India that states can of Anglo-Kuki war fought against the
have their own flags. British in 1917-1919.
2. A flag is enumerated in the Seventh  In Manipur, Mizoram and Assam and other
Schedule. parts of country, the festival is organized every
Which of the statement(s) given above is/are year as thanksgiving for a bountiful harvest.
correct?
 It is one of the most important festivals of
a. 1 only Kuki-Chin-Mizo communities.
b. 2 only 2. c
c. Both 1 and 2  DAR&PG is the nodal agency of the
Government of India in-charge of policy
d. Neither 1 nor 2
making, monitoring and coordinating
departments for public grievances.
9) Saint Kitts & Nevis Island which was in news  Centralized Public Grievance Redress
recently is located in? and Monitoring System
(CPGRAMS) of Department of
a. Gulf of mexico
Administrative Reforms and Public
b. Caribbean Sea Grievances (DAR&PG) is an online web-
enabled system over NICNET (NIC Network)
c. Sargasso sea
developed by National Informatics Centre
d. Bering sea (NIC).
 It was developed in association with
Directorate of Public Grievances (DPG) and
10) Consider the following statements with
Department of Administrative Reforms and
respect to Air Independent Propulsion
Public Grievances (DAR&PG).
(AIP) system
1. AIP allows the nuclear submarines to  CPGRAMS is the platform based on web
operate without access to atmospheric technology which primarily aims to enable
oxygen. submission of grievances by the aggrieved
citizens from anywhere and anytime (24x7)
2. It allows diesel-electric submarines to basis to
recharge their batteries without Ministries/Departments/Organizations/State
running their engines. Governments who scrutinize and take action

www.shankariasacademy.com | www.iasparliament.com
10

for speedy and favourable redress of these  Tamil Nadu has reported the lowest
grievances. occupancy rate in prisons compared to other
large States in the country.
 Tracking grievances is also facilitated on this
portal through the system generated unique 6. a
registration number.
 As of today, there are officially five dwarf
3. b planets in our Solar System.
 White goods are large home 1. Pluto, Eris, Makemake, Haumea and
appliances such as refrigerators, Ceres.
freezers, washing machines, dishwashers
and air conditioners.  Now, there is a claimant for a sixth dwarf
planet, Called Hygiea.
 They are traditionally available only in white
colour.  It has so far been taken to be an asteroid. It lies
in the asteroid belt between Mars and Jupiter.
 Even though you can purchase them today in a
wide range of different colors, they continue  Using observations made through the
being called white goods. European Space Organisation’s SPHERE
instrument at the Very Large Telescope (VLT),
 The term contrasts with brown goods, which astronomers have now found Hygiea may
are relatively light electronic possibly be a dwarf planet.
consumer durables such as computers,
digital media players, TVs and radios.  If it qualifies, Hygiea will be the smallest dwarf
planet in the Solar System.
4. c
 The International Astronomical Union (IAU)
 The Cabinet Secretariat has recently notified sets four criteria for a dwarf planet,
rules reasserting the Ministry of Home Affairs
(MHA) as the authority that would decide on 1. It orbits around the Sun,
the imposition of AFSPA in the Union 2. It is not a moon,
Territories of Jammu and Kashmir (J&K) and
Ladakh. 3. It has not cleared the neighbourhood around
its orbit.
 Earlier, before the State of J&K was
bifurcated and downgraded it was the MHA 4. It has sufficient mass for its self-gravity to
or the Governor that was the designated overcome rigid body forces so that it assumes a
authority for notifying the Armed Forces hydrostatic equilibrium (nearly round) shape.
(Jammu and Kashmir) Special Powers Act,  Hygiea already satisfies three criteria.
1990 (21 of 1990).
7. b
 AFSPA, which empowers security forces to
conduct operations anywhere and arrest  The Finance Bill, 2017 introduced, electoral
anyone without a warrant, is in force in J&K bonds, an interest-free bearer bonds (like
since July 5,1990. Promissory Notes) that can be purchased
from specified branches of the State
5. d Bank of India only.
 With an average occupancy rate of 115% of  Issued in multiples of Rs 1,000, 10,000, 1 lakh,
their capacity, Indian jails continue to remain 10 lakh, and 1 crore.
congested and overcrowded, numbers in
the National Crime Records  Every political party registered under section
Bureau’s “Prison Statistics India – 29A of the Representation of the Peoples Act,
2017” report have revealed. 1951 and has secured not less than 1%
votes polled in the former Lok Sabha or
 In 16 of the 28 States covered in the report, legislative assembly elections are eligible to
occupancy rate was higher than 100%. receive electoral bonds.

www.shankariasacademy.com | www.iasparliament.com
11

 Electoral bonds are valid for 15 days from  Even the Flag Code of India, 2002 does
date of issuance and no payment is made to not impose prohibitions on a State flag.
any payee if bonds are deposited after 15 days.
 By implication, the Code provides space for a
 Buyers of the bonds have to submit full KYC State flag as long as it does not offend the
details at the time of buying. dignity and honour of the national flag.
 But the beneficiary political party is not 9. b
required to reveal the identity of the entity that
 Recently, Eritrea and Saint Kitts &
has given it the bond(s).
Nevis have become the newest members of
8. a the International Solar Alliance (ISA).
 The deadline for a final Naga peace accord  The two nations signed the ISA Framework
passed on recently. Agreement during the ISA assembly held in
New Delhi on October 31, 2019.
 Among the issues that have been contentious
are the demand for a separate Naga  Saint Kitts and Nevis is a twin island
constitution and use of the Naga flag, for country located in the eastern Caribbean
decades a symbol of Naga nationalism. Sea.
 The tallest leader of the Naga struggle, Dr A Z  With this, the total membership of the ISA has
Phizo, met M K Gandhi in Delhi on July 19, risen to 83.
1947.
 According to Naga historians, Gandhi agreed
that the Nagas would celebrate their
independence a day ahead of India, on August
14, 1947.
 To this day, Nagas across Nagaland, Manipur,
Assam and Arunachal Pradesh celebrate
August 14 as Independence Day.
 On August 14, the National Socialist Council of
Nagalim (Isak-Muivah), hoists the now
contentious Naga flag.
 Under the Constitution, a flag is not
enumerated in the Seventh Schedule.
 There is no explicit provision in the 10. b
Constitution for a state flag and also there
is no prohibition in the Constitution for the  AIP systems allow conventional
State to have its own flag. submarines to recharge their batteries
without surfacing for air.
 Article 51A ordains that every citizen shall
abide by the Constitution and respect its ideals  It enables them to remain underwater for
and institutions, the national flag, and the extend periods of time and not expose
national anthem. themselves to detection.

 There is no other provision regulating  AIP allows the non-nuclear submarines to


hoisting of flags, either by the States or by the operate without access to atmospheric oxygen.
public.  It allows diesel-electric submarines to
 It is clear that there is no prohibition under recharge their batteries without running
the Constitution to hoist any flag other than their engines.
the national flag.

www.shankariasacademy.com | www.iasparliament.com
12

 It is based on the combustion of stored oxygen 3) The National Waterway – 2 passes through
and ethanol to augment battery-powered
1. Sadiya
propulsion.
2. Silghat
 It can generate electricity, powering a
submarine to operate and also generate 3. Pandu
oxygen, lighting and amenities for crew. 4. Haldia
5. Dhubri
04-11-2019 Choose the correct answer from the given
1) Consider the following statements with option
respect to Project BHOG a. 1, 2 and 3 only
1. It is to encourage all Places of Worship b. 1, 2 and 4 only
(PoW) to adopt and maintain food
safety and hygiene in the preparation c. 1, 2, 4 and 5 only
of Prasad. d. 1, 2, 3 and 5 only
2. All the PoW that distribute Prasad are
henceforth required to obtain FSSAI
license/registration. 4) Consider the following statements with
respect to ‘Dhanush’
3. This is applicable to Hindu temples
only. 1. It is an attack Helicopter in the fleet of
IAF.
Which of the statements given above are
correct? 2. It is indigenously developed by
Ordnance Factory Board (OFB).
a. 1 and 2 only
Which of the statement(s) given above is/are
b. 2 and 3 only correct?
c. 1 and 3 only a. 1 only
d. 1, 2 and 3 b. 2 only
c. Both 1 and 2
2) Consider the following statements with d. Neither 1 nor 2
respect to Nanak Naam Lewa or a
Nanakpanthi
1. They are the people who believes in 5) Consider the following statements with
Guru Nanak and follows his teachings in respect to Lingdum Monastery
life. 1. It is a Buddhist pilgrimage site located
2. They are not limited to just one religion in Arunachal Pradesh.
(Sikhism) and can be from any religion. 2. It belongs to the Gelug school of
3. The core value of the philosophy is Mahayana Buddhism.
‘There is only one giver of life, one God’. Which of the statement(s) given above is/are
Which of the statements given above are correct?
correct? a. 1 only
a. 1 and 2 only b. 2 only
b. 2 and 3 only c. Both 1 and 2
c. 1 and 3 only d. Neither 1 nor 2
d. 1, 2 and 3

www.shankariasacademy.com | www.iasparliament.com
13

6) With respect to Indo-German Partnership 3. All the member countries of Shanghai


for Green Urban Mobility, sometimes seen in Cooperation Organization (SCO) except
the news recently, consider the following Pakistan will participate in this
statements: exercise.
1. Under the partnership, Germany will Which of the statement(s) given above is/are
spend 1 billion euros over a three-year correct?
period.
a. 1 only
2. The main aim of this partnership is to
b. 1 and 3 only
finance several environment friendly
urban mobility projects. c. 1, 2 and 3
Which of the statement(s) given above d. None
is/are incorrect?
a. 1 only
10) Consider the following statements with
b. 2 only respect to Alzheimer Disease
c. Both 1 and 2 1. ‘Tau’ a protein occurs in tangled
formations in the brain of Alzheimer
d. Neither 1 nor 2
patients, disrupts the ability of neurons
to communicate with one another in the
brain.
7) NEO Surveillance Mission is a planned
space-based telescope to search for near-Earth 2. ‘Oligomannate (GV-971)’ is a newly
asteroids developed by which of the following approved drug for Alzheimer Disease.
space agencies?
Which of the statement(s) given above is/are
a. ESA correct?
b. NASA a. 1 only
c. JAXA b. 2 only
d. ROSCOSMOS c. Both 1 and 2
d. Neither 1 nor 2
8) Dustlik-2019 is the first-ever joint military
exercise between India and which of the
following countries? Answers
a. Uzbekistan 1. a

b. Kazakhstan  Recently Tamil Nadu State Food Safety and


Drug Administration has told temples to
c. Kyrgyzstan obtain the Blissful Hygiene Offering to God
d. Tajikistan (BHOG) quality certificate f0r the temple
prasadam.
 FSSAI initiated Project BHOG (Blissful
9) Consider the following statements with Hygienic Offering to God) for Food Safety in
respect to SCOJtEx-2019 Places of Worship.
1. It is a Shanghai Cooperation  It is to encourage all Places of Worship (PoW)
Organization’s (SCOs) joint exercise on to adopt and maintain food safety and hygiene
urban earthquake search and rescue. in the preparation of Prasad.
2. Indian Army in cooperation with Indian
 All the PoW that distribute Prasad, including
Coast Guard is hosting the event.
vendors that sell Prasad in the vicinity of the

www.shankariasacademy.com | www.iasparliament.com
14

PoW, are henceforth required to obtain FSSAI


license/ registration.
 They also need to follow the sanitary and
hygiene practices as per Food Safety and
Standards regulations.
 All places of worship, including temples,
mosques and gurudwaras, are covered by
the project.
2. d
 The Kartarpur Corridor can’t be limited to
Sikhs, as Punjab Chief Minister urged
Pakistan to waive the requirement of a
passport “to all citizens of secular India” and
not just to Sikhs.
 Any person who believes in Guru Nanak and 4. b
follows his teachings in life, irrespective of
belonging to any religion is a Nanak  The Indian Army, which began inducting the
Naam Lewa or a Nanakpanthi. indigenously upgraded Dhanush artillery
guns, will have the first regiment in place by
 During his four Udaasis (travels), Guru March 2020.
Nanak had spread the message of oneness and
people from different faiths had become his  It is an indigenously developed artillery
followers. gun and upgraded version of the Swedish
Bofors gun procured by India.
 Nanak simply cannot be limited to just one
religion (Sikhism) because the core value of his  It is compatible with all North Atlantic Treaty
philosophy is, Organisation (NATO) 155mm ammunition
systems.
1. “Sabhna jiya ka ik daata” (There is only
one giver of life, one God) and  It is also referred to as Desi Bofors developed
by Ordnance Factory Board (OFB) and
2. “Na koi Hindu, na Musalman” (There is
manufactured by Jabalpur-based Gun Carriage
no Hindu, no Muslim).
Factory (GCF).
 The syncretic faith of
5. d
the ‘Nanakpanthi’ communities continues to
be practiced in the Indus belt.  Lingdum Monastery is a Buddhist
pilgrimage center following the Zurmang
 They do not have a defining line drawn
Kagyu sect of the ancient school of Tibetan
between cultures and faiths.
Buddhism.
 People might identify them as Hindus or Sikhs,
 It is under the spiritual direction of H. E.
but Guru Nanak is the essential fabric of their
Zurmang Gharwang Rinpoche, who is the 12th
existence.
successive successor of the Kagyu sect.
3. d
 President Ram Nath Kovind recently visited
 Sadiya-Dhubri stretch of the river Lingdum monastery in Ranka, Sikkim.
Brahmaputra (891 Km) is declared as
6. a
NationalWaterway 2.
 German Chancellor Angela Merkel has recently
 Allahabad-Haldia stretch of the Ganga-
said her country will spend 1 billion euros
Bhagirathi-Hooghly river (1620 Km) is
($1.12 billion) on green urban mobility projects
declared as NationalWaterway 1.
conceived under the new German-Indian
partnership.

www.shankariasacademy.com | www.iasparliament.com
15

 This will be over a five-year period.  The main focus of Joint Exercise on Urban will
be to test the region’s preparedness and
 The funds will be used to finance resilience towards effective activation of Inter-
several environment friendly governmental interaction for immediate
projects such as the introduction of electric response.
buses to replace the ones running on diesel for
public transport in urban centres.  All the eight-member countries of SCO
namely China, India, Kazakhstan, Kyrgyzstan,
 She also said, both the countries will earmark Pakistan, Russia, Tajikistan and Uzbekistan
200 million Euros to reform bus sector will participate in this exercise.
in Tamil Nadu.
10. c
7. b
 Recently China approves first new Alzheimer’s
 NEO Surveillance Mission is a planned Drug ‘Oligomannate (GV-971)’ in 17 Years.
space-based telescope to search for near-Earth
asteroids under development by NASA.  Oligomannate, which uses extract from
marine brown algae as raw material,
 The Jet Propulsion Laboratory will lead received a green light to treat mild-to-
development of the mission under NASA’s moderate level AD.
planetary defence program.
 Tau is a protein found in the brain, but under
 NEO Surveillance Mission is based on the certain conditions, it will clump together into
NEOCam mission that was a finalist in the tangles that injure nearby tissue and may lead
previous competition for Discovery-class to cognitive decline.
planetary science missions.
 Tau occurs in tangled formations in the brain
 Launch is planned no earlier than 2025. of Alzheimer patients, disrupts the ability of
8. a neurons to communicate with one another in
the brain.
 The first-ever India-Uzbekistan joint
military exercise - Dustlik-2019 will begin at
Chirchiq Training Area near Tashkent.
05-11-2019
 The exercise will focused on counter-terrorism. 1) Consider the following statements with
 The exercise will enable sharing of best respect to Brown Blotched Bengal Tree Frog
practices and experiences between the Armed 1. It is commonly found in the deep
Forces of the two countries and will lead to jungles of the state of West Bengal.
greater operational effectiveness.
2. It belonged to the genus Polypedates
9. a and its body colour is yellowish-brown
 Union Home Minister will inaugurate the to greenish-brown.
Shanghai Cooperation Organization Joint Which of the statement(s) given above
Exercise on Urban Earthquake Search and is/are incorrect?
Rescue -SCOJtEx-2019 in New Delhi.
a. 1 only
 National Disaster Response Force
(NDRF) is hosting the event. b. 2 only

 The aim of the exercise is to rehearse the c. Both 1 and 2


disaster response mechanism, share d. Neither 1 nor 2
knowledge, experience and technology for
mutual coordination.
 This stimulation exercise will also provide an 2) Consider the following statements with
opportunity to enhance the coordination and respect to Drukpa Order
co-operation involving multi-agency 1. Drukpa Lineage is a Buddhist tradition
operations in an earthquake scenario. that began in the Himalayas.

www.shankariasacademy.com | www.iasparliament.com
16

2. The founder of the Drukpa order flouted a. 1 only


centuries of Buddhist tradition that
b. 2 only
barred nuns from physical exercise.
c. Both 1 and 2
Which of the statement(s) given above is/are
correct? d. Neither 1 nor 2
a. 1 only
b. 2 only 6) Danakil Depression which was in news
recently is located in?
c. Both 1 and 2
a. Ethiopia
d. Neither 1 nor 2
b. Sudan
c. Mali
3) Consider the following statements with
respect to Gaofen-7 which was in news recently d. Eritrea
1. It is an optical satellite for mapping
purposes, surveying, and
Geoinformation. 7) Consider the following statements with
respect to South Asia Co-operative
2. It was launched by The Roscosmos. Environment Programme (SACEP)
Which of the statement(s) given above is/are 1. It is an inter-governmental organization
correct? aims to promote and support
protection, management and
a. 1 only
enhancement of the environment in the
b. 2 only region.
c. Both 1 and 2 2. Its member countries include
Afghanistan, Bangladesh, Bhutan,
d. Neither 1 nor 2
India, Maldives, Nepal, Pakistan and Sri
Lanka.
4) ICEDASH and ATITHI which was in news is 3. The 14th meeting of the SACEP
related to? Governing Council was held in March
2018 in Colombo.
a. To nurture entrepreneurship and create
business models for low cost products Which of the statement(s) given above is/are
correct?
b. A desi version e-commerce marketing platform
a. 1 only
c. For improved monitoring and pace of Customs
clearance of imported goods b. 2 and 3 only
d. Online portal for income certificate c. 1 and 3 only
d. 1, 2 and 3
5) Consider the following statements with
respect to Aadi Mahotsav
8) Consider the following statements with
1. It is a joint initiative of Ministry of respect to Dark Matter
Tribal Affairs and TRIFED.
1. The presence of dark matter is indicated
2. The theme of the festival is “A by its gravitational effects on visible
Celebration of the Spirit of Tribal Crafts, bodies.
Culture and Commerce”.
2. Ninety per cent of our Milky Way galaxy
Which of the statement(s) given above is made up of this visible dark matter.
is/are not correct?

www.shankariasacademy.com | www.iasparliament.com
17

Which of the statement(s) given above  These tree frogs were


is/are incorrect? found commonly in urban habitats of
West Bengal.
a. 1 only
b. 2 only  Specimens were not discovered from deep
jungles but from residential areas in two
c. Both 1 and 2 districts of West Bengal.
d. Neither 1 nor 2  It was established that the mid-sized tree frog
as the 26th species under the
genus Polypedates, found throughout south
9) With respect to Virus, consider the following and southeast Asia.
statements:
 The new species has been named Brown
1. By itself, the virus is a lifeless particle. Blotched Bengal Tree Frog ( Polypedates
2. It reproduces very fast in the cell it has bengalensis) from the series of six to nine dark
invaded. brown blotches that extend laterally from
behind the frog’s eye to the vent.
3. It consists of an outer coat of protein
and an inner core of either DNA or RNA  The frog’s body colour is yellowish-brown
or both. to greenish-brown.

Which of the statements given above are 2. c


correct?  Recently, Union Minister of State for Tourism
a. 1 and 2 only and Culture congratulates the Kung Fu
Nuns for receiving Asia Society’s ‘Game
b. 1 and 3 only Changer Award’ for their heroic human rights
c. 2 and 3 only activism

d. 1, 2 and 3  The Kung Fu Nuns of the Drukpa


Order are the fearless daughters of India who
represent a new generation of
10) Consider the following statements with Buddhists using spirituality to inspire real-
respect to Predatory Pricing life action & impact.
1. It is specifically defined as sale or goods  They are a strong community of 700 nuns and
or services at a price below the cost of growing, with a thousand year-old Drukpa
production, with a view to reduce or legacy.
eliminate competition.
 They belong to the Drukpa Lineage, a
2. India does not have any legislation on thousand-year-old Buddhist
what constitutes predatory pricing. tradition that began in the Himalayas.
Which of the statement(s) given above is/are  The Gyalwang Drukpa, the spiritual leader
correct? and founder of the Drukpa order, encouraged
his nuns to train in Kung Fu to build
a. 1 only
confidence as leaders.
b. 2 only
 In doing so, he flouted centuries of
c. Both 1 and 2 Buddhist tradition that barred nuns from
physical exercise.
d. Neither 1 nor 2
 They are the “only Buddhist nuns” in the world
to practice KungFu.
Answers  The nuns use their martial arts expertise to
1. a challenge gender roles in India’s conservative
culture.
 Bengal’s cities yield new species of tree frog.

www.shankariasacademy.com | www.iasparliament.com
18

3. a Commerce”, which represents the basic ethos


of tribal life.
 Gaofen 7 is a a new remote sensing
satellite for mapping purposes for  The festival, will feature exhibition-cum-sale of
National Administration of Surveying, tribal handicrafts, art, paintings, fabric,
Mapping and Geoinformation of China jewellery and much more.
(NASMG).
6. a
 It was launched by China National Space
 The Danakil Depression in
Administration (CNSA).
northeastern Ethiopia is one of the world’s
 The new satellite was developed by the hottest places, as well as one of its lowest, at
‘Chinese Academy of Aerospace Science and 100 metres below sea level.
Technology Group’.
 A new study published in ‘Nature Ecology &
 The development of the Gaofen-7 has achieved Evolution’, says that active and naturally
a breakthrough in sub-meter level 3D occurring life cannot be sustained at Danakil.
mapping camera technology, meeting the
 It identifies two barriers for life,
highest mapping accuracy requirement among
the Gaofen series Earth observation satellites. 1. Magnesium-dominated brines that cause cells
to break down
4. c
2. An environment having simultaneously very
 Finance Minister Launches two new IT
low pH and high salt, a combination that
Initiatives, ICEDASH & ATITHI.
makes adaptation highly difficult.
 It is for improved monitoring and pace of
 At the northern end of the Great Rift Valley,
Customs clearance of imported
and separated by live volcanoes from the Red
goods and facilitating arriving international
Sea, the plain was formed by the evaporation
passengers
of an inland water body.
 ICEDASH, Ease of Doing Business
 All the water entering Danakil evaporates, and
monitoring dashboard of the Indian Customs
no streams flow out from its extreme
helping public see the daily Customs clearance
environment. It is covered with more than 10
times of import cargo
lakh tonnes of salt.
 ATITHI, Easy to use mobile app for 7. d
international travelers to file the Customs
declaration in advance.  It is an inter-governmental organization which
was established in 1982.
 Passengers can use this app to file declaration
of dutiable items and currency with the Indian  Its member countries Afghanistan,
Customs even before boarding the flight to Bangladesh, Bhutan, India, Maldives,
India. Nepal, Pakistan and Sri Lanka.
 This dashboard has been developed by CBIC in  It aims to promote and support protection,
collaboration with NIC. management and enhancement of the
environment in the region.
5. d
 The 14th meeting of the SACEP Governing
 The Delhi Aadi Mahotsav will be organised at
Council was held in March last year
Dilli Haat, INA, New Delhi between 16th to
in Colombo.
30th November, 2019.
8. b
 It is a joint initiative of Ministry of Tribal
Affairs and Tribal Cooperative Marketing  It is now believed that there exists more matter
Development Federation of India (TRIFED).. in the Universe than is visible to us as stars,
nebulae, galaxies etc.
 The theme of the festival is “A Celebration of
the Spirit of Tribal Crafts, Culture and

www.shankariasacademy.com | www.iasparliament.com
19

 The matter that astronomers believe exists but  ‘Predatory pricing’ figures in the section on
which they have not been able to detect is abuse of dominant position by a market player.
known as dark matter.
 It expressly forbids any enterprise or group
 The presence of dark matter is indicated by from ‘abusing its dominant position’ in the
its gravitational effects on visible market, either by imposing unfair conditions
bodies so though it has mass, nobody yet or an unfair and discriminatory price —
knows what it is made up of. including predatory price — resulting in denial
of market access.
 Ninety per cent of our own galaxy, the Milky
Way is believed to be made up of  Predatory price is specifically defined as sale
this invisible matter. or goods or services at a price below
the cost of production, with a view to
9. d
reduce or eliminate competition.
 By itself, the virus is a lifeless
particle consisting of an outer coat of protein
and an inner core of either DNA 06-11-2019
(deoxyribonucleic acid) or RNA (ribonucleic
acid) or both. 1) Consider the following statements with
respect to Wastelands Atlas - 2019
 It lacks some of the substances needed by an
organism to live and reproduce on its own. 1. It is the fifth edition of Wastelands Atlas
which provides robust geospatial
 So in that sense it is not a living thing. information on wasteland.
 However, when it enters a living cell it uses the 2. It also provides State and district wise
genes of the cell it has invaded to reproduce. distribution of different categories of
wastelands area in the country.
 It comes alive, so to say when it has parasitized
a living cell. 3. It is published jointly by the Department
of Land Resources and National Remote
 It reproduces very fast in the cell it has Sensing Centre.
invaded.
Which of the statement(s) given above is/are
 In less than half an hour it produces about 200 correct?
copies of itself.
a. 1 only
 These new viruses break out of the cell and
each of them latch on to an adjoining cell and b. 1 and 2 only
reproduce and in a short while there are c. 2 and 3 only
thousands of the viruses of that particular kind
in that part of the host’s body. d. 1, 2 and 3

10. a
2) National Crisis Management Committee
 Predatory pricing is a strategy whereby a
(NCMC) is headed by?
Goliath in an industry with deep pockets prices
its goods or services at rock-bottom levels, so a. Prime Minister
that no rivals can compete with it.
b. Cabinet Secretary
 Once everyone else runs up huge losses and is
c. Union Home Minister
forced out of the business, the Goliath makes
hay by withdrawing the freebies and fleecing d. Union Defence Minister
consumers.
 In India, the Competition Act of 2002 lays
down the ground-rules on what constitutes
predatory pricing.

www.shankariasacademy.com | www.iasparliament.com
20

3) Consider the following statements with Which of the statement(s) given above is/are
respect to Deep Ocean Mission correct?
1. It will help India harness various living a. 1 only
and non-living (water, mineral and
b. 2 only
energy) resources from the seabed and
deep water. c. Both 1 and 2
2. Under the Mission, India will also study d. Neither 1 nor 2
climate change in the deep oceans.
3. Ministry of Earth Sciences (MoES) is the
nodal agency for the mission. 6) Consider the following statement with
respect to Trade deficit
Which of the statements given above are
correct? 1. It represents an outflow of domestic
currency to foreign markets.
a. 1 and 2 only
2. Depreciation of a currency may
b. 1 and 3 only improves the trade balance.
c. 2 and 3 only Which of the statement(s) given above is/are
correct?
d. 1, 2 and 3
a. 1 only
b. 2 only
4) Consider the following statements
c. Both 1 and 2
1. It is also called as tannumai.
d. Neither 1 nor 2
2. It is considered to be the most
melodious and auspicious percussion
instrument among all the Avanaddha
Vadyas. 7) Consider the following statements with
respect to First Battle Of Panipat (1526)
3. Indian Vice President has recently
released a Monograph about this 1. The ‘Tulughma’ and the ‘Araba’ are the
instrument. war tactics followed by Babur.

Identify the instrument that correctly matches 2. It introduced gunpowder firearms and
with the above description: field artillery to Indian battlefields.

a. Mridangam Which of the statement(s) given above


is/are not correct?
b. Shehnai
a. 1 only
c. Kamaicha
b. 2 only
d. Pullankuzhal
c. Both 1 and 2
d. Neither 1 nor 2
5) Consider the following statements with
respect to RCEP agreement
1. It is a proposed free-trade agreement 8) Consider the following statements with
among 10 APEC countries and Australia, respect to Voyager 2
China, India, Japan, New Zealand and 1. It is the first human-made object to to
Korea. leave the heliosphere to enter into the
2. India has recently signed this Interstellar medium (ISM).
agreement. 2. It is the only spacecraft to have visited
all four gas giant planets Jupiter,
Saturn, Uranus and Neptune.

www.shankariasacademy.com | www.iasparliament.com
21

Which of the statement(s) given above is/are  The Atlas, published jointly by Department
correct? of Land Resources and National
Remote Sensing Centre, provides robust
a. 1 only
geospatial information on wasteland.
b. 2 only
 It is helpful in rolling back the wastelands for
c. Both 1 and 2 productive use through various land
d. Neither 1 nor 2 development programmes and schemes.
 Wastelands Atlas-2019 also provides State and
district wise distribution of different categories
9) Consider the following of wastelands area in the country and will be
1. Accounting and Finance helpful in addressing land degradation.

2. Communications 2. b

3. Legal 3. d

4. Health & Wellness  The design for the manned submersible’s


sphere has been successfully developed by
5. Environment ISRO recently.
Which of these comes under Champion  The submersible vehicle is expected to travel to
Sectors in Services? a depth of approximately 6,000 metres under
a. 1, 2 and 3 only the sea for various studies, whereas
submarines can reach only about 200 metres.
b. 1, 2 and 4 only
 This initiative is a part of the Deep Ocean
c. 1, 2, 4 and 5 only Mission.
d. 1, 2, 3, 4 and 5  As part of the ambitious Rs 10,000 crore Deep
Ocean Mission, India will also study climate
change in the deep oceans.
10) Consider the following statements with
respect to Bru Tribes or Riang  Studying climate change, marine biodiversity
and survey for compounds like hydrocarbons
1. They are one of the a Particularly and minerals are part of the deep ocean
Vulnerable Tribal Group (PVTG). mission.
2. They follow a traditional life patterns  Ministry of Earth Sciences (MoES) is the
includes shifting cultivation, use of Nodal Agency for this mission.
primitive tools, habitation in
inaccessible areas.  The mission proposes to explore the deep
ocean similar to the space exploration started
Which of the statement(s) given above is/are
by ISRO.
correct?
a. 1 only  Underwater robotics and ‘manned’
submersibles are key components of the
b. 2 only Mission which will help India harness various
living and non-living (water, mineral and
c. Both 1 and 2
energy) resources from the seabed and deep
d. Neither 1 nor 2 water.
4. a
Answers  Indian Vice President has recently released the
Monograph on mridangam: “Musical
1. d Excellence of Mridangam”
 Agriculture Minister releases 5th edition 5. d
of Wastelands Atlas – 2019.

www.shankariasacademy.com | www.iasparliament.com
22

 The Regional Comprehensive Economic used to easily manoeuvre the canons. These
Partnership (RCEP) is a trade deal that is two tactics made Babur’s artillery lethal.
currently under negotiation among 16
8. b
countries — the 10 member countries of the
Association of Southeast Asian Nations  NASA's Voyager 2 spacecraft has crossed the
(ASEAN), and the six countries with which the elusive boundary that marks the edge of the
ASEAN bloc has free trade agreements (FTA). Sun's realm and the start of Interstellar space
(ISM).
 India has decided recently to not join the
Regional Comprehensive Economic  This makes Voyager 2 the second human-
Partnership (RCEP) agreement over India's made object to journey out of the Sun's
concerns not being addressed in the deal. influence, following the US space
agency's Voyager 1's solar exit in 2012.
6. c
 Voyager 2 is the only spacecraft to have
 A trade deficit of a country shows the
visited all four gas giant planets -
difference between what it earns from its
Jupiter, Saturn, Uranus and Neptune.
exports and what it pays for its imports.
 It also discovered 16 moons, the phenomena
 If this number is in negative, that is, the total
like Neptune's mysteriously transient Great
value of goods imported is more than the total
Dark Spot, the cracks in Europa's ice shell, and
value of goods exported by the country, then it
ring features at every planet.
is referred to as a “trade deficit”.
 Voyager 1 visited and got gravity assists from
 Depreciation of a currency, a weaker
Jupiter and Saturn before heading off toward
currency makes imports more expensive and
the edge of the solar system.
exports cheaper and it may improves the trade
balance.  Voyager 2 swung past Jupter, Saturn,
7. d Neptune, and Uranus.

 The First Battle of Panipat was fought between


the invading forces of Babur and the Lodi
Empire, which took place on 21 April 1526.
 It marked the beginning of the Mughal
Empire.
 Mughal forces of Babur, the Timurid ruler of
Kabulistan, defeated the much larger ruling
army of Ibrahim Lodi,Sultan of Delhi.
 The new war tactics introduced by
Babur were the “Tulughma” and the
“Araba”.
 Tulughma meant dividing the whole army into 9. d
various units, viz. the Left, the Right and the
Centre.  Recently, 5th Global Exhibition on Services
(GES) was launched in New Delhi.
1. The Left and Right divisions were further
subdivided into Forward and Rear divisions.  The event is organised by Department of
Commerce, Ministry of Commerce and
 The Centre Forward division was then Industry in partnership with Services Export
provided with carts (Araba) which were Promotion Council and the Confederation of
placed in rows facing the enemy and tied to Indian Industry.
each other with animal hide ropes.
 This initiative will enhance the
 Behind them were placed cannons protected competitiveness of India's service sectors
and supported by mantelets which could be

www.shankariasacademy.com | www.iasparliament.com
23

through the implementation of focused and 2. Article 28 of the agreement enables


monitored Action Plans. parties to withdraw from the agreement
with immediate effect.
 The objective is to give focused attention to
the following 12 identified Champion Which of the statement(s) given above is/are
Services Sectors for promoting their correct?
development and realizing their potential:
a. 1 only
1. IT & ITES
b. 2 only
2. Tourism and Hospitality Services,
c. Both 1 & 2
3. Accounting & Finance
d. Neither 1 nor 2
4. Infra & Construction
5. Transport & Logistics
2) Tapan Ray Committee sometimes seen in
6. Communications the news is,
7. Education a. To revamp the he FPI regime
8. Media & Entertainment b. To spell out revival plans for Telecom sector
9. Health & Wellness c. To review the supervisory framework for Core
Investment Companies
10. Legal
d. To set guidelines for transfer of RBI’s surplus
11. Banking, Financial & Insurance funds to government
12. Environment.
 Boosting these 12 champion sectors will 3) Consider the following statements with
help in pushing overall economic growth and respect to Mission Innovation (MI)
job creation.
1. It is a global initiative working to
10. c accelerate clean energy innovation.
 The Brus, also called Reangs, are scattered 2. India is a founding member and a
across Assam, Mizoram and Tripura. steering committee member.
 Recognised by the Constitution of India as a Which of the statement(s) given above is/are
Particularly Vulnerable Tribal Group (PVTG) correct?
 PVTG status owing to their traditional life a. 1 only
patterns (shifting cultivation, use of primitive
tools, habitation in inaccessible areas) b. 2 only
c. Both 1 & 2
 The Centre and the state governments of
Tripura and Mizoram has signed an agreement d. Neither 1 nor 2
for repatriation of Bru community from
Tripura to Mizoram.
4) Consider the following statements with
respect to Ethanol
07-11-2019 1. At 95% purity, ethanol is used for
1) Consider the following statement with blending with petrol.
respect to Paris Agreement 2. Ethanol Blended Petrol releases fewer
1. All the signatory parties emissions into the environment and is
formally ratified the agreement. considered cleaner in nature.

www.shankariasacademy.com | www.iasparliament.com
24

3. The new Biofuel Policy 2018 has fixed a 7) Consider the following statements with
target of achieving 20% ethanol respect to Harmonized System (HS) Code
blending with petrol by 2030.
1. It is a six digit identification code used
Which of the statement(s) given above is/are by customs officers to clear every
correct? commodity that enters or crosses any
state borders within India.
a. 1 and 2 only
2. It was developed by Bureau of Indian
b. 2 and 3 only
Standards (BIS).
c. 1 and 3 only
Which of the statement(s) given above is/are
d. 1,2 and 3 only correct?
a. 1 only
5) Consider the following statements with b. 2 only
respect to ‘Chandra’s Atmospheric
c. Both 1 and 2
Composition Explorer-2’ (CHACE-2)
d. Neither 1 nor 2
1. It is a Quadrupole Mass Spectrometer
(QMA) capable of scanning the lunar
neutral exosphere.
8) Consider the following statements with
2. Its primary objective is to carry out an respect to Shala Darpan portal
in-situ study of the composition and
1. It is an end to end e-Governance school
distribution of the lunar neutral
automation and management system for
exosphere.
Navodaya Vidyalaya Samiti (NVS).
Which of the statement(s) given above is/are
2. It was launched by Ministry of Human
correct?
Resources and Development (MHRD).
a. 1 only
Which of the statement(s) given above is/are
b. 2 only correct?
c. Both 1 and 2 a. 1 only
d. Neither 1 nor 2 b. 2 only
c. Both 1 and 2
6) Consider the following statements with d. Neither 1 nor 2
respect to IndAIR
1. It is India’s first interactive online
9) Fordow Nuclear Plant sometimes seen in
repository that aims to make air quality
the news recently was located in which of the
research available to everyone.
following countries?
2. It was launched by the Ministry of
a. Iran
Environment, Forest and Climate
Change (MoEFCC). b. Russia
Which of the statement(s) given above is/are c. China
correct?
d. Saudi Arabia
a. 1 only
b. 2 only
c. Both 1 and 2
d. Neither 1 nor 2

www.shankariasacademy.com | www.iasparliament.com
25

10) Consider the following statements with 2. c


respect to Freedom on the Net’ (FoTN) Report,
2019  The Reserve Bank of India (RBI) has
constituted a working Committee to review
1. India was among the worst 10 countries the regulatory guidelines and
in world for internet, digital media supervisory framework applicable for
freedom according to the report. core investment companies (CIC).
2. It was published by the International  Recently the Committee suggested norms for
Telecommunication Union (ITU). CICs.
Which of the statement(s) given above is/are  The panel advices to limit CIC to two levels is
correct? another reform for better financing
a. 1 only arrangement.

b. 2 only
c. Both 1 and 2
d. Neither 1 nor 2

Answers
1. d
 At COP 21 in Paris, on 12 December 2015,
Parties to the UNFCCC reached a landmark
agreement to combat climate change.
 There are 197 signatories to the Paris
Agreement. But 10 nations remain yet to
ratify, including two major emitters.
 As of October 2019, the countries yet to
formally ratify the agreement are Angola,
Eritrea, Iran, Iraq, Kyrgyzstan, Lebanon,
Libya, South Sudan, Turkey, and Yemen.
 Article 28 of the agreement enables parties to
withdraw from the agreement,
3. c
1. At any time after three years from the date on
which this Agreement has entered into force  Recently Indian government launched
for a Party, that Party may withdraw from this the Mission Innovation (MI) Face to
Agreement by giving written notification to the Face meeting of Innovation Challenges
Depositary. (IC).

2. Any such withdrawal shall take effect upon  Department of Biotechnology (DBT) under
expiry of one year from the date of Ministry of Science & Technology is nodal
receipt by the Depositary of the notification agency coordinating and steering activities of
of withdrawal, or on such later date as may be Mission Innovation (MI) in India.
specified in the notification of withdrawal.
 Mission Innovation is a global
3. Any Party that withdraws from the Convention initiative working to accelerate clean energy
shall be considered as also having withdrawn innovation.
from this Agreement.
 It was launched on 30 November 2015 by 20
countries during COP-21.

www.shankariasacademy.com | www.iasparliament.com
26

 MI currently comprises of 24- lunar neutral exosphere in the range of 1-300


member countries and the European amu (atomic mass unit).
Commission.
 Its primary objective is to carry out an in-
 The main objective of MI is to identify major situ study of the composition and distribution
gap areas in Clean Energy Innovation and how of the lunar neutral exosphere and its
to address the issues beyond 2020 to make MI variability.
more impactful.
 Argon-40 also known as 40Ar is an isotope of
 India is one of the three countries responsible the noble gas Argon. Argon is third most
for crystalizing the idea of global initiative. abundant gas in the Earth’s Atmosphere.
 Being a founding member and as steering  ISRO states that 40Ar is a major constituent of
committee member, India has played an the lunar exosphere.
active role in all Mission Innovation activities.
 40Ar is a condensable gas and reacts
4.b differently at various temperatures and
pressures.
 Recently, Ministry of Environment and Forests
announced that mills would not require  It condenses during lunar night and after lunar
separate environmental clearance to produce dawn, the gas then again starts getting
additional ethanol from B-heavy molasses. released to the lunar exosphere.
 At 95% purity, it is called rectified 6. a
spirit and is used as the intoxicating
 The National Environmental
ingredient in alcoholic beverages.
Engineering Research Institute
 At 99%-plus purity, ethanol is used (NEERI) along with the Council of
for blending with petrol. Scientific and Industrial Research
(CSIR) has launched India’s first web
 Using ethanol fuel to power automobiles repository documenting air quality studies
results in significantly low levels of toxins in done in the last 60 years.
the environment.
 The Indian Air quality Interactive
 For environmental purposes, ethanol is less Repository (IndAIR) has archived
harmful than unblended gasoline. Carbon approximately 700 scanned materials from
monoxide production from ethanol fuel is pre-Internet era (1950-1999), 1,215 research
significantly lower. articles, 170 reports and case studies and 100
 The current consumption of fuel sets 3300 cases, to provide the history of air pollution
million litres of ethanol requirement to research and legislation.
achieve 10% of blending target by 2022.  Such a repository on air pollution is one of the
 The National Policy on Biofuels-2018 sets first in the world.
ambitious target of 20% in blending by 7. d
2030.
 HS Stands for Harmonized System and it is
5. c a six digit identification code.
 Recently, Chandrayaan-2 has  It was developed by the WCO (World
detected ‘Argon-40’ in the lunar exosphere Customs Organization) and custom
with the help of CHACE-2. officers use HS Code to clear every commodity
 The Chandra’s Atmospheric Composition that enters or crosses any international
Explorer-2 (CHACE-2) payload aboard border.
the Chandrayaan-2 orbiter.  In a move to give impetus to the export of
 It is a neutral mass spectrometer-based Khadi, the Ministry of Commerce and
payload which can detect constituents in the Industry has recently allocated a separate
Harmonized System (HS) code for this fabric.

www.shankariasacademy.com | www.iasparliament.com
27

 Allocation of HS code to the khadi will help the  Iran insists that the latest move is not a
government to plan the export strategies violation of the nuclear deal, but is based on
effectively. the Articles 26 and 36 of the agreement.
 Earlier, Khadi did not have exclusive HS code 10. d
and all the export data regarding this fabric
 Pakistan is among the worst 10 countries in
used to come under the normal fabric head.
the world for internet and digital media
8. c freedom, according to a report 'Freedom on
the Net' (FoTN) for the year 2019 released by
 ShaalaDarpan is an end to end e- the Freedom House, an international
Governance school automation and internet rights group.
management system for Navodaya
Vidyalaya Samiti (NVS).  The report titled 'The Crisis of Social Media',
recording an overall decline in global internet
 This single integrated platform has been freedom between June 2018 and May 2019.
developed for information sharing and
knowledge dissemination for the 22000  The watchdog in its report placed Pakistan at
employees and over 2 lakh students across 26, out of 100, one place down from last year's
schools and offices of ranking.
NavodayaVidyalayaSamiti.
 India scores 55 out of 100 and listed under
 It has been implemented at Navodaya the category of “Partly Free Countries”.
Vidyalaya Samiti as the first major initiative to
enable automation of all activities of the
country’s largest residential schooling system 08-11-2019
through a single umbrella - 636 schools, 8
Regional Offices, 8 NLIs and Head Quarter. 1) The Agreement on Reciprocal Logistics
Support (ARLS) sometimes seen in the news
 The best part is this portal has been designed recently is a planned defence logistics pact
from OPEN SOURCE TECHNOLOGY and between India and?
hence the cost incurred was very low and up-
dation cost in future will remain low. a. USA

 The Navodaya Vidyalayas are co-educational b. Israel


residential schools established by c. Japan
the Navodaya Vidyalaya Samiti which is
an autonomous organization under d. Russia
MHRD to provide quality modern education.
9. a 2) Consider the following statements with
 Iran has recently stepped up activity at its respect to India Justice Report 2019
underground Fordow nuclear plant. 1. It is the first ever ranking of Indian
States based on justice delivery released
 With the presence of inspectors from the
by the Ministry of Social Justice and
International Atomic Energy Agency (IAEA),
Empowerment.
Iran started injecting uranium gas into
centrifuges in Fordow, state television 2. The report has been prepared based on
reported. the four pillars of justice delivery —
police, judiciary, prisons and legal aid.
 French President Emmanuel Macron called
Iran's latest move 'grave', saying it explicitly 3. Maharashtra has topped the list of 18
signalled Iran's intent for the first time to leave large-medium States in the overall
the deal - formally known as the Joint ranking.
Comprehensive Plan of Action.
Which of the statement(s) given above is/are
correct?
a. 3 only

www.shankariasacademy.com | www.iasparliament.com
28

b. 1 and 2 only Which of the statement(s) given above is/are


correct?
c. 2 and 3 only
a. 1 only
d. 1, 2 and 3
b. 1 and 2 only
c. 1 and 3 only
3) “India Internet 2019 Report” sometimes
seen in the news recently was released by? d. 1, 2 and 3
a. Google
b. Facebook 6) Consider the following statement with
respect to ‘Financial Stability and Development
c. Internet and Mobile Association of India
Council’ (FSDC)
d. Ministry of Electronics and Information
1. It is an apex-level statutory body to
Technology
strengthen and institutionalize the
mechanism for maintaining financial
stability.
4) Consider the following statements with
respect to Acute Respiratory Infections (ARI) 2. The FSDC sub-committee is chaired by
the Governor of RBI.
1. It is a serious ailment that prevents
normal breathing function. Which of the statement(s) given above is/are
correct?
2. According to the National Health
Profile, 2019 released recently, ARI a. 1 only
accounted for more than half of
b. 2 only
mortality in the communicable disease
category. c. Both 1 and 2
Which of the statement(s) given above is/are d. Neither 1 nor 2
correct?
a. 1 only
7) Consider the following statements with
b. 2 only respect to the Feni river
c. Both 1 and 2 1. It is a trans-boundary river originates in
the Meghalaya, flows through Tripura
d. Neither 1 nor 2
and then enters Bangladesh.
2. India and Bangladesh signed an MoU
5) With respect to Documentation for a drinking water supply scheme for
Identification Number (DIN) Sabroom town in Tripura.
system sometimes seen in the news recently,
Which of the statement(s) given above is/are
consider the following statements:
correct?
1. It aims to bring transparency and
a. 1 only
accountability in the indirect tax
administration through widespread use b. 2 only
of information technology.
c. Both 1 and 2
2. It was launched by the Central Board of
d. Neither 1 nor 2
Indirect Taxes (CBIC).
3. The Government has already executed
the DIN system in the direct tax
administration.

www.shankariasacademy.com | www.iasparliament.com
29

8) Samudra Shakti is a joint Naval Exercise  It would be crucial for joint exercises.
between
 The air force too will benefit by finding it
a. India - Indonesia easier to deploy aircraft for the same purpose.
b. India –France  This access will also be for ports in the Russian
c. India – Saudi Arabia part of the Arctic, allowing access to energy
resources there.
d. India – Malaysia
 Russia, on the other hand, will be able to
access Indian ports and air bases.
9) Zero Carbon Bill which was in news recently  Russia has also assured India access to energy
was passed by which country? resources in the vast Arctic region.
a. Latvia  India has a similar arrangement with the
b. Bhutan US called the Logistics Exchange
Memorandum of Understanding
c. Denmark (LEMOA), signed between the two countries
d. New Zealand in August 2016.
2. c
10) Consider the following statements with  According to the India Justice Report
respect to Fall Armyworm (FAW) 2019 released recently by the Tata Trusts in
collaboration with Centre for Social
1. It is an insect that is native to Indian Justice, Common Cause,
subcontinent. and Commonwealth Human Rights
2. It affects Maize crop only. Initiative, among others, Maharashtra has
topped the list of 18 large-medium States in
Which of the statement(s) given above is/are the overall first-ever ranking of Indian States
correct? on justice delivery, followed by Kerala, Tamil
a. 1 only Nadu, Punjab and Haryana.
b. 2 only  In this category, Jharkhand, Bihar and Uttar
Pradesh are at the bottom, while among the
c. Both 1 and 2 list of seven smaller States, Goa leads the
d. Neither 1 nor 2 group.
 The report has been prepared based on
publicly available data of different government
Answers entities on the four pillars of justice delivery —
1. d police, judiciary, prisons and legal aid.
3. c
 India and Russia are expected to conclude a
mutual logistics agreement The Agreement  According to the report, titled ‘India
on Reciprocal Logistics Support Internet 2019,’ released by the Internet and
(ARLS). Mobile Association of India (IAMAI), Kerala’s
Internet penetration rate is the second highest
 ARLS is an arrangement that will allow access
in the country (54%), next only to Delhi NCR
to India and Russia, to each other’s military
with 69% penetration.
facilities for supplies and fuel, expanding the
logistics support and operational turnaround  The Internet penetration rate (defined
of the Indian military. as number of individuals aged above 12
per 100 population who accessed the
 This will be especially beneficial for the Indian
Internet in the last month; survey period
Navy, which has a large number of Russian
January-March 2019) was the lowest in Odisha
origin ships, which will get access to Russian
(25), Jharkhand (26) and Bihar (28).
ports for supplies and refueling.

www.shankariasacademy.com | www.iasparliament.com
30

4. a 6. b
 Acute Respiratory Infections (ARI) is a  Recently, the Finance Minister chairs 21st
serious ailment that prevents normal Meeting of the Financial Stability and
breathing function and kills an estimated 2.6 Development Council (FSDC).
million children annually every year
worldwide.  The Council reviewed the current global and
domestic macro-economic situation and
 Indians face the double burden of heavy air financial stability and vulnerabilities issues,
pollution in addition to the high rate of ARI including inter-alia, those concerning NBFCs
which hits children the hardest. and Credit Rating Agencies.
 National Health Profile – 2019 says that Acute  It is an apex level autonomous body to
Respiratory Infections (ARI) accounted for strengthen and institutionalize the mechanism
69.47% of morbidity (poor health) in the for maintaining financial stability.
communicable disease category leading to
27.21% mortality (Loss of Life).  It is not a statutory body and no separate
funds have been allocated for the council to
 ARI is highest in the Andhra Pradesh, Gujarat, undertake its activities.
Karnataka, Kerala, Tamil Nadu, Uttar Pradesh
and West Bengal.  Its members include -
1. The heads of financial sector regulators (RBI,
 Implications – Polluted air penetrate and
SEBI, PFRDA, and IRDA)
inflame the linings of bronchial tubes and
lungs. 2. Finance Secretary, Department of Economic
Affairs
 This leads to respiratory illness such as chronic
bronchitis, emphysema, heart disease, asthma, 3. Secretary, Department of Financial Services
wheezing, coughing and difficulty in breathing.
4. Chief Economic Adviser
 It poses a high risk to pregnant women and the 5. Chairman of the Insolvency and Bankruptcy
baby. Board
 The foetus receiving polluted air from mother  It focuses on financial literacy and financial
can increase health risk of unborn babies. inclusion.
 In pregnant women, it can cause a medical  The FSDC sub-committee is chaired by
condition called intrauterine inflammation. the Governor of RBI.
5.d 7. b
 The Documentation Identification  Recently, he Union Cabinet approved an MoU
Number (DIN) system of Central Board between India and Bangladesh on the
of Indirect Taxes (CBIC) has come into withdrawal of 1.82 cusecs of water from the
existence recently. Feni river by India for a drinking water supply
 Now, any CBIC communication will have to scheme for Sabroom town in Tripura.
have a Documentation Identification Number.  The Feni river, which forms part of the India-
 It is aimed at bringing transparency and Bangladesh border, originates in the South
accountability in the indirect tax Tripura district, passes through Sabroom
administration through widespread use of town on the Indian side, and meets the Bay of
information technology. Bengal after it flows into Bangladesh.

 The Government has already executed the  In Tripura, a 150-metre long, 4-lane bridge
DIN system in the direct tax across the Feni is being built between India
administration. and Bangladesh, where the river forms the
border between the two countries.

www.shankariasacademy.com | www.iasparliament.com
31

 Once ready, it would connect Tripura with  It has quickly spread across virtually all of
Chittagong port in Bangladesh, which is only Sub-Saharan Africa. In July 2018 it was
70 km away from the Indo-Bangla border. confirmed in India and Yemen.
 It would play an important role in the  In India it was reported for the first-time in
proposed economic corridor through India, Karnataka.
Bangladesh, China and Myanmar.
8. a
09-11-2019
 The Indian Navy – Indonesian
Navy Bilateral Maritime Exercise ‘Samudra 1) Consider the following statement with
Shakti’ Progresses in the Bay of Bengal. respect to ‘Special Protection Group’ (SPG)

 INS Kamorta, an Anti-Submarine Warfare 1. Cabinet secretariat of India is the


Corvette is jointly exercising with Indonesian governing body of SPG.
Warship KRI Usman Harun, a multi-role 2. Every member of the Group shall hold
Corvette. office during the pleasure of the
9. d President.
Which of the statement(s) given above is/are
 New Zealand passed historic climate
correct?
legislation to reduce its carbon emissions to
zero by 2050 and meet its commitments under a. 1 only
the Paris climate accords.
b. 2 only
 Zero Carbon Bill will preserve a new 2050
c. Both 1 and 2
greenhouse gas reduction target into law and
require that future governments have plans to d. Neither 1 nor 2
meet the target.
 Environment groups around the world are
2) Consider the following statements with
applauding the island country of nearly five
respect to Maternal mortality ratio (MMR)
million people.
1. It is the death of a woman while
10. d
pregnant or within 42 days of
 Recently, proper precaution and timely termination of pregnancy.
management by the state agriculture 2. SDG sets the global MMR target of less
department have succeeded in thwarting an than 70 per 100,000 live births by
attack by the Fall Armyworm (FAW) on 2030.
maize crop in Odisha.
3. Kerala, Maharashtra and Tamil Nadu
 The Fall Armyworm (FAW), or Spodoptera only have already met the SDG target.
frugiperda, is an insect that is native to
tropical and subtropical regions of the Which of the statements given above are
Americas. correct?

 In the absence of natural control or good a. 1 and 2 only


management, it can cause significant damage b. 2 and 3 only
to crops.
c. 1 and 3 only
 It prefers maize, but can feed on more
than 80 additional species of crops, d. 1, 2 and 3
including rice, sorghum, millet, sugarcane,
vegetable crops and cotton.
 FAW was first detected in Central and
Western Africa in early 2016.

www.shankariasacademy.com | www.iasparliament.com
32

3) Consider the following statements with Which of the statement(s) given above is/are
respect to Overseas Citizen of India (OCI) correct?
1. Section 7A of the OCI card rules states a. 1 only
that an applicant is not eligible for the
b. 2 only
OCI card if he, his parents or
grandparents have ever been a citizen of c. 3 only
Pakistan, Bangladesh or Sri Lanka.
d. None of the above
2. OCI does not confer political rights.
3. They are exempted from registering
with Foreigners Regional Registration 6) Consider the following statements with
Office (FRRO) no matter how long their respect to Ber Sahib Gurudwara
stay. 1. It is located in Sultanpur Lodhi in the
Which of the statement(s) given above is/are state of Punjab.
correct? 2. It is the place where Guru Nanak Dev
a. 1 and 2 only spent more than 14 years of his life and
attained enlightenment under a peepal
b. 2 and 3 only tree.
c. 1 and 3 only Which of the statement(s) given above is/are
correct?
d. 1, 2 and 3
a. 1 only
b. 2 only
4) Consider the following statements with
respect to Cloud seeding technology c. Both 1 and 2
1. It is a kind of a weather modification d. Neither 1 nor 2
technology to create new clouds.
2. It induce moisture in the clouds so as to
cause a rainfall. 7) Dhrupad sometimes seen in the news
recently is associated with which of the
Which of the statement(s) given above is/are following?
correct?
a. Regional Music
a. 1 only
b. Hindustani Music
b. 2 only
c. Carnatic Classical Music
c. Both 1 and 2
d. Musical Instrument
d. Neither 1 nor 2

8) With respect to Temporary Protected Status


5) Consider the following statements with (TPS) sometimes seen in the news recently,
respect to Ozone consider the following statements:
1. It is found only in the stratosphere. 1. It is a temporary status given to eligible
nationals of designated countries who
2. The Cartagena Protocol is the landmark
are present in the United Kingdom
multilateral environmental agreement
(U.K).
that regulates the production and
consumption of chemicals referred to as 2. It was initially meant to aid people from
ozone depleting substances (ODS). countries facing wars or natural
disasters.
3. Ozone depletion is not limited to the
area over the South Pole. Which of the statement(s) given above
is/are incorrect?

www.shankariasacademy.com | www.iasparliament.com
33

a. 1 only 1. c
b. 2 only  The Centre has decided to withdraw Special
Protection Group (SPG) security cover to the
c. Both 1 and 2
Gandhi family.
d. Neither 1 nor 2
 The Special Protection Group (SPG) was
raised in 1985 with the intention to provide
9) Consider the following statements with proximate security cover to the Prime
respect to Indian Ocean Rim Association Minister, former Prime Ministers and their
(IORA) immediate family members.

1. It is an inter-governmental organisation  The Special Protection Group Act, 1988 was


of 22 coastal countries bordering the constituted to provide regulation of an armed
Indian Ocean, as well as 9 dialogue force and so SPG is a statutory body.
partners.  The general superintendence, direction and
2. China and India is the largest control of the SPG is exercised by the Central
contributor to the IORA Special Fund. Government and Cabinet secretariat acts as
the governing body of SPG.
3. IORA Council of Ministers is the apex
body which consists of the foreign  No member of the Group shall be at liberty,
ministers of member states. 1. to resign his appointment during the terms of
Which of the statement(s) given above is/are his engagement; or
correct? 2. to withdraw himself from all or any of the
a. 3 only duties of his appointment,
b. 1 and 3 only 3. except with the previous permission in writing
of the prescribed authority.
c. 2 and 3 only
 Every member of the Group shall hold
d. 1, 2 and 3 office during the pleasure of the
President.
10) Consider the following statements with 2. d
respect to Steel Scrap Recycling Policy  Recently released ‘Special Bulletin on Maternal
1. It was issued by the Ministry of Steel. Mortality in India 2015-2017’ of the Sample
Registration System states that India registers
2. It aims to promote circular economy in 26.9% decline in Maternal Mortality Rate since
the steel sector. 2013.
3. The target of the policy is to minimise
 Maternal Mortality Ratio (MMR) is defined as
the dependency on imports.
the number of maternal deaths per 100,000
Which of the statement(s) given above is/are live births due to causes related to pregnancy
correct? or within 42 days of termination of pregnancy,
regardless of the site or duration of pregnancy.
a. 3 only
 SDG 3- Health target states that by 2030,
b. 1 and 2 only
reduce the global maternal mortality ratio
c. 2 and 3 only to less than 70 per 100 000 live births.
d. 1, 2 and 3  Assam recorded the highest MMR 229
maternal deaths.
 Uttar Pradesh (216), Madhya Pradesh (188),
Answers Rajasthan (186), Odisha (168), Bihar (165) and
Chhattisgarh (141) follows the list.

www.shankariasacademy.com | www.iasparliament.com
34

 Kerala has the lowest MMR, at 42. It is 5. c


followed by Maharashtra (55), Tamil
 It has been reported that the annual ozone
Nadu (63). They have already met the SDG
hole over the Antarctic has been found to be at
target of 70 per 100,000 MMR,
its smallest since the 1980s.
 Andhra Pradesh (74), Jharkhand (76) and
 Ozone (chemically, a molecule of three oxygen
Telangana (76) are within striking distance.
atoms) is found mainly in the upper
3. b atmosphere, an area called stratosphere,
between 10 and 50 km from the earth’s
 Recently government has revoked author surface.
Aatish Taseer’s Overseas Citizen of India card.
 About 90% of total ozone in the atmosphere
 According to Section 7A of the OCI card is in the stratosphere, and 10% is in
rules, an applicant is not eligible for the OCI the troposphere.
card if he, his parents or grandparents have
ever been a citizen of Pakistan or  The Montreal Protocol on Substances that
Bangladesh. Deplete the Ozone Layer is the landmark
multilateral environmental agreement that
 OCI is not to be misconstrued as 'dual regulates the production and consumption of
citizenship'. OCI does not confer political nearly 100 man-made chemicals referred to
rights. as ozone depleting substances (ODS).
 OCI cardholders can enter India multiple  The ozone depletion is not limited to the
times, get a multipurpose lifelong visa to area over the South Pole (Antarctica).
visit India.
 Research has shown that ozone depletion
 They are exempt from registering with occurs over the latitudes that include North
Foreigners Regional Registration Office America, Europe, Asia, and much of Africa,
(FRRO) no matter how long their stay. Australia, and South America.
 If an individual is registered as an OCI for a 6. a
period of five years, he/she are eligible to apply
for Indian citizenship.  PM Modi recently visited Ber Sahib
Gurudwara where he visited the Ber
4. b tree under which Shri Guru Nanak Dev Ji is
 Recently, Haryana Deputy CM has written to said to have meditated for over 14 years.
PM requesting him to undertake cloud seeding 7. b
plan to combat the air pollution engulfing
Delhi and NCR.  Dhrupad is a musical form in Hindustani
Music.
 Cloud seeding is a kind of a weather
modification technology to create artificial  It emphasis more on strict adherence to raga
‘rainfall’. and systematized arrangement of musical
parts.
 It works only when there is enough pre-
existing clouds in the atmosphere.  It usually covers themes such as religion,
philosophy, devotion and praise of deities,
 It is an artificial way to induce moisture patrons and celebration of seasons.
in the clouds so as to cause a
rainfall. It does not create new clouds. 8. a

 In this process, either silver iodide or dry  Temporary Protected Status (TPS) was
ice is dumped onto the clouds by using an initially meant to aid people from countries
aircraft or an artillery gun which leads to a rain facing wars or natural disasters.
shower.
 Citizens of Honduras were originally granted
TPS in 1999 in the wake of overwhelming
damage from Hurricane Mitch, while migrants

www.shankariasacademy.com | www.iasparliament.com
35

from El Salvador and Haiti gained protection  The policy said that Ministry of Road
in 2001 and 2010 respectively following Transport and Highways (MoRTH) and the
earthquakes. Department of Heavy Industries are working
towards ‘Extended Producer Responsibility’ by
 About 400,000 citizens of El Salvador,
requiring the vehicle manufacturers to
Honduras, and Haiti have been able to stay in
incentivise scrapping of unfit vehicles in
the US through Temporary Protected Status
exchange for price discounts for purchase of
(TPS), a protection allowing them to legally
new vehicles.
live and work in the United States
(US) typically offered to citizens of countries  The Policy has aimed setting up an
suffering from natural disasters or armed Environmentally Sound Management system
conflict. for ferrous scrap which can encourage
processing & recycling of ferrous scraps
9. b
through organized and scientific metal
 IORA is the apex regional organisation of 22 scrapping centers across India to minimize
Member States and nine (9) Dialogue Partners, dependency on import of scrap and make
stretching from South Africa in the west, India self-sufficient in scrap availability.
running up the eastern coast of Africa, along
 The target is to make India a producer
the Gulf to South and Southeast Asia, ending
of high quality ferrous scrap for quality
with Australia in the east.
steel production thus minimising the
 IORA stands committed to work for a peaceful dependency on imports.
and prosperous Indian Ocean rim.
 The current 22 Member States are: Australia,
Bangladesh, Comoros, India, Indonesia, Iran,
11-11-2019
Kenya, Madagascar, Malaysia, Maldives, 1) Consider the following statements with
Mauritius, Mozambique, Oman, Seychelles, respect to MILAN Exercise
Singapore, Somalia, South Africa, Sri Lanka,
Tanzania, Thailand, United Arab Emirates and 1. It is a series of multilateral naval
Yemen. exercise commenced in 1995.

 China is only a dialogue partner and not a 2. It will be conducted every year at
member state. Andaman and Nicobar Islands.
3. For the first time, MILAN 2020 will be
 India is the largest contributor to the IORA
held on the mainland at
Special Fund.
Visakhapatnam.
 IORA’s apex body is the Council of Foreign Which of the statements given above are
Ministers (COM) that meets annually. correct?
 19th IORA Council of Ministers meeting has a. 1 and 2 only
recently begun.
b. 1 and 3 only
 The theme is "Promoting a Shared Destiny
and Path to Prosperity in the Indian Ocean. c. 1, 2 and 3
10. d d. None

 Steel Scrap Recycling Policy was issued


recently by the Ministry of Steel. 2) With respect to English Channel, consider
 The policy also aims to promote circular the following statements:
economy in the steel sector. 1. It is one of the busiest shipping lanes in
the world and is a part of the Atlantic
 Promotion of a formal and scientific collection,
Ocean.
dismantling and processing activities for end
of life products that are sources of recyclable
scraps also feature in the policy.

www.shankariasacademy.com | www.iasparliament.com
36

2. It separates the island of Britain from d. 1, 2 and 3


the northern France and joins the North
Sea to the Atlantic Ocean.
5) Consider the following statements with
Which of the statement(s) given above is/are
respect to
correct?
Places in News – Country
a. 1 only
1. Ghumar – Israel
b. 2 only
2. Al-Baqoura – Jordan
c. Both 1 and 2
Which of the pair(s) given above is/are
d. Neither 1 nor 2
correctly matched?
a. 1 only
3) Consider the following statements
b. 2 only
1. Heliopause is formed by the Sun's
c. Both 1 and 2
magnetic field and solar winds that can
reach speeds of three million kilometres d. Neither 1 nor 2
per hour.
2. Heliosphere is the relatively thin
contact boundary where solar wind of 6) Kalapani a place which was in news recently
charged particles and interstellar wind is located in
collide. a. Andaman and Nicobar Islands
Which of the statement(s) given above b. Nepal
is/are incorrect?
c. Uttarakhand
a. 1 only
d. Tibet
b. 2 only
c. Both 1 and 2
7) Consider the following statements with
d. Neither 1 nor 2 respect to Taku glacier
1. It is one of the thickest known alpine
4) Consider the following statements with glaciers in the world.
respect to Orangutans 2. It is located in Oceania.
1. Like humans, orangutans have Which of the statement(s) given above is/are
opposable thumbs. correct?
2. Earlier, orangutans were found a. 1 only
throughout eastern Asia including Java,
China and Laos but now restricted only b. 2 only
to islands of Sumatra and Bornea. c. Both 1 and 2
3. Both the Sumatran and Bornean d. Neither 1 nor 2
orangutans are listed as critically
endangered under the IUCN Red List.
Which of the statement(s) given above is/are 8) Consider the following statements with
correct? respect to Anemia

a. 2 only 1. Iron deficiency is thought to be the most


common cause of anaemia globally.
b. 1 and 2 only
c. 1 and 3 only

www.shankariasacademy.com | www.iasparliament.com
37

2. Anemia Mukt Bharat has the target of Answers


reduction in anemia prevalence by one-
third of NFHS 4 levels by 202 1. b

Which of the statement(s) given above is/are  MILAN series of biennial a multilateral naval
correct? exercise which commenced in 1995, was
conducted at Andaman and Nicobar
a. 1 only Command (ANC) until last year, is being
b. 2 only conducted for the first time on
the mainland at Eastern Naval
c. Both 1 and 2 Command (ENC) Visakhapatnam with
d. Neither 1 nor 2 increased scope and complexity of the
Exercise.
 MILAN 2020 will be held in March 2020.
9) Consider the following statements with
respect to Black Hole  MILAN 2020 is aimed to enhance professional
interaction between friendly foreign navies and
1. It is a surface in space where gravity learn from each other’s strengths and best
pulls so much that even light can not get practices in the maritime domain.
out.
 The event would also provide an excellent
2. It can be seen directly with special Space opportunity for Operational Commanders of
telescopes. friendly foreign navies to interact with each
Which of the statement(s) given above is/are other in areas of mutual interest.
correct? 2. c
a. 1 only
b. 2 only
c. Both 1 and 2
d. Neither 1 nor 2

10) Consider the following statements with


respect to Bioremediation
1. It is a process that uses only Bacteria to
remove contaminants, pollutants, and
toxins from soil and water.
2. It is traditionally used to clean oil spills.
Which of the statement(s) given above is/are
correct?
a. 1 only
b. 2 only
c. Both 1 and 2
3. c
d. Neither 1 nor 2
 Heliosphere is formed by the Sun's magnetic
field and solar winds that can reach speeds of
three million kilometres per hour.

www.shankariasacademy.com | www.iasparliament.com
38

 Heliopause is the relatively thin contact and announces Ghumar and Al-
boundary where solar wind of charged Baqoura’s return to ‘full sovereignity’.
particles and interstellar wind collide.
6. c
4. d
 The new political map of India, recently
 Orangutans are the largest arboreal mammals released by the government to account for the
in the world. They are apes, who are closely bifurcation of Jammu and Kashmir, has
related to humans, having 97% of DNA in triggered fresh protests over an old issue in
common. Kathmandu.
 Like humans, orangutans have opposable  Mapped within Uttarakhand is a 372-sq
thumbs. km area called Kalapani, bordering far-west
Nepal and Tibet.
 They prefer to live in swamp forests.
 While the Nepal government and political
 Unlike other apes and monkeys, orangutans parties have protested, India has said the new
prefer to lead a solitary life. map does not revise the existing boundary with
 Thousands of years ago, orangutans could be Nepal.
found throughout eastern Asia including Java,
China and Laos.
 But today, their population is restricted
islands of Sumatra and Bornea.
 The Sumatran orangutans have a narrow face
and longer beard than their Bornean
counterparts.
 Both the species were endangered and feature
on the IUCN (International Union for
Conservation of Nature) Red List under
the critically endangered category.
 They are threatened by the loss of habitat,
deforestation, climate change and wildfires.
5. b
 Jordan has pulled out of a 25-year-old
landmark agreement that allowed Israeli
farmers to cultivate in two pieces of
agricultural land that was leased by Israel.
 Under the deal, part of the 1994 Jordan-Israel
peace treaty, two territories straddling the
border were recognised as under Jordanian  Nepal’s western boundary with India was
sovereignty but with special provisions marked out in the Treaty of Sugauli between
allowing Israeli farmers to work the land and the East India Company and Nepal in 1816.
visitors to tour the Isle of Peace park in the  Nepali authorities claim that people living in
area. the low-density area were included in the
 But in 2018, Jordan said it did not want to Census of Nepal until 58 years ago.
continue the arrangement, in what was widely 7. a
seen as a sign of increasingly strained
diplomatic relations.  A new analysis found that one of the world’s
thickest glaciers, Taku Glacier has began
 King Abdullah II has formally declared retreating.
recently the end of the 25-year special regime

www.shankariasacademy.com | www.iasparliament.com
39

 The visible retreat is shown on new satellite needs, which vary by age, sex, altitude,
photos released by NASA taken in 2014 and smoking, and pregnancy status.
2019.
 Iron deficiency is thought to be the most
 It is located near Juneau, in common cause of anaemia globally,
the U.S. state of Alaska. although other conditions, such as folate,
vitamin B12 and vitamin A deficiencies,
 The massive glacier, which measures 4,860 chronic inflammation, parasitic infections, and
feet from surface to floor, was gaining mass for inherited disorders can all cause anaemia.
almost 50 years, but now appears to be
shrinking.  The Anemia Mukt Bharat focusses on six
target beneficiary groups, through six
8. c interventions and six institutional mechanisms
 A recent study published in The Lancet Global to achieve the envisaged target under the
Health, which looked at anaemia among men POSHAN Abhiyan.
in India, found that nearly a quarter of them  It focuses on the target of reduction in
(23.2% in a sample of 1 lakh men) in the age anemia prevalence by one-third of
group 15-54 had some form of anaemia. NFHS 4 levels by 2022.
9. d
 Recently, in a first researchers have created
simulations explaining how the largest of black
hole mergers may have happened, showing one
may have devoured another "in a 'Pac-Man-
like' behaviour.
 A black hole is a place/ region of space
where gravity pulls so much that even light can
not get out.
 It does not have a surface, like a planet or
star.
 Because no light can get out, people can't see
black holes. They are invisible and cannot
be seen directly.
 Although black holes cannot be seen, we
know they exist from the way they affect
nearby dust, stars and galaxies.
 Many of them are surrounded by discs of
material.
 As the discs swirl around them like a
whirlpool, they become extremely hot and give
off X-rays.
 Space telescopes with special tools can help
find black holes.
 The special tools can see how stars that are
very close to black holes act differently than
other stars.

 Anaemia is a condition in which the number of


red blood cells or their oxygen-carrying
capacity is insufficient to meet physiologic

www.shankariasacademy.com | www.iasparliament.com
40

10. b 2. It would be funded and managed by the


Defence Innovation Organisation (DIO)
 The United Nations has rolled out a project to works under the aegis of Ministry of
use bioremediation process with the help of Defence.
naturally occuring soil bacteria to
decontaminate land poisoned by oil spills in Which of the statement(s) given above is/are
war-torn Iraq. correct?
 The UNEP has partnered the Iraqi a. 1 only
government for the project. b. 2 only
 Bioremediation is defined as a process c. Both 1 and 2
that uses microbes, bacteria, and other
living organisms or their enzymes to treat d. Neither 1 nor 2
polluted sites for regaining their original
condition.
3) Consider the following statements with
 It is traditionally used to clean oil spills respect to International Treaty of Plant
or contaminated groundwater. Genetic Resources for Food and Agriculture
 It may be done "in situ," at the site of the (ITPGRFA)
contamination, or "ex situ," away from the 1. It is also known as Seed Treaty.
site.
2. The objectives of this treaty include fair
12-11-2019 and equitable sharing of the benefits
arising out of world’s plant genetic
1) With respect to Swachh – Nirmal Tat
resources.
Abhiyaan Campaign sometimes seen in the
news recently, consider the following 3. It also recognizes farmers' rights,
statements: subject to national laws.
1. It aims to maintain the rural amenities Which of the statement(s) given above is/are
including libraries clean and easily correct?
accessible.
a. 2 only
2. It was launched by the Ministry of Rural
b. 1 and 3 only
Development.
c. 1, 2 and 3
Which of the statement(s) given above is/are
correct? d. None
a. 1 only
b. 2 only 4) Consider the following statements with
respect to BRICS summit
c. Both 1 and 2
1. It will be held annually.
d. Neither 1 nor 2
2. 11th edition of the BRICS summit will be
held in Brasilia, Brazil.
2) With respect to Innovations for Defence
3. The theme for the 11th summit is
Excellence (iDEX) initiative sometimes seen in
Collaboration for inclusive growth and
the news recently, consider the following
shared prosperity in the 4th Industrial
statements:
Revolution.
1. It primarily aims to foster innovation
Which of the statement(s) given above is/are
and technology development in Defence,
correct?
Aerospace and Irrigation technologies
by engaging Industries including a. 3 only
MSMEs, start-ups, individual
innovators. b. 1 and 2 only

www.shankariasacademy.com | www.iasparliament.com
41

c. 1 and 3 only b. 1 and 2 only


d. 1, 2 and 3 c. 1 and 3 only
d. 1, 2 and 3
5) World’s first Compressed Natural Gas
(CNG) port terminal to be built in which of the
9) Consider the following statements with
following states?
respect to Cheraman Juma Masjid
a. Gujarat
1. It was built during the reign of Delhi
b. Telangana Sultanate.
c. Tamil Nadu 2. It is the first structure in India to have
arches and domes.
d. Maharashtra
Which of the statement(s) given above is/are
correct?
6) JARVIS sometimes seen in the news recently
a. 1 only
is associated with?
b. 2 only
a. Monolith like robots used in space activities
c. Both 1 and 2
b. Sub Neptune exo-planet discovered by NASA
d. Neither 1 nor 2
c. AI enabled video analytics platform used in
Prisons
d. Cyber safety manual for girls released by the 10) Which of the following is true
Ministry of Women and Child Development regarding India’s Electricity Demand?
1. India’s electricity demand fell in the
recent times.
7) Mihir Shah Committee sometimes seen in
the news recently is associated with which of 2. Industrial states like Maharashtra and
the following? Tamil Nadu witness steady growth in
demand.
a. National Water Policy
Which of the statement(s) given above is/are
b. New National Energy Policy
correct?
c. Steel Scrap Recycling Policy
a. 1 only
d. New Exploration Licensing Policy
b. 2 only
c. Both 1 and 2
8) Consider the following statements with
d. Neither 1 nor 2
respect to Suranga Bawadi
1. It is an integral part of the ancient Karez
system of supplying water through Answers
subterranean tunnels.
1. d
2. It was built in the 16th century by Ali
Adil Shah – I of Bijapur Sultanate.  Ministry of Environment, Forest and Climate
Change (MoEF&CC) are undertaking a mass
3. It was recently included in the cleanliness-cum-awareness drive in 50
UNESCO’s World Heritage List. identified beaches under the “Swachh –
Which of the statement(s) given above is/are Nirmal Tat Abhiyaan”.
correct?  The identified beaches are in 10 coastal
a. 1 only States/Union Territories (UTs) namely
Gujarat, Daman & Diu, Maharashtra, Goa,

www.shankariasacademy.com | www.iasparliament.com
42

Karnataka, Kerala, Tamil Nadu, Puducherry,  It also recognizes farmers' rights, subject to
Andhra Pradesh, and Odisha. national laws.
 The beaches have been identified after the 4. b
consultation with the States/UTs.
 The theme of the 11th BRICS summit is
 Environment Education Division of the "Economic Growth for an Innovative
Ministry and Society of Integrated Coastal Future.”
Management (SICOM) under the aegis of
5. a
Environment Ministry will be responsible for
the overall coordination for the drive in 50  Gujarat Government has recently given its
beaches. nod to the world’s first CNG port
2. d terminal at Bhavnagar.

 Innovations for Defence Excellence  It will be developed jointly by U.K.-


(iDEX) launched by the Government in April headquartered Foresight Group and Mumbai-
2018, primarily aims at creation of an based Padmanabh Mafatlal Group.
ecosystem to foster innovation and technology 6. c
development in Defence and Aerospace by
engaging Industries including MSMEs, start-  Uttar Pradesh is using an Artificial
ups, individual innovators, R&D institutes & Intelligence (AI)-powered video analytics
academia, and provide them grants/funding solution, JARVIS, to keep an eye on prison
and other support to carry out R&D which has activities, including unauthorised access and
good potential for future adoption for Indian violence.
defence and aerospace needs.  The AI-powered video analytics tool is now live
 iDEX will be funded and managed by a with 700 cameras from 70 prisons for multi-
‘Defence Innovation Organization purpose analytics, including frisking,
(DIO)’ which has been formed as a ‘not for unauthorised access, crowd analysis, violence,
profit’ company as per Section 8 of the camera working and intrusion detection
Companies Act 2013 for this purpose, by the amongst others.
two founder members i.e. Defence Public 7. a
Sector Undertakings (DPSUs) - HAL & BEL.
 The Union Water Resources Ministry has
 iDEX will function as the executive arm of finalised a committee to draft a new National
DIO, carrying out all the required activities Water Policy (NWP).
while DIO will provide high level policy
guidance to iDEX.  It will be chaired by Mihir Shah, who is a
former Planning Commission member and a
3. c water expert.
 International Treaty of Plant Genetic  The committee has 10 principal members.
Resources for Food and Agriculture
(ITPGRFA) is also known as Seed Treaty.  The NWP currently in force was drafted in
2012 and is the third such policy since 1987.
 It is a comprehensive international agreement
for ensuring food security.  The committee is expected to produce a report
within six months.
 The objectives of this Treaty are the
conservation and sustainable use of plant 8. b
genetic resources for food and agriculture and
 Suranga Bawadi, an integral part of the
the fair and equitable sharing of the
ancient Karez system of supplying water
benefits arising out of their use, in
through subterranean tunnels built during Adil
harmony with the Convention on Biological
Shahi era in Vijayapura, is now set to get
Diversity, for sustainable agriculture and food
funding for restoration.
security.

www.shankariasacademy.com | www.iasparliament.com
43

 A New York-based non-governmental b. 2 only


organisation has included it in the World
c. Both 1 and 2
Monument Watch list for 2020 along
with 24 other monuments from across the d. Neither 1 nor 2
world.
 The monument has been selected under the 2) Consider the following statements
‘Ancient Water System of the Deccan Plateau’
by World Monuments Fund [the NGO], 1. It is India’s largest inland saltwater
which monitors restoration of ancient lake.
monuments across the globe.
2. It is recognized as wetland of
 Though the Karez system was built in the 16th international importance.
century by Ali Adil Shah–I, his successor, 3. It is a key wintering area for tens of
Ibrahim Adil Shah–II, brought in several thousands birds.
changes by adding more structures to
strengthen it. Which of the statement(s) given above is/are
correct?
 According to historians, the Adil Shahis built
the magnificent underground system to supply a. Dal Lake
water to the city, which had a population of b. Tso Moriri
nearly 12 lakh then.
c. Wular Lake
9. d
d. Sambhar Lake
 The Cheraman Juma Masjid at Kodungalloor,
Kerala is the India’s first-ever mosque built
in 629 A.D. 3) “Arrokoth” sometimes seen in the news
 Delhi sultanate starts from 1206 A.D. recently is a/an?

10. a a. Most distant object ever explored

 India’s electricity demand fell 13.2% in b. Cheap electricity scheme launched by Madhya
October from the same month last year, Pradesh
registering the steepest fall in 154 months. c. AI mechanism to detect lightning
 Except Himachal and J&K the other states d. None of the above
registered a fall in electricity demand in
October 2019 from the same month a year
before. 4) How is National Emergency different from
the President’s rule?
1. During National Emergency, the state
13-11-2019 executive is dismissed and the state
1) With respect to Exercise TIGER TRIUMPH, legislature is either suspended or
consider the following statements: dissolved whereas during the
President’s rule, the state executive and
1. It is a maiden naval exercise held legislature continue to function.
between India and Bangladesh.
2. National Emergency affects
2. Its main aim is to focus on maritime fundamental rights of the citizens
terrorism and drug trafficking through whereas President’s rule has no effect
seas. on Fundamental Rights of the citizens.
Which of the statement(s) given above is/are 3. While there is a maximum period
correct? prescribed for National Emergency,
a. 1 only there is no maximum period prescribed
for President’s rule.

www.shankariasacademy.com | www.iasparliament.com
44

Select the correct answer using the codes given d. Neither 1 nor 2
below:
a. 2 only
8) Consider the following statements with
b. 1 and 2 only respect to Indra-2019
c. 2 and 3 only 1. It is a tri-service exercise.
d. 1, 2 and 3 2. It is between India and Russia.
Which of the statement(s) given above is/are
correct?
5) Consider the following statements
a. 1 only
1. It is also called as Vietnamese mouse-
deer. b. 2 only
2. The rabbit sized animal looks partially c. Both 1 and 2
as mouse and partially as deer.
d. Neither 1 nor 2
3. It was recently rediscovered in Vietnam
after 30 years.
9) Consider the following statements with
Identify the species that correctly matches with
respect to “X-57 Maxwell”
the above description:
1. It is the first all-electric experimental
a. Tiliger
aircraft
b. Elephant shrew
2. It is a joint project of NASA and ISRO
c. Sunda Pangolin
Which of the statement(s) given above is/are
d. Silver-backed Chevrotain correct?
a. 1 only
6) K-Fon project which was in news is launched b. 2 only
by?
c. Both 1 and 2
a. Kerala
d. Neither 1 nor 2
b. Andhra Pradesh
c. Gujarat
10) Consider the following statements with
d. Maharashtra respect to National Sustainable Spice Program
(NSSP)
1. It targets to convert a quarter of all the
7) Consider the following statements with
spices grown in the country sustainable
respect to Pneumonia
by 2025.
1. It can be caused by viruses, bacteria, or
2. It is an initiative by Ministry of
fungi.
agriculture and farmers welfare.
2. All such types of Pneumonia can be
Which of the statement(s) given above is/are
treated with antibiotics.
correct?
Which of the statement(s) given above is/are
a. 1 only
correct?
b. 2 only
a. 1 only
c. Both 1 and 2
b. 2 only
d. Neither 1 nor 2
c. Both 1 and 2

www.shankariasacademy.com | www.iasparliament.com
45

Pluto, was surveyed by the NASA spaceship


New Horizons in January, with images
Answers showing it consisted of two spheres stuck
1. d together in the shape of a snowman.

 The first tri-Service exercise between India  Its technical designation is 2014 MU69.
and the US, named 'Tiger Triumph' which 4. a
will largely focus on amphibious HADR
(humanitarian relief and disaster National Emergency (Article 352)
relief) operations, will kick off along the  It can be proclaimed only when the security of
eastern coast in the Bay of Bengal. India or a part of it is threatened by war,
 It was announced by US President Donald external aggression or armed rebellion.
Trump during the “Howdy Modi Event” at  During its operation, the state executive
Houston. and legislature continue to function
2. d and exercise the powers assigned to
them under the Constitution. Its effect is that
 Thousands of migratory birds of about ten the Centre gets concurrent powers of
species were found dead around Sambhar administration and legislation in the state.
Lake near Jaipur.
 There is no maximum period
 Sambhar Lake is India’s largest inland salt prescribed for its operation. It can be
lake. continued indefinitely with the approval of
 It is a recognised wetland of international Parliament for every six months.
importance, and hosts thousands of migratory President’s Rule (Article 356)
birds during winters.
 It can be proclaimed when the government of a
 The dead birds included plovers, common state cannot be carried on in accordance with
coots, black winged stilts, northern shovelers, the provisions of the Constitution due to
ruddy shelducks and pied avocets. reasons which may not have any connection
3. a with war, external aggression or armed
rebellion.
 During its operation, the state executive is
dismissed and the state legislature is
either suspended or dissolved. The
president administers the state through the
governor and the Parliament makes laws for
the state. In brief, the executive and legislative
powers of the state are assumed by the Centre.
 There is a maximum period
prescribed for its operation, that is, three
years. Thereafter, it must come to an end and
the normal constitutional machinery must be
restored in the state.
 President's rule has been recently imposed in
Maharashtra.
 Ultima Thule, the farthest cosmic body ever
visited by a spacecraft, has been  The state legislative assembly has been kept
under suspended animation.
renamed Arrokoth, or 'sky' in the Native
American Powhatan language. 5. d
 The icy rock, which orbits in the dark and  A small deer-like creature that conservationists
frigid Kuiper Belt about a billion miles beyond feared was extinct has been photographed

www.shankariasacademy.com | www.iasparliament.com
46

recently, in Vietnam, for the first time in  According to the WHO’s factsheet on
almost three decades. pneumonia, infectious agents may include
bacteria, viruses and fungi.
 Scientists had last spotted the Silver-Backed
Chevrotain, also called the Vietnamese  Pneumonia caused by bacteria can be
mouse-deer, in 1990 in central Vietnam. treated with antibiotics, but only one third
of children with pneumonia receive the
 The rabbit-size chevrotain lives in a tropical
antibiotics they need.
forest-covered part of the Asian nation, is often
targeted by poachers. 8. c
 Mouse-deer aren’t actually deer (or mice, as it  It is the India-Russia Military Exercise.
happens) but ungulates, a group that
includes deer along with other herbivorous,  This year’s iteration of the bilateral exercise
hoofed animals. will include air, ground, and naval components
making it the tri-service exercise.
9. a
 It is a small, experimental airplane powered by
electricity.
 All-electric technology will make flying
cleaner, quieter, and more sustainable.
 It is a project of NASA.
 Located in NASA’s Armstrong Flight Research
Center in Edwards, California, the
experimental aircraft is in the first of three
configurations, called Modification II or Mod
II.
6. a
 This version replaces a traditional combustion
 Recently Kerala government launched K-Fon
engine with electric cruise motors that are
(Kerala-Fiber Optic Network) project.
potentially quieter and more efficient, as well
 It will provide free high speed internet as being better for the environment.
connection to over 20 Lakh Below Poverty
10. a
Line (BPL) families.
 The National Sustainable Spice Programme
 The program is aimed at giving a push to the
(NSSP) is an join initiative of,
digitization of government services.
1. The World Spice Organisation (an arm of the
 Kerala was the first in India to announce right All-India Spices Exporters Forum),
to access the Internet as a basic human
right. 2. Spices Board of India,
7. a 3. IDH (the Sustainable Trade Initiative) and
 Since 2009, November 12 has been observed 4. GIZ Global Project of Germany
as ‘World Pneumonia Day’ to raise awareness
 The NSSP targets to convert a quarter of all the
about the infection. spices grown in the country sustainable by
 It is a form of acute respiratory infection that 2025.
affects the lungs.
 The multi-stakeholder programme will help
 WHO identifies pneumonia is the single farmers reduce the use of pesticides and
largest cause of death in children worldwide. contaminants.

 Every year, it kills an estimated 1.4 million  It will address issues such as high dependency
children under the age of five years. on chemical fertilisers and exploitation of

www.shankariasacademy.com | www.iasparliament.com
47

natural resources, which contribute to climate 3) Consider the following statements with
change and outbreak of new diseases respect to Negative interest rates
1. These are rates that are charged on the
principal and on previously earned
14-11-2019 interest retrospectively.
1) Consider the following statements with 2. It leads to a condition where a saver is
respect to Asia-Pacific Rural and Agricultural paying for parting with cash and a
Credit Association (APRACA) borrower is being perversely
1. It is a regional association that incentivised to borrow more.
represents 81 member institutions from Which of the statement(s) given above is/are
21 countries including India. correct?
2. It is the world’s premier research a. 1 only
organization dedicated to reducing
poverty and hunger through rice b. 2 only
science. c. Both 1 and 2
Which of the statement(s) given above is/are d. Neither 1 nor 2
correct?
a. 1 only
4) Consider the following statements
b. 2 only
1. A person disqualified under the Tenth
c. Both 1 and 2 Schedule of the Indian Constitution is
d. Neither 1 nor 2 eligible to contest in re-election to that
constituency.
2. As per the existing constitutional
2) Consider the following statements with mandate, the Speaker is not empowered
respect to Right to Information (RTI) Act, to disqualify any member for the entire
2005 term of the House.
1. RTI is applicable to the whole of India Which of the statement(s) given above is/are
including Jammu and Kashmir and correct?
Ladakh.
a. 1 only
2. While the office of the Chief Justice of
India (CJI) is under the RTI’s ambit, the b. 2 only
Central Bureau of Investigation (CBI) is c. Both 1 and 2
exempt.
d. Neither 1 nor 2
Which of the statement(s) given above is/are
correct?
a. 1 only 5) Consider the following statements
b. 2 only 1. Houses under the ‘Banglar Abas’
scheme was allocated by clearing acres
c. Both 1 and 2 of mangrove forest in this Island.
d. Neither 1 nor 2 2. The very severe cyclone Bulbul made
landfall recently at this Island.
Identify the Island that correctly matches with
the above description:
a. Havelock Island
b. Diu Island

www.shankariasacademy.com | www.iasparliament.com
48

c. Sagar Island c. Both 1 and 2


d. Majuli Island d. Neither 1 nor 2

6) Consider the following statements with 9) Consider the following statements regarding
respect to Space the composition of the council of ministers:
1. Gravity is present everywhere, even in 1. Deputy Ministers are not members of
space. the Cabinet.
2. There is no air in space and it is almost a 2. Parliament secretaries often hold the
perfect vacuum. rank of the Minister of State.
Which of the statement(s) given above is/are Which of the statements given above is/are
correct? correct?
a. 1 only a. 1 only
b. 2 only b. 2 only
c. Both 1 and 2 c. Both 1 and 2
d. Neither 1 nor 2 d. Neither 1 nor 2

7) Consider the following statements with 10) Which of the following are Temperate
respect to Emperor penguin Grasslands?
1. These are endemic to Antarctica. 1. Steppes
2. These are the tallest and heaviest of all 2. Pampas
living penguin species.
3. Savannas
Which of the statement(s) given above is/are
4. Downs
correct?
Select the correct answer using the codes given
a. 1 only
below:
b. 2 only
a. 1 and 2 only
c. Both 1 and 2
b. 2, 3 and 4 only
d. Neither 1 nor 2
c. 1, 2 and 4 only
d. 1, 2, 3 and 4
8) Consider the following statements with
respect to World Congress on Rural and
Agricultural Finance Answers
1. The 6th edition of the meeting was held 1. a
recently in Nairobi, Kenya.
 Asia-Pacific Rural and Agricultural Credit
2. It is being jointly hosted by Asia Pacific Association (APRACA), representing 81
Economic Cooperation and National member institutions from 21 countries, is a
Bank for Agriculture and Rural regional association that promotes cooperation
Development. and facilitates mutual exchange of information
Which of the statement(s) given above is/are and expertise in the field of rural finance.
correct?  Over the years it has developed a repository of
a. 1 only knowledge in managing and serving rural
financial institutions, financial inclusion and
b. 2 only

www.shankariasacademy.com | www.iasparliament.com
49

to help avert financial risks and improve defection law) but held that their ouster
efficiency. was no bar to contesting re-polls.
 6th World Congress on Rural and Agriculture  Neither under the Indian Constitution nor
Finance was jointly organised by National under the statutory scheme it is contemplated
Bank for Agriculture and Rural Development that disqualification under the Tenth Schedule
(NABARD) and Asia - Pacific Rural would operate as a bar for contesting re-
Agricultural and Credit Association elections.
(APRACA) recently.
 In the light of the existing constitutional
 The International Rice Research mandate, the Speaker is not empowered to
Institute (IRRI) is the world’s premier disqualify any member till the end of the term.
research organization dedicated to reducing
5. c
poverty and hunger through rice science;
improving the health and welfare of rice 6. c
farmers and consumers; and protecting the
rice-growing environment for future  When you drop a pen on Earth, it falls to the
generations. ground. This is because of the gravitational
pull of the planet.
2. c
 This gravity is everywhere, even in space.
 Supreme Court has recently ruled that
the office of the Chief Justice of India  It is gravity that holds moon in orbit around
(CJI) is a public authority under Earth and Earth around the sun.
the Right to Information (RTI) Act.  It also keeps the sun in the Milky way.
 While the office of the CJI is now under the  Gravity causes every object to pull every other
RTI’s ambit, the CBI is exempt. object towards it. However, the effect of gravity
 As Jammu and Kashmir Reorganisation Act decreases as distance increases.
became operational on October 31, 2019  At extreme distances, the gravitational pull of
the central RTI Act became applicable in an object exerted on another could be almost
J&K and Ladakh. zero, but never totally absent. This is called
3. b microgravity.

 A negative interest means the instead of the Then, why do objects float in orbit?
bank paying you money to keep in the savings  Because they are in free fall. There is no air in
account, you pay the bank to do so. space. It is almost a perfect vacuum.
 It also means that anyone can borrow money  In a vacuum, gravity causes all objects to fall at
from the bank and pay back less than what he the same rate, irrespective of their mass.
borrowed.
 Earth's gravity is still pulling down on
 Negative interest rates are expected to make astronauts or spacecraft or space station in
consumers save less and spend more; they are orbit.
also expected to make banks lend more.
 A spacecraft counters Earth's downward pull
 In essence, negative interest rates are by maintaining enough horizontal speed to
expected to boost economic activity when continually slide sideways as it simultaneously
all other efforts fail. falls towards the planet.
4. c  The continuous free fall around the planet
 The Supreme Court has recently upheld the gives astronauts the impression of being
disqualification of 17 dissident Congress and weightless and floating.
Janata Dal (Secular) legislators by then 7. c
Karnataka Assembly Speaker K.R. Ramesh
Kumar under the Tenth Schedule (anti- 8. d

www.shankariasacademy.com | www.iasparliament.com
50

 The 6th World Congress is being jointly d. Neither 1 nor 2


hosted by Asia-Pacific Rural and
Agricultural Credit
Association (APRACA), National Bank 3) Consider the following statements with
for Agriculture and Rural respect toParis Peace Forum
Development (NABARD) and Ministry of
1. It is an international event on global
Agriculture, Government of India.
governance and multilateralism, held
 This 6th version of the congress was attended annually in Paris, France.
by 300 delegates across the globe who engaged 2. It is under the ambit of UNFCC
in interactive discussions to unleash the
potential role of rural and agricultural finance. 3. The only theme of the forum is the
environment.
9. c
Which of the statement(s) given above is/are
10. c correct?
 Savannas are Tropical Grasslands. a. 1 only
b. 2 only
15-11-2019 c. 3 only
1) Consider the following statements with d. None of the above
respect to Atal Innovation Mission (AIM)
1. It is a flagship initiative set up by
theMinistry of skill development. 4) ‘ShishuSuraksha app’ which was in news
recently was launched in
2. Atal Tinkering Labs and Mentor India
Campaign are the only two initiatives a. Arunachal Pradesh
under AIM. b. Sikkim
Which of the statement(s) given above is/are c. Punjab
correct?
d. Assam
a. 1 only
b. 2 only
5) Consider the following statements with
c. Both 1 & 2 respect toHAYABUSA 2
d. Neither 1 nor 2 1. It is a tiny unmanned spacecraft that
has been sampling the surface of the
near-earth asteroid Ryugu.
2) Consider the following statements with
respect toWholesale Price Index (WPI) 2. It is the mission of Japan Space
Exploration Agency (JAXA)
1. It measures and tracks the changes in
the price of goods and services in the 3. It is the second Japanese spacecraft to
stages before the retail level. land on an asteroid.

2. India uses WPI changes as a central Which of the statement(s) given above is/are
measure of inflation. correct?

Which of the statement(s) given above a. 1 and 2 only


is/are not correct? b. 2 and 3 only
a. 1 only c. 1 and 3 only
b. 2 only d. 1,2 and 3 only
c. Both 1 & 2

www.shankariasacademy.com | www.iasparliament.com
51

Answers  Some countries use WPI changes as a central


measure of inflation, but India has adopted
1. d new CPI to measure inflation.
 Recently, a group of selected student  WPI is published by the Economic Adviser in
innovators of the Atal Tinkering Lab the Ministry of Commerce and Industry, where
Marathon 2018, called on the President of CPI is released by CSO.
India.
 India uses the base year 2011-12 for calculating
 The Atal Innovation Mission (AIM) is a WPI.
flagship initiative set up by the NITI Aayog to
promote innovation and entrepreneurship 3. a
across the length and breadth of the country.
 Recently, the second edition of annual Paris
 There are six major initiatives under Peace Forum was held at Paris, France from 11
AIM: to 13thNovember 2019.
1. Atal Tinkering Labs - Creating problem-  The Paris Peace Forum is part of the
solving mindset across schools in India. momentum launched by French
President, Emmanuel Macron, to reaffirm
2. Atal Incubation Centers - Fostering world the importance of multilateralism and
class start-ups and adding a new dimension to collective action in facing current challenges.
the incubator model.
 It is not under the ambit of UNFCC.
3. Atal New India Challenges - Fostering
product innovations and aligning them to the  It is an international event on global
needs of various sectors/ministries. governance and multilateralism, held annually
4. Mentor India Campaign - A national in Paris, France.
Mentor network in collaboration with public  It has a specific focus on governance solutions
sector, corporates and institutions, to support in six major themes:
all the initiatives of the mission.
o Peace & Security
5. Atal Community Innovation Center - To
stimulate community centric innovation and o Development
ideas in the unserved /underserved regions of o Environment
the country including Tier 2 and Tier 3 cities.
o New technologies
6. ARISE (Atal Research & Innovation for
Small Enterprises) - To stimulate o Inclusive economy
innovation and research in the MSME o Culture & Education
industry.
 Its objective - encouraging international
2. c cooperation and collective action for a peaceful
 India’s October wholesale price world;
inflation WPIeases to 0.16%.  Its approach are presentations, discussions
 A wholesale price index (WPI) is an index that and progress on innovative, real-world
measures and tracks the changes in the solutions.
price of goods(not services) in the stages 4. d
before the retail level – that is, goods that are
sold in bulk and traded between entities or  On Children’s Day, Assam got a mobile
businesses instead of consumers. application that will enable people to lodge
a complaint about the violation of child
 WPI is unlike the Consumer Price Index rights.
(CPI), which tracks the prices of goods
and services purchased by consumers.  “ShishuSuraksha”, the mobile application,
is the brainchild of Assam State Commission
for Protection of Child Rights (ASCPCR).

www.shankariasacademy.com | www.iasparliament.com
52

 The purpose of the e-box is to empower 2) Consider the following statements with
citizens to take moral responsibility of respect to Sisseri River Bridge
protecting our future generations.
1. It is a part of Trans Arunachal Highway
 The app can be used by anyone in Assam, and is located at Lower Dibang Valley in
coming across any incident of children’s rights the state of Arunachal Pradesh.
violation, to lodge a complaint which will be 2. It was constructed by Project Indira of
directly registered at ASCPCR. Border Roads Organisation (BRO).
5. d Which of the statement(s) given above
 Recently, HAYABUSA 2 spacecraft is/are incorrect?
starts yearlong journeyto Earth with samples a. 1 only
from a nearby asteroid.
b. 2 only
 It is a tiny unmanned spacecraftthat has
been sampling the surface of the near-Earth c. Both 1 and 2
asteroid 162173Ryugu, capturing images, d. Neither 1 nor 2
blasting a little crater in it, and firing a
"bullet" into its exterior to dislodge particles.
 It is the mission of Japan Space 3) Which of the following are projects
Exploration Agency (JAXA). of Border Roads Organisation (BRO) to
maintain and develop the difficult geographical
 Hayabusa2 is the second Japanese terrain and strategically important area
spacecraft to land on an asteroid, after in Arunachal Pradesh?
Hayabusa.
1. Yeti
 Hayabusatouched down on a small near-
Earth asteroid called 25143 Itokawa and 2. Indira
safely brought back minute rocky particlesfor 3. Udayak
analysis.
4. Arunank
5. Brahmank
16-11-2019 6. Abhimanyu
1) With respect to Online Child Sexual Abuse Select the correct answer using the codes given
and Exploitation (OCSAE) below:
Prevention/Investigation Unit, consider the
following statements: a. 2, 4 and 5 only
1. The unit does not have any power to b. 3, 4 and 5 only
probe offences covered under the
c. 3, 5 and 6 only
Protection of Children from Sexual
Offences (POCSO) Act and the d. All of the above
Information Technology (IT) Act.
2. It was setup by the Ministry of Women
4) Health, Safety and Dignity of Sanitation
and Child Development.
Workers Report was authored by?
Which of the statement(s) given above is/are
a. World Bank
correct?
b. WaterAid
a. 1 only
c. International Labour Organization
b. 2 only
d. All of the above
c. Both 1 and 2
d. Neither 1 nor 2

www.shankariasacademy.com | www.iasparliament.com
53

5) Bali Yatra which was in news recently is 2. It is under the Ministry of Skill
celebrated in? Development and Entrepreneurship.
a. West Bengal Which of the statement(s) given above is/are
correct?
b. Chhattisgarh
a. 1 only
c. Andaman and Nicobar Isalnds
b. 2 only
d. Odisha
c. Both 1 and 2
d. Neither 1 nor 2
6) Consider the following statements with
respect to Bharatiya Poshan Krishi Kosh
1. It will be a repository of diverse crops 9) Climate Protection Act which was in news
across 128 agro-climatic zones in India recently is passed in?
for better nutritional outcomes.
a. New Zealand
2. It is under the Ministry of Women and
b. Bhutan
Child Development (WCD).
c. Germany
Which of the statement(s) given above is/are
correct? d. Norway
a. 1 only
b. 2 only 10) Consider the following pairs
c. Both 1 and 2 1. Anpara Power Plant – Madhya Pradesh
d. Neither 1 nor 2 2. Kudankulam Nuclear Power Plant –
Telangana
Which of the pair(s) given above is/are correct?
7) Consider the following statements with
respect to Household Consumer Expenditure a. 1 only
Survey
b. 2 only
1. It collects information regarding
expenditure on consumption of goods c. Both 1 and 2
and services (food and non-food) d. Neither 1 nor 2
consumed by households.
2. It is carried out by NITI Aayog.
Answers
Which of the statement(s) given above is/are
correct? 1. d
a. 1 only  The Central Bureau of Investigation
(CBI) has set up an Online Child Sexual Abuse
b. 2 only and Exploitation (OCSAE)
c. 3 only Prevention/Investigation Unit.
d. None of the above  It will also probe such offences covered
under various provisions of the IPC,
the Protection of Children from Sexual
8) Consider the following statements with Offences Act and the Information
respect to IndiaSkills 2020 Technology Act, apart from other relevant
laws.
1. It is an initiative to train Indian
graduates in Artificial Intelligence (AI).  The unit will function under the agency's
Special Crime Zone in Delhi.

www.shankariasacademy.com | www.iasparliament.com
54

 It will collect and disseminate  Bali Yatra is a unique festival


information on online child sexual abuse in Odisha which celebrates India’s ancient
and exploitation. links to Indonesia!
2. b  It is organised at the banks of the river
Mahanadi, every year from the day of Kartika
 Union Defence Minister has recently
Purnima according to the Odia Calendar.
inaugurated the important Sisseri River
Bridge at Lower Dibang Valley  The festival marks the start of the journey of
in Arunachal Pradesh. hundreds of traders from the state who would
set sail to the distant land of Bali, Sumatra,
 It is a 200-metre long bridge between Jonai-
Java (Jawa) in Indonesia, Borneo and Sri
Pasighat-Ranaghat-Roing road and will
Lanka, for the expansion of trade and culture.
provide connectivity between Dibang Valley
and Siang.  It is one of the oldest cultures and ancient
business trade fair practices of the state.
 The Sisseri River Bridge provides connectivity
to Tinsukia via Dhola-Sadia bridge. 6. c
 It was constructed by Project  Bharatiya Poshan Krishi Kosh, a
Brahmank of Border Roads repository of diverse crops across 128 agro-
Organisation (BRO). climatic zones in India, will be launched for
better nutritional outcomes and to fight
 This bridge is also strategically important malnutrition.
from a military viewpoint and will be a part
of Trans Arunachal Highway.  It is under the Ministry of Women and Child
Development.
3. b
 In consultation with Ministry of WCD and Bill
 To maintain and develop the difficult
& Melinda Gates Foundation, the project team
geographical terrain and strategically
will select around 12 high focus states which
important area, BRO has four projects in
are representative of the geographical, social,
Arunachal Pradesh
economic, cultural and structural diversities of
namely Vartak, Arunank, Brahmank and
India.
Udayak.
 In each of the states or group of states the team
 It immensely contributes to the socio-
will identify a local partner organization which
economic development of the region.
has relevant work experience in Social and
4. d Behavior Change Communication (SBCC) and
nutrition for developing the food atlas.
 Despite laws and regulations being in place in
India, the practice of manual scavenging has 7. a
not been curtailed but has been forced
 Recently, government scraps the Household
underground, a report jointly authored by
Consumer Expenditure Survey 2017-
the International Labour Organization,
18 over “data quality”.
Water-Aid, World Bank, and WHO.
 The Household Consumer Expenditure Survey
 The most undesirable and high-risk jobs are
generates estimates of household Monthly
typically subcontracted to temporary, informal
Per Capita Consumer Expenditure
workers despite a substantial formal and
(MPCE) and the distribution of households
permanent sanitation workforce now being
and persons over the MPCE classes.
present in India, with basic working conditions
protected by law, the report highlighted.  It is designed to collect information regarding
expenditure on consumption of goods and
5. d
services (food and non-food) consumed by
 Recently, Bali Yatra was inaugurated in households.
Cuttack on November 12.

www.shankariasacademy.com | www.iasparliament.com
55

 The results, after release, are also used for  Last week, New Zealand passed the Zero-
rebasing of the GDP and other macro- Carbon Law in a bid to comply with it’s Paris
economic indicators. climate accord commitments and become a
carbon-neutral nation by the year 2050.
 The National Statistical Office
(NSO), Ministry of Statistics and 10. d
Programme Implementation carried out
1. Anpara Power Plant – Uttar Pradesh
this all-India survey.
2. Kudankulam Nuclear Power Plant – Tamil
 It is usually conducted at quinquennial Nadu
intervals.
8. b
18-11-2019
 IndiaSkills 2020 provides a platform for
skilled and talented Indian 1) Signal Fish which was in news recently was
youngsters to showcase their abilities at discovered in?
regional and national level competitions in
a. Andaman and Nicobar Islands
over 50 skills.
b. Gulf of Khambhat
 It is conducted by the Ministery of Skill
Development and Entrepreneurship. c. Kerala coast
 The age limit for the competition is for youth d. Gulf of Mannar
born on or after 1 January 1999 with an
exemption.
2) Which of the following are the goals of
 It is a biennial competition to scout talent the Curiosity Rover?
from the country, offering them a platform to
showcase their skills at national and 1. Determine whether life ever arose on
international competitions. Mars.
 The last IndiaSkills competition was organized 2. Characterize the climate of Mars.
in 2018, wherein 22 states and more than 100 3. Characterize the geology of Mars.
corporates participated, that gave 355
competitors a chance to display their skills 4. Prepare for human exploration.
across various skill competitions. Select the correct answer using the codes given
9. c below:
 Recently, the German parliament passed a. 1 and 2 only
the Climate Protection Act in an attempt to b. 1, 2 and 4 only
reach its climate target by 2050.
c. 1, 2 and 3 only
 This will be Germany’s first climate action
law. d. 1, 2, 3 and 4

 With this, a price on carbon emissions in the


transport and heating sectors will be imposed 3) Consider the following statements with
along with some other measures to combat respect to Places of Worship Act, 1991
climate change.
1. The aim of the Act was to freeze the
 Furthermore, German lawmakers also status of any place of worship as it
supported to hike the prices of domestic and existed on August 15, 1947.
European flight tickets.
2. The Act does not apply to the place of
 From 2021, companies that market diesel and worship commonly referred to as Ram
petrol, heating oil and natural gas in the Janmabhoomi-Babri Masjid in Ayodhya.
country will need to obtain pollution rights for
the amount of greenhouse gases they emit.

www.shankariasacademy.com | www.iasparliament.com
56

3. This law will have overriding effect over b. 2 only


any other law in force.
c. Both 1 and 2
Which of the statement(s) given above is/are
d. Neither 1 nor 2
correct?
a. 3 only
6) CASE Instrument sometimes seen in the
b. 1 and 2 only
news recently was contributed by _________
c. 1, 2 and 3 for _________ Mission?
d. None a. NASA – ESA
b. ESA – NASA
4) Consider the following statements with c. NASA – ROSCOSMOS
respect to Bharatiya Poshan Krishi Kosh
d. JAXA – ROSCOSMOS
1. It is a repository of diverse crops across
128 agro-climatic zones for better
nutritional outcomes. 7) Consider the following statements
2. It aims at reducing malnutrition 1. He is also called as Punjab Kesari.
through a multi-sectoral results-based
framework, including agriculture, 2. His important works include ‘The Arya
among women and children across the Samaj’, ‘Young India’, and ‘England’s
country. Debt to India’.

3. Ministry of Women and Child 3. He died at Lahore in 1928 after he was


Development has collaborated with Bill attacked by police during a protest rally
and Melinda Gates Foundation for this against the Simon Commission.
project. Identify the personality who correctly matches
Which of the statement(s) given above is/are with the above description:
correct? a. Bal Gangadhar Tilak
a. 1 only b. Bipin Chandra Pal
b. 1 and 2 only c. Lala Lajpat Rai
c. 1 and 3 only d. Abul Kalam Azad
d. 1, 2 and 3
8) Consider the following statements with
5) Consider the following statements with respect to Diesel
respect to Health Systems for a New India: 1. Below the Cloud point, Diesel is prone
Building Blocks Report which was released to waxing.
recently:
2. A special winter-grade diesel with a low
1. It was released recently by the Ministry pour-point of -33 deg C, does not lose its
of Health and Family Welfare. fluidity even in extreme winter
2. The report has out rightly suggested conditions.
that Pradhan Mantri Jan Arogya Yojana Which of the statement(s) given above is/are
(PM-JAY) should be extended to whole correct?
of India.
a. 1 only
Which of the statement(s) given above is/are
correct? b. 2 only

a. 1 only c. Both 1 and 2

www.shankariasacademy.com | www.iasparliament.com
57

d. Neither 1 nor 2 of 70 metres during a marine biodiversity


survey.
 The signal fishes are spotted in the coral
9) Consider the following statements with
reefs. So it indicates the presence of patchy
respect to Epilepsy
corals off the Kerala coast.
1. It is a chronic communicable disease of
2. d
the brain that affects people of all ages.
2. It is seen as a major health problem and  According to the new study, Oxygen behaves in
it is considered to be a Mental health a mysterious way on Mars.
disorder.  NASA’s Curiosity rover’s has added a new
Which of the statement(s) given above is/are mystery for scientists studying Mars, with
correct? samples from the rover showing a seasonal
change around Oxygen levels near the surface
a. 1 only of the Gale Crater on the red planet.
b. 2 only  NASA found that Oxygen levels throughout
c. Both 1 and 2 spring and summer rose by as much as 30%.

d. Neither 1 nor 2  According to NASA, Curiosity has 4 main


science goals in support of the agency's Mars
exploration program:
10) ‘Nadu-Nedu’ scheme which was in news 1. Determine whether life ever arose on Mars.
recently was launched in
2. Characterize the climate of Mars.
a. Andhra Pradesh
3. Characterize the geology of Mars.
b. Kerala
4. Prepare for human exploration.
c. Telangana
3. c
d. Karnataka
 The Act declares that the religious character of
a place of worship shall continue to be the
Answers same as it was on August 15, 1947.

1. c  It says no person shall convert any place of


worship of any religious denomination into
 India's first Signal Fish was found in Kerala one of a different denomination or section.
deep waters.
 It was intended to pre-empt new claims by any
group about the past status of any place of
worship and attempts to reclaim the structures
or the land on which they stood.
 It declares that all suits, appeals or any other
proceedings regarding converting the
character of a place of worship, which are
pending before any court or authority on
August 15, 1947, will abate as soon as the law
comes into force.
 No further legal proceedings can be instituted.
 The discovered species is the world’s largest  However, there is an exception to the bar on
Signal Fish. instituting fresh proceedings with regard to
 The species named Pteropsaron suits that related to conversion of status that
indicum was collected by trawlers at a depth happened after August 15, 1947.

www.shankariasacademy.com | www.iasparliament.com
58

 This saves legal proceedings, suits and appeals  The report identified five focus areas of future
regarding chance of status that took place after health system - to deliver on unfinished public
the cut-off date. health agenda, change health financing away
from out of pocket so spend into large insurers,
 These provisions will not apply to integrate service delivery vertically and
1. ancient and historical monuments and horizontally, empower citizens to become
archaeological sites and remains that are better buyers of health, harness the power of
covered by the Ancient Monuments and digital health.
Archaeological Sites and Remains Act, 1958
 Even as the report has not out rightly
2. a suit that has been finally settled or disposed suggested that Pradhan Mantri Jan Arogya
of and Yojana (PM-JAY), government’s cashless
health insurance scheme covering 10 crore
3. any dispute that has been settled by the parties poor families for Rs five lakh annually, should
or conversion of any place that took place by be extended to whole of India, it discretely
acquiescence before the Act commenced. mentioned that PM-JAY should be considered
 The Act does not apply to the place of worship with an eye on its potential to influence the
commonly referred to as Ram Janmabhoomi- overall healthcare transformation in India,
Babri Masjid in Ayodhya. beyond its current explicit mandate.

 This law will have overriding effect over any 6. a


other law in force, it said.  NASA has announced that it will contribute an
4. d instrument to a European space mission that
will explore the atmospheres of planets or exo-
 Union Government has recently planets orbiting stars beyond our Sun for the
announced Bharatiya Poshan Krishi first time.
Kosh which is a repository of diverse crops
across 128 agro-climatic zones for better  The instrument will aid the European Space
nutritional outcomes. Agency’s (ESA’s) ARIEL or Atmospheric
Remote-sensing Infrared Exoplanet
 The Kosh aims at reducing malnutrition Large-survey mission.
through a multi-sectoral results-based
framework, including agriculture, among  NASA’s instrument has been called
women and children across the country. the CASE or Contribution to ARIEL
Spectroscopy of Exoplanets and it will be
 Ministry of Women and Child Development managed by NASA’s Jet Propulsion Laboratory
has collaborated with Bill and Melinda Gates in Pasadena, California.
Foundation for this project.
 The ARIEL spacecraft with CASE on board is
 The Bharatiya Poshan Krishi Kosh seeks to expected to launch in 2028.
promote healthy dietary practices and tackle
under-nutrition in a sustainable manner. 7. c

 The initiative will be built on an essential  Lala Lajpat Rai, the firebrand Indian
understanding of social, behavioural and nationalist leader was affectionately called as
cultural practices that promote and reinforce ‘Punjab Kesari’.
healthy dietary behaviours both at the  Rai is remembered for his role during the
individual and community level. Swadeshi movement and for his advocacy of
5. d education.

 Government think tank Niti Aayog recently  The patriot died at Lahore in 1928 after he was
released a report ‘Health Systems for a attacked by police during a protest rally
New India: Building Blocks—Potential against the Simon Commission.
Pathways to Reforms’.  His important works include: ‘The Arya
Samaj’, ‘Young India’, ‘England’s Debt to
India’, ‘Evolution of Japan’, ‘India’s Will to

www.shankariasacademy.com | www.iasparliament.com
59

Freedom’, ‘Message of the Bhagwad Gita’,  In many parts of the world, people with
‘Political Future of India’, ‘Problem of National epilepsy and their families suffer from stigma
Education in India’, ‘The Depressed Glasses’, and discrimination.
and the travelogue ‘United States of America’.
10. a
8. c
 Andhra Pradesh government has launched
 Recently, 1st Winter-Grade Diesel the ‘Nadu-Nedu’ programme.
outlet was inaugurated in Ladakh.
 The programme aims to introduce English
 The move will boost tourism in extreme medium from classes 1 to 6 in state-run
weather conditions and help in the economic schools.
development of the region.
 The scheme will be implemented in 15,715
 Under normal conditions the paraffin schools in the first phase.
wax added to diesel improves viscosity and
lubrication.  It will cover all schools in three years’ time
with a budget of Rs 12,000 crores.
 However, when temperatures begin to fall, this
paraffin wax thickens and turns into a cloudy  This will transform all government schools
mixture. This point is known as a fuel’s ‘Cloud with the required infrastructure and
point’. upgradation of skills besides setting up of
English labs in addition to providing basic
 Indian Oil has come up with an innovative amenities like clean environs, furniture, toilets,
solution to this problem by introducing a running water, compound wall, blackboard,
special Winter-grade diesel with a low renovation and involve parent committee.
pour-point of -33 deg Celsius, which does
not lose its fluidity function even in extreme
winter conditions. 19-11-2019
 This winter grade diesel also meets BIS 1) Consider the following statements with
specification of BS-VI grade diesel. respect to ASEAN Defence Ministers’ Meeting-
9. d Plus (ADMM-Plus)

 In India, November 17th is observed every year 1. ADMM-Plus is a platform comprising


as National Epilepsy Day to create ASEAN and its eight dialogue partners
awareness about epilepsy. including India and United States.

 It is a chronic noncommunicable 2. The grouping focuses its work on


disease of the brain that affects people of all maritime security, counter-terrorism,
ages. humanitarian assistance and cyber
Security.
 It is seen as a major health problem that was
3. India is not participating in the 6th
earlier considered to be a ‘Mental health
edition of ADMM-Plus in response to
disorder’, however, with advancement in
the failure in reaching consensus for
diagnosis and epidemiological studies, epilepsy
Regional Comprehensive Economic
is now considered a ‘Neurological
Partnership (RCEP) Agreement.
disorder’.
Which of the statement(s) given above is/are
 Around 50 million people worldwide have correct?
epilepsy, making it one of the most common
neurological diseases globally. a. 2 only
 Nearly 80% of people with epilepsy live in low- b. 1 and 2 only
and middle-income countries. c. 2 and 3 only
 It is estimated that up to 70% of people living d. 1, 2 and 3
with epilepsy could live seizure- free if
properly diagnosed and treated.

www.shankariasacademy.com | www.iasparliament.com
60

2) Consider the following statements with d. Neither 1 nor 2


respect to Kimberley Process Certification
Scheme (KPCS)
5) Consider the following statements with
1. Kimberley Process is a joint initiative
respect to Exercise Za’ir-Al-Bahr
involving Government, International
Diamond Industry and Civil Society to 1. It is a multilateral maritime exercise
stem the flow of Rough Diamonds. held between members of Gulf
Cooperation Council (GCC).
2. The Chair of Kimberley Process is
selected annually based on election 2. For the first time, India and China is
among the member countries. participating in this exercise as special
observers.
3. India is one of the founder members of
Kimberley Process Certification Scheme Which of the statement(s) given above is/are
and is the Chair of Kimberley Process correct?
for the year 2019.
a. 1 only
Which of the statement(s) given above is/are
correct? b. 2 only

a. 3 only c. Both 1 and 2

b. 1 and 3 only d. Neither 1 nor 2

c. 2 and 3 only
d. None 6) Consider the following statements with
respect to Starlink satellites which was in news
recently
3) Injeti Srinivas Committee sometimes seen 1. It aimed at providing low-cost and
in the news recently is associated with which of reliable space-based Internet services to
the following? the world.
a. Corporate Governance 2. It was launched by the European Space
Agency (ESA)
b. New Direct Tax Legislation
Which of the statement(s) given above is/are
c. Decriminalisation of Company Law
correct?
d. Water management in North East
a. 1 only
b. 2 only
4) Consider the following statements with
c. Both 1 and 2
respect to National Mission on Cultural
Mapping (NMCM) d. Neither 1 nor 2
1. The Mission will compile data of artists,
art forms & geo location with inputs
from Central Ministries, State 7) Consider the following statements with
Governments & art/culture bodies. respect to Pradhan Mantri Matru Vandana
Yojana (PMMVY)
2. It has been set up by the Ministry of
Culture in 2017. 1. It is to support lactating mothers and
pregnant women by compensating them
Which of the statement(s) given above is/are for loss of wages during their
correct? pregnancy.
a. 1 only 2. It is targeted only at women delivering
their first child.
b. 2 only
c. Both 1 and 2

www.shankariasacademy.com | www.iasparliament.com
61

Which of the statement(s) given above is/are Which of the statement(s) given above is/are
correct? correct?
a. 1 only a. 1 only
b. 2 only b. 2 only
c. Both 1 and 2 c. Both 1 and 2
d. Neither 1 nor 2 d. Neither 1 nor 2

8) Which of the following countries have a Answers


coastline on the Barents Sea?
1. b
1. Norway
 Consistent with the ADMM guiding principles
2. Finland of open and outward looking, the 2ndADMM
3. Russia in Singapore in 2007 adopted the Concept
Paper to establish the ADMM-Plus.
4. Sweden
 The ADMM-Plus is a platform for ASEAN and
Choose the correct option from the option its eight Dialogue Partners to strengthen
given below security and defence cooperation for peace,
a. 1 and 2 only stability, and development in the region.

b. 1 and 3 only  The ADMM-Plus countries include ten ASEAN


Member States, namely, Brunei Darussalam,
c. 2 and 3 only Cambodia, Indonesia, Lao PDR, Malaysia,
d. 1, 2 and 4 only Myanmar, the Philippines, Singapore,
Thailand and Viet Nam, and eight Plus
countries, namely Australia, China, India,
9) Consider the following statements with Japan, New Zealand, ROK, Russian Federation
respect to Agni II and the United States.

1. It is a surface-to-surface medium range  The Inaugural ADMM-Plus was convened in


nuclear-capable missile. Ha Noi, Viet Nam, on 12 October 2010.
2. It is developed by the Defence Research  The Defence Ministers then agreed on five
and Development Organisation (DRDO). areas of practical cooperation to pursue under
this new mechanism, namely maritime
Which of the statement(s) given above is/are security, counter-
correct? terrorism, humanitarian assistance
a. 1 only and disaster relief, peacekeeping
operations and military medicine.
b. 2 only
 To facilitate cooperation on these areas,
c. Both 1 and 2 Experts' Working Groups (EWGs) are
d. Neither 1 nor 2 established.
 In 2013 and 2016, new priority areas
like humanitarian mine action and
10) Consider the following statements with cyber security were agreed respectively.
respect to World Gold Council
 Union Minister for Home Affairs
1. It is an association under International
recently attended the 6th ASEAN
Council on Mining and Metals (ICMM).
Defence Ministers’ Meeting – Plus.
2. Its members includes most of the
world’s largest gold mining companies.

www.shankariasacademy.com | www.iasparliament.com
62

2. a remove criminality and de-clog the criminal


justice system.
 The Kimberley Process is a joint initiative
involving Government, International Diamond  The government had earlier, through the
Industry and Civil Society to stem the flow Companies (Amendment) Act 2019,
of Conflict Diamonds. decriminalised 16 minor procedural or
technical lapses and had categorised them as
 “Conflict Diamonds” means rough ‘civil wrongs’.
diamonds used by rebel movements or
their allies to finance conflict aimed at 4. c
undermining legitimate governments.
 National Mission on Cultural Mapping
 It is also described in United Nations Security (NMCM) has been set up by the Ministry of
Council (UNSC) resolutions. Culture in 2017.
 At present, KPCS has 55 members  Mission will compile data of artists, art forms
representing 82 countries including EU with & geo location with inputs from Central
28 members. Ministries, State Governments & art/culture
bodies.
 The Kimberley Process is chaired, on a
rotating basis, by participating countries.  Specially designed data capture form with
technical collaboration of National E-
 KP Vice-Chair is generally elected by Governance Division (NEGD)/Ministry of
KP Plenary each year who becomes the Electronics and Information Technology
Chair in the next year. (MEITY) has been formulated for data
 India is the current Chair of KPCS with collection.
Russian Federation as Vice Chair for the year 5. d
2019.
 Navies of India and Qatar have started a
 Since 2003, India has been actively five-day bilateral maritime
participating in the KPCS process and is a exercise Za'ir-Al-Bahr (Roar of the
member of all Working Groups of KP (except Sea) at Doha that includes surface action, air
for WGAAP). defence, maritime surveillance as well as social
 Department of Commerce is the nodal and sports events.
Department and Gem & Jewellery  The inaugural edition of the bilateral maritime
Export Promotion Council (GJEPC) is exercise would further strengthen the robust
designated as the KPCS Importing and defence co-operation between the two
Exporting Authority in India. countries, especially in the fight against
 GJEPC is responsible for issuing KP terrorism, maritime piracy and security.
Certificates and is also the custodian of KP 6. a
Certificates received in the country.
 A spray of 60 Starlink satellites fired into
3. c orbit, the first operational batch of what is
 The government had set up the Company intended to eventually evolve into
Law Committee under the chairmanship of a constellation of nearly 12,000
Corporate Affairs Secretary Injeti satellites.
Srinivas to further decriminalise the  It aimed at providing low-cost and
provisions of the Companies Act, 2013, reliable space-based Internet services to
based on the gravity of the offences and to take the world.
other measures to “provide further ease of
living for corporates in the country”.  It was launched by SpaceX, the world’s
leading private company in space technology.
 The government-appointed high-level panel
has recently recommended amendments to 46  But astronomers worry the SpaceX satellites
provisions under the company law to reduce or are too bright and could form a

www.shankariasacademy.com | www.iasparliament.com
63

"megaconstellation" that blots out the star and 8. b


interferes with the work of telescopes.
 Recently, it was found that King Crabs,
 Astronomers and scientists have also which are not native to Barents sea, have
complained about increased “light-pollution”. slowly conquered the region ever since their
introduction.
 Climate change isalso making it easier to fish
the crabs, with much of the Barents ice-free for
longer months.
 The Barents Sea is classified as a ‘Marginal
sea of the Arctic Ocean’.
 Climatic changes have led to the Barents Sea
having characteristics similar to the Atlantic
Ocean.
 The Barents Sea is off the northern coasts
of Russia and Norway.

7. c
 A recent survey found that around Maternity
scheme reaches only one-third of beneficiaries.
 The exercise, titled Jaccha Baccha-Survey
(JABS), is critical of the Pradhan Mantri
Mantru Vandana Yojana (PMMVY), launched
in 2017 as part of obligations under the
National Food Security Act.
 Researchers assert that extrapolation of RTI
data show only 31% of eligible mothers got
benefits.
 Almost 63% of pregnant women in rural India
work right until the day of delivery.
 And, 49% say they felt exhausted during
pregnancy, due to lack of food and rest, found
in the survey.
 Pradhan Mantri Matru Vandana Yojana
(PMMVY) is a vital programme to support
lactating mothers and pregnant  The Russian and Norwegian governments were
women by compensating them for loss of involved in a border dispute over the position
wages during their pregnancy. of their respective border in the sea.
9. c
 The PMMVY is targeted only at women
delivering their first child.  India on Saturday successfully conducted the
first, night trial of Agni-II from the Abdul
 A cash amount of Rs. 6,000 is transferred to
Kalam Island off the Odisha coast.
the bank account of the beneficiary in 3
instalments upon meeting certain conditions  It is a surface-to-surface medium
including early registration of pregnancy, range nuclear-capable missile.
having at least one ante-natal check-up and
registration of child birth.  Agni-II, an intermediate range ballistic missile
(IRBM), has already been inducted into the
armed forces.

www.shankariasacademy.com | www.iasparliament.com
64

 Agni-II was developed by Advanced Systems 2) Consider the following statements with
Laboratory along with respect to End-to-end encryption Technology
other DRDO laboratories and integrated by
1. It scrambles messages in such a way
the Bharat Dynamics Limited.
that they can be deciphered only by the
10. b sender and the intended recipient.
 Gold remains the third-most popular 2. It steers governments away from mass
investment choice, with 46% of all global retail surveillance and toward a more
investors choosing gold products, next only to targeted, constitutional form of
saving accounts (78%) and life insurance intelligence gathering.
(54%), reveals a new World Gold Council 3. Law enforcement agencies can get
(WGC) survey of retail consumers. access to messages encrypted through
 The World Gold Council is the market this technology by presenting
development organisation for the gold technology companies with a warrant.
industry. Which of the statement(s) given above is/are
 It is a nonprofit association of the world's correct?
leading gold producers and it does not come a. 2 only
under ICMM.
b. 1 and 2 only
 Its purpose is to stimulate and sustain demand
for gold, provide industry leadership, and be c. 1 and 3 only
the global authority on the gold market. d. 1, 2 and 3
 Its members includes most of the world’s
largest gold mining companies.
3) Consider the following statements with
 It is based in the UK, with operations in India, respect to Mission Parivar Vikas
China, Singapore and the US , and it is the lead
for the gold industry worldwide. 1. It focus only on districts having the
highest total fertility rates in the
country.
20-11-2019 2. The main objective is to accelerate
access to high quality family planning
1) Consider the following statements with choices based on information, reliable
respect to Arundhati scheme services and supplies within a rights-
1. Under the Scheme, government of West based framework.
Bengal provides 1 Tola Gold to Brides Which of the statement(s) given above
parents. is/are not correct?
2. The scheme is limited only to the a. 1 only
weaker section of the society whose
annual income is below Rs 5 lakh b. 2 only
irrespective of caste, creed and religion. c. Both 1 and 2
Which of the statement(s) given above is/are d. Neither 1 nor 2
correct?
a. 1 only
4) ‘SAANS Campaign’ which was in news
b. 2 only recently is related to
c. Both 1 and 2 a. To reduce child mortality due to pneumonia
d. Neither 1 nor 2 b. An anti-tobacco awareness campaign
c. To create awareness among consumers

www.shankariasacademy.com | www.iasparliament.com
65

d. Cancer Awareness Campaign 2. Increasing levels of organic molecules


can enhance microbial activity and
produce more greenhouse gases.
5) Consider the following statements with
Which of the statement(s) given above is/are
respect to Central Drugs Standard Control
correct?
Organisation (CDSCO)
a. 1 only
1. CDSCO under the Ministry of chemicals
and fertilizers regulates the safety, b. 2 only
efficacy and quality of notified medical
c. Both 1 and 2
devices.
d. Neither 1 nor 2
2. Drug Controller General of India
(DCGI) an organ of CDSCO is
responsible for approving and licensing
of drugs and medical devices. 8) In Modern World History, Velvet
Revolution refers to which of the following?
Which of the statement(s) given above is/are
correct? a. Fall of Berlin Wall

a. 1 only b. Modernization efforts in Iran

b. 2 only c. Abolition of slavery in Haiti

c. Both 1 and 2 d. Fall of communist control over Czechoslovakia

d. Neither 1 nor 2
9) Apart from Farm fires which of the following
is/are the main reasons for Delhi’s poor air
6) Consider the following statements with quality?
respect to Tuberculosis (TB)
1. Wind Speed
1. The National Strategic Plan aims to end
TB by 2030 in India inline with the 2. Wind direction
global target. 3. Geographic location
2. NI-KSHAY functions as the National TB Select the correct answer using the codes given
Surveillance System and enables below:
reporting of various surveillance data
both in the public and private sector to a. 1 and 2 only
the Government of India. b. 1 and 3 only
Which of the statement(s) given above is/are c. 2 and 3 only
correct?
d. 1, 2 and 3
a. 1 only
b. 2 only
10) Consider the following statements with
c. 3 only respect to Plague
d. None 1. Both Pneumonic and Bubonic plague
were caused by the Rattus Virus.

7) Consider the following statements with 2. Septicaemic plague is a rarer variant of


respect to Greenhouse gas emissions plague which infects the lungs.

1. Freshwater bodies do not emit Which of the statement(s) given above is/are
Greenhouse gasses. correct?
a. 1 only

www.shankariasacademy.com | www.iasparliament.com
66

b. 2 only  Think of it as a locked mailbox. Anyone with a


public key can put something in your box and
c. Both 1 and 2
lock it, but only you have the private key to
d. Neither 1 nor 2 unlock it.
How is it different from other forms of
encryption?
Answers
 A more common form of encryption, known
1. b as transport layer encryption, relies on
 In a unique initiative to check child a third party, like a tech company, to
marriage, Assam Govt approves Arundhati encrypt messages as they move across the web.
scheme to provide 1 Tola Gold (11.66 grams)  With this type of encryption, law enforcement
to Brides parents belonging to the and intelligence agencies can get access to
economically weaker sections of the society. encrypted messages by presenting technology
 With the aim to reduce child marriage under companies with a warrant or national security
the Prohibition of Child Marriage Act that a letter. The sender and recipient would not have
girl in India can’t get married before the age of to know about it.
18, and a boy can’t get married before 21 years  End-to-end encryption ensures that no
of age, this scheme can be availed by formal one can eavesdrop on the contents of a
registration of the marriage under the Special message while it is in transit.
Marriage (Assam) Rules, 1954 at the time of
marriage.  It forces spies or snoops to go directly to the
sender or recipient to read the content of the
 This is only limited to the weaker encrypted message.
section of the society whose annual income is
below Rs 5 lakh irrespective of caste, creed,  Or they must hack directly into the sender’s or
religion etc. recipient’s device, something that can be
harder to do “at scale” and makes mass
2. b
surveillance much more difficult.
End-to-end encryption Technology
 Privacy activists, libertarians, security experts
How it Works? and human rights activists argue that end-to-
end encryption steers governments away
 It scrambles messages in such a way that they from mass surveillance and toward a more
can be deciphered only by the sender and targeted, constitutional form of intelligence
the intended recipient. gathering.
 As the label implies, end-to-end encryption  But intelligence and law enforcement agencies
takes place on either end of a communication. argue that end-to-end encryption makes it
 A message is encrypted on a sender’s device, much harder to track terrorists, pedophiles
sent to the recipient’s device in an unreadable and human traffickers.
format, then decoded for the recipient. 3. d
 There are several ways to do this, but the most  The Ministry of Health and Family Welfare
popular works like this: A program on your launched “Mission Parivar Vikas” in 145
device mathematically generates two high focus districts having the highest
cryptographic keys — a public key and a total fertility rates in the country.
private key.
 These 145 districts are in the seven high focus,
 The public key can be shared with anyone who high TFR states of Uttar Pradesh, Bihar,
wants to encrypt a message to you. Rajasthan, Madhya Pradesh, Chhattisgarh,
 The private key, or secret key, decrypts Jharkhand and Assam that constitute 44% of
messages sent to you and never leaves your the country’s population.
device.

www.shankariasacademy.com | www.iasparliament.com
67

 The main objective of ‘Mission Parivas responsible for approving and licensing of
Vikas’ will be to accelerate access to high drugs and medical devices.
quality family planning choices based on
 The DCGI is advised by the Drug Technical
information, reliable services and supplies
Advisory Board (DTAB) and the Drug
within a rights-based framework.
Consultative Committee (DCC).
 These 145 districts have been identified based
6. b
on total fertility rate and service delivery
(PPIUCD and Sterilization performance) for  The National Strategic Plan 2017-25 for
immediate, special and accelerated efforts to ending TB by 2025 was developed in 2017.
reach the replacement level fertility goals of 2.1
by 2025.  India’s target for complete elimination of
Tuberculosis (TB) by 2025, is five years
4. a ahead of the global target of 2030.
 Union Minister for Health and Family Welfare  NI-KSHAY-(Ni=End, Kshay=TB) is the web
has launched SAANS Campaign which enabled patient management system for TB
stands for ‘Social Awareness and Action control under the Revised National
to Neutralise Pneumonia Successfully’. Tuberculosis Programme (RNTCP).
 The aim of the campaign is to reduce  It is developed and maintained by the Central
child mortality due to pneumonia, which TB Division (CTD), Ministry of Health and
contributes to around 15% deaths annually of Family Welfare, in collaboration with the
children under the age of five. National Informatics Centre (NIC), and the
World Health Organization Country office for
 The campaign was launched in Gandhinagar,
India.
Gujarat at the 6th National Summit on Good,
Replicable Practices and Innovations.  It also functions as the National TB
5. b Surveillance System and enables reporting
of various surveillance data to the Government
 CDSCO is the national regulatory body under of India.
the Ministry of Health and Family
Welfare for pharmaceuticals and medical  It is used by health functionaries at various
devices in India. levels across the country both in the public
and private sector, to register cases under
 It functions under the Drugs and Cosmetics their care, record treatment details, monitor
Act. treatment adherence and to transfer cases
between care providers.
 CDSCO has six zonal offices, four sub-zonal
offices, 13 port offices and seven laboratories 7. b
under its control.
 Researchers from the University of Cambridge
 Major functions of CDSCO: in the UK noted that every drop of fresh water
contains thousands of different organic
1. Regulatory control over the import of drugs molecules that have previously gone
2. Approval of new drugs and clinical trials unnoticed.
3. Meetings of Drugs Consultative Committee  The study found that Climate change may
(DCC) cause the levels of greenhouse gases
emitted by freshwater lakes to increase
4. Drugs Technical Advisory Board (DTAB), by between 1.5 and 2.7 times.
5. Approval of certain licenses as Central License
 Freshwater ecosystems constitute a small
Approving Authority is exercised by the
fraction of our planet but play a
CDSCO headquarters.
disproportionately large and critical role in the
 Drug Controller General of India global carbon cycle.
(DCGI) is an organ of the CDSCO which is
 The amount of carbon that inland waters emit
is comparable to the net amount of carbon

www.shankariasacademy.com | www.iasparliament.com
68

absorbed by living organisms on Earth’s land it was around 12 km/hr later due to which the
surface and in its oceans. air quality improved.
 Organic molecules are a food source for  At greater speeds, pollutants far away can
microbes in the lake sediments, which break reach a spot within few hours and also be
them down and release carbon dioxide and flushed out as quickly if the wind speed holds.
methane as by-products.
 Geographic location – Delhi’s location —
 Increasing levels of organic molecules on the Indo-Gangetic Plain (IGP), the most
can therefore enhance microbial activity polluted belt in the country is also one of the
and produce more greenhouse gases. main reasons behind Delhi’s Air Pollution.
 Since the same microbes can make greenhouse 10. d
gases from many different organic molecules,
 Pneumonic plague and Bubonic plague were
the diversity of organic molecules was shown
types of plague caused by the Yersinia pestis
to be more closely linked with levels of
bacterium.
greenhouse gas concentrations than the
diversity of the microbes.  Bites from infected fleas are the most common
cause of bubonic plague infection, but the
 An elevated diversity of organic molecules may
pneumonic variant — where the bacterium is
elevate greenhouse gas concentrations in
breathed into the lungs — is more dangerous
waters because there are more molecules that
because it is spread through coughing.
can be broken down by sunlight penetrating
the water.  A rarer third variant of the diseases
8. d is Septicaemic Plague, which infects
the bloodstream.
 With the so-called Velvet Revolution of
1989, Czechoslovakia freed itself
of communist control and set out to adapt 21-11-2019
its command economy to the free market.
1) Consider the following statements with
 The government introduced a program based respect to Hong Kong International
on policies of price liberalization, the opening Convention sometimes seen in the news
of markets to foreign trade and investment, recently:
internal convertibility of the country’s
currency, privatization of state-owned 1. It was adopted by UN Environment for
enterprises, and tax reform. the Safe and Environmentally Sound
Recycling of Ships in 2009.
 30th anniversary of the Velvet Revolution
was held recently. 2. India has both signed and ratified the
convention.
9. d
Which of the statement(s) given above is/are
 Wind direction and Speed – According to correct?
officials at the Central Pollution Control
a. 1 only
Board, a change in wind direction and a
sudden increase in wind velocity were b. 2 only
the main reasons behind Delhi going from
severe to poor in a little over a day. c. Both 1 and 2
d. Neither 1 nor 2
 Wind blowing northwest brings pollution from
Punjab and Haryana to Delhi. Later, the
direction changed to northerly this improves
the air quality. 2) Consider the following statements with
respect to India – Europe 29 Business Forum
 The average wind speed in Delhi was under 10
1. The Europe 29 (E29) region refers to the
km/hr. Because of an active western
29 countries in Central, Eastern and
disturbance, the wind speed gradually rose and
Northern Europe.

www.shankariasacademy.com | www.iasparliament.com
69

2. It has been organized by the Ministry of Social Defence which includes drug
External Affairs and Confederation of abuse prevention, welfare of senior
Indian Industry (CII). citizens etc.
Which of the statement(s) given above is/are 2. It is an autonomous body under the
correct? administrative control of the Ministry of
Social Justice and Empowerment.
a. 1 only
Which of the statement(s) given above is/are
b. 2 only
correct?
c. Both 1 and 2
a. 1 only
d. Neither 1 nor 2
b. 2 only
c. Both 1 and 2
3) Consider the following statements with
d. Neither 1 nor 2
respect to National Action Plan for Drug
Demand Reduction (NAPDDR)
1. It is an action plan prepared for the 6) Consider the following statements with
period from 2020-2025. respect to Idris Elba
2. It aims at reduction of adverse 1. It is a newly discovered wasp species.
consequences of drug abuse through
2. It is a parasite which infects vegetables
education, de-addiction and
and acts as a major threat for plants.
rehabilitation of affected individuals
and their families. Which of the statement(s) given above is/are
correct?
3. It was drafted by the Ministry of Health
and Family Welfare. a. 1 only
Which of the statement(s) given above is/are b. 2 only
correct?
c. Both 1 and 2
a. 2 only
d. Neither 1 nor 2
b. 1 and 2 only
c. 2 and 3 only
7) Consider the following statements with
d. 1, 2 and 3 respect to ‘moon Europa’
1. It is one of the Galilean moons orbiting
Jupiter.
4) IMD World Talent Ranking, 2019 was
released recently by which of the following? 2. NASAs Galileo spacecraft studied the
planet Jupiter and its moons.
a. INSEAD
Which of the statement(s) given above is/are
b. World Economic Forum
correct?
c. International Institute for Management
a. 1 only
Development
b. 2 only
d. Both A and C
c. Both 1 and 2
d. Neither 1 nor 2
5) Consider the following statements with
respect to National Institute of Social Defence
(NISD)
1. It is the nodal training and research
Institute for interventions in the area of

www.shankariasacademy.com | www.iasparliament.com
70

8) Consider the following statements with d. 1, 2, 3, 4, 5 and 6 only


respect to Gharial
1. Presently the wild populations of
gharials are found in Bangladesh, India Answers
and Nepal only. 1. d
2. It is listed in Schedule I of Wildlife  The International Maritime
(Protection) Act, 1972 and as Critically Organisation (IMO) adopted the Hong
Endangered on IUCN Red List. Kong International Convention for
Which of the statement(s) given above is/are the Safe and Environmentally Sound
correct? Recycling of Ships in 2009.
a. 1 only  It aimed at ensuring that ships being recycled
after reaching the end of their operational lives
b. 2 only do not pose any unnecessary risks to human
c. Both 1 and 2 health, safety and to the environment.
d. Neither 1 nor 2  Cabinet has recently approved the proposal
for enactment of Recycling of Ships Bill,
2019 and accession to the Hong Kong
9) 'Orange the World' campaign which was in International Convention for Safe and
news recently is related to Environmentally Sound Recycling of Ships,
2009.
a. Create awareness of patient safety and to make
healthcare safer  The move will help India double its ship-
recycling capacity over the next five years.
b. To help stop the further emergence and spread
of antibiotic resistanceTowards creating gender  Germany became the 13th country to accede
equality in work place to the convention. One of the conditions for
the convention to come into force is that at
c. Towards creating gender equality in work place
least 15 countries ratify it.
d. To end violence against Women around the
Background
world
 India is the leader in the global ship recycling
industry, with a share of over 30% of the
10) The Production Gap which was in news market.
recently was published by
 As per UNCTAD report on Review of Maritime
1. Stockholm Environment Institute (SEI), Transport, 2018, India had demolished 6323
2. International Institute for Sustainable tonnes in 2017, of known ship scrapping across
Development (IISD), the world.

3. Overseas Development Institute (ODI),  The ship-recycling industry is a labour-


intensive sector, but it is susceptible to
4. Centre for International Climate and concerns on environmental safety.
Environmental Research (CICERO),
2. c
5. Climate Analytics,
 India – Europe 29 Business Forum has
6. UN Environment Programme (UNEP). been recently organized by the Ministry of
Which of the statement(s) given above is/are External Affairs, Government of India
correct? and Confederation of Indian Industry
(CII) in New Delhi.
a. 1, 2, 3 and 6 only
 The Europe 29 (E29) region refers to the 29
b. 1, 2, 3, 4 and 6 only countries in Central, Eastern and
c. 1, 2, 4,5 and 6 only Northern Europe.

www.shankariasacademy.com | www.iasparliament.com
71

 The India - Europe 29 Business Forum will act prevention, transgender and other social
as a bridge and help close the geographical and defence issues.
knowledge gap between the two regions.
6. a
 The Europe 29 region comprises Albania,
 Idris Elba is tiny new wasp species recently
Liechtenstein, Austria, Lithuania, Bosnia &
discovered in Mexico.
Herzegovina, Macedonia, Bulgaria, Malta,
Croatia, Moldova, Cyprus, Montenegro, Czech  It was found living as a parasite in the
Republic, Norway, Denmark, Poland, Estonia, eggs of another insect, known as the
Romania, Finland, Serbia, Greece, Slovak ‘bagrada bug’, which is a major pest of
Republic, Hungary, Slovenia, Iceland, Sweden, cruciferous vegetables such as cabbage and
Latvia, Switzerland and Turkey. broccoli.
3. a  So researchers suggest that the species might
prove to be a "protector" for many crops.
 National Action Plan for Drug Demand
Reduction (NAPDDR) has been prepared
for 2018-2023.
 It has been drafted by the Ministry of Social
Justice and Empowerment.
 It aims at reduction of adverse consequences
of drug abuse through a multi-pronged
strategy involving education, de-addiction and
rehabilitation of affected individuals and their
families.
4. c
 According to the IMD World Talent
Ranking, 2019, India has slipped 6 places to
59 rank on the global annual list of 63
countries.
 It was released by International Institute  The discovery of this wasp marks an important
for Management Development (IMD). step towards the development of efficient and
natural control of the stink bug species
 The ranking is based on the performance in Bagrada hilaris in North America.
three main categories -- investment and
development, appeal and readiness. 7. c

5. c  Recently, NASA just detected water vapor on


a moon of Jupiter.
 The National Institute of Social Defence
(NISD), New Delhi, is an autonomous body  The water vapor above the surface of Jupiter's
under the administrative control of moon Europa for the first time.
the Ministry of Social Justice and  Europa is the one of the four Galilean
Empowerment. moons orbiting Jupiter, and the sixth-
 It is the nodal training and research closest to the planet.
Institute for interventions in the area of  It's yet another clue that Europa could host
Social Defence. alien life.
 Though social defence covers the entire gamut  Galileo spacecraft orbited Jupiter, studied
of activities and programmes for the protection the planet and made close passes by all its
of society, this Institute is currently focusing major moons.
on human resource development in the
areas of drug abuse prevention, welfare
of senior citizens, beggary

www.shankariasacademy.com | www.iasparliament.com
72

8. c 10. d
 Recently, scientists found a secret breeding  The Production Gap report is a first-of-
site of one of the world's most endangered its-kind report published jointly by,
reptiles Gharial, breeding in Nepal after 37
1. UN Environment Programme (UNEP),
years.
2. Stockholm Environment Institute (SEI),
 Gharials, sometimes called gavials, are a type
of Asian crocodilian distinguished by their 3. International Institute for Sustainable
long, thin snouts. Development (IISD),
 Historically, the range of gharial distribution 4. Overseas Development Institute (ODI),
has dwindled throughout Pakistan, Bhutan, 5. Centre for International Climate and
India, Nepal and Bangladesh. Environmental Research (CICERO),
 Presently, the wild populations of 6. Climate Analytics.
gharials can only be found in
Bangladesh, India and Nepal.  It is is the first assessment of the gap
between Paris Agreement goals and
 They are a fresh-water crocodiles. They countries’ planned production of coal,
live in deep fast-flowing rivers. oil and gas.
 They are listed in Schedule I of Wildlife  It measured the gap between planned fossil
(Protection) Act, 1972 and as Critically fuel production, climate targets of countries,
Endangered on IUCN Red List. and goals to limit warming by 1.5 degree
9. d Celsius and 2°C goals committed to under the
Paris Agreement.
 ‘Orange the World campaign’ was
organised recently by UN Women and other  This report addresses the necessary winding
United Nations agencies. down of the world’s production of fossil fuels
in order to meet climate goals.
 25th November has been designed as
the Orange Day by the UNiTE to End  The report reviews, across 10 fossil-fuel-
Violence against Women Campaign. producing countries, the policies and actions
that expand fossil fuel production and, in turn,
 The Orange color symbolizes a brighter widen the gap.
future, free of violence.
 It also provides policy options that can help
 It serves as a means of demonstrating the countries better align production with climate
solidarity in eliminating all forms of goals.
violence.
 This is especially relevant over the next year,
 Orange is therefore used as the color of as countries prepare new or updated
the International Day for the Nationally determined contributions (NDCs),
Elimination of Violence against Women. which set out their new emission reduction
 As a show of solidarity, the UNESCO globe will plans and climate pledges under the Paris
be illuminated in orange. Agreement.

 It also calls upon people from all walks of life,


across generations, to take our boldest stand
yet against rape.
 The India Gate in Delhi was lit in the
orange colour on Wednesday evening to
mark the UN campaign.

www.shankariasacademy.com | www.iasparliament.com
73

22-11-2019 Which of the statement(s) given above is/are


correct?
1) In which of the following conditions,
Government of India can terminate Citizenship a. 1 only
of an Indian? b. 2 only
1. The citizen has obtained the citizenship c. Both 1 and 2
by fraud
d. Neither 1 nor 2
2. The citizen has shown disloyalty to the
Constitution of India
3. The citizen has unlawfully traded or 4) Consider the following statements with
communicated with the enemy during a respect to Sowa Rigpa
war 1. It is a traditional Greek system of
4. The citizen has been ordinarily resident medicine.
out of India for seven years 2. The majority of theory and practice of
continuously Sowa-Rigpa are similar to Ayurveda.
Select the correct answer using the codes given Which of the statement(s) given above is/are
below: correct?
a. 1 and 3 only a. 1 only
b. 1, 2 and 4 only b. 2 only
c. 1, 3 and 4 only c. Both 1 and 2
d. 1, 2, 3 and 4 d. Neither 1 nor 2

2) Consider the following statements with 5) Consider the following statements with
respect to Tuvalu Islands respect to New Space India Limited (NSIL)
1. Tuvalu is a group of nine tiny islands in 1. It is a wholly owned Government of
the South Atlantic Ocean. India undertaking under the
2. River Salinas flows through this group administrative control of Department of
of islands. Space (DOS).
Which of the statement(s) given above is/are 2. It is the only commercial arm of the
correct? Indian Space Research Organisation
(ISRO).
a. 1 only
Which of the statement(s) given above is/are
b. 2 only correct?
c. Both 1 and 2 a. 1 only
d. Neither 1 nor 2 b. 2 only
c. Both 1 and 2
3) Consider the following statements with d. Neither 1 nor 2
respect to Global Bio-India Summit, 2019
1. It is one of the largest biotechnology
stakeholders conglomerate being held 6) Consider the following statements with
in India for the first time. respect to Shaheen-1
2. It has been organised by the 1. It is a surface-to-surface ballistic missile
Department of Biotechnology (DBT) of test fired by Pakistan.
Ministry of Science and Technology.

www.shankariasacademy.com | www.iasparliament.com
74

2. It is capable of delivering all types of d. Neither 1 nor 2


warheads.
Which of the statement(s) given above
9) MADAD portal is an online grievances
is/are not correct?
monitoring system for which of the following?
a. 1 only
a. Railways
b. 2 only
b. Consular Services
c. Both 1 and 2
c. Coal Quality Monitoring
d. Neither 1 nor 2
d. Transparency in Education system

7) Consider the following statements with


10) Consider the following statements with
respect to POSHAN Abhiyaan
respect to Bacteriophages
1. It is a Ministry of Women and Child
1. These are viruses that can kill the
Development’s flagship programme to
bacteria.
improve nutritional outcomes for
children, pregnant women and lactating 2. These are found naturally where
mothers. bacteria are present.
2. NITI Aayog closely monitors and Which of the statement(s) given above is/are
undertakes periodic evaluations of correct?
POSHAN Abhiyaan.
a. 1 only
3. A special target for stunting is set at 25%
by 2022. b. 2 only

Which of the statement(s) given above is/are c. Both 1 and 2


correct? d. Neither 1 nor 2
a. 1 and 2 only
b. 2 and 3 only Answers
c. 1 and 3 only 1. d
d. 1, 2 and 3 By Deprivation: It is a compulsory termination of
Indian citizenship by the Central government, if:
8) Consider the following statements with 1. the citizen has obtained the citizenship by
respect to FASTag fraud
1. It is a reloadable tag that be read from 2. the citizen has shown disloyalty to the
up to several feet away and need not to Constitution of India
be within direct line-of-sight of the 3. the citizen has unlawfully traded or
reader to be tracked. communicated with the enemy during a war
2. Person reside within a range of 10 km 4. the citizen has, within five years after
from the toll plaza can avail a registration or naturalisation, been imprisoned
concession on toll to be paid via FASTag. in any country for two years and
Which of the statement(s) given above is/are 5. the citizen has been ordinarily resident out of
correct? India for seven years continuously.
a. 1 only  The Ministry of Home Affairs (MHA) has
b. 2 only recently cancelled the citizenship of TRS MLA
from Vemulawada in north Telangana
c. Both 1 and 2 Chennamaneni Ramesh.

www.shankariasacademy.com | www.iasparliament.com
75

 The MHA has cancelled his citizenship on the  It has brought academia, innovators,
ground of misrepresentation of facts at the researchers, start-ups, medium and large
time of applying for citizenship in 2008. companies together on the same platform.
2. d  Biotechnology is recognized as the sunrise
sector- a key driver for contributing to India’s
 Tuvalu is a group of nine tiny islands in
USD 5 Trillion economy target by 2025.
the South Pacific which won independence
from the United Kingdom in 1978.  The three-day event has been organized by
the Department of Biotechnology
 Five of the islands are coral atolls, the other
(DBT) of Ministry of Science &
four consist of land rising from the sea bed.
Technology along with its Public Sector
 Formerly known as the Ellice Islands, all Undertaking, Biotechnology Industry
are low-lying, with no point on Tuvalu being Research Assistance Council (BIRAC).
higher than 4.5m above sea level. 4. b
 Life on the islands is simple and often  Recently, Union cabinet approves
harsh. There are no streams or rivers, so establishment of National Institute of
the collection of rain is essential. Sowa Rigpa (NISR) in Leh.
 It has recently rejected offers from Chinese  It will be an autonomous national
firms to build artificial islands that would institute under the Ministry of
help it deal with rising sea levels. AYUSH with the mandate of undertaking
 Tuvalu saw the offers as an attempt to reduce interdisciplinary education and research
Taiwan's influence in the region. programmes.

 But it instead reaffirmed its support for  “Sowa-Rigpa” (Science of


Taiwan. healing) commonly known as “Tibetan
system of medicine” is one of the oldest,
living and well documented medical tradition
of the world.
 It has been originated from Tibet and
popularly practice in India, Nepal, Bhutan,
Mongolia and Russia.
 The majority of theory and practice of
Sowa-Rigpa is similar to “Ayurveda”.
 rGyud-bZhi (four
tantra) the fundamental text book of this
medicine was composed by Yuthog Yonten
Gonpo who is believed to be the father of
Sowa Rigpa.
5. a
 New Space India Limited (NSIL), is a
wholly owned Government of India
undertaking (CPSE), under the administrative
control of Department of Space (DOS).
 It is to commercially exploit the research and
3. c development work of Indian Space Research
Organisation (ISRO).
 Global Bio-India is one of the largest
biotechnology stakeholders conglomerate  It is the second commercial arm of
being held in India for the first time. ISRO after Antrix.

www.shankariasacademy.com | www.iasparliament.com
76

 Antrix Corporation looks at foreign  It can be read from up to several feet


markets whereas NSIL will focus mostly on away and does not need to be within
domestic industries for commercialisation direct line-of-sight of the reader to be
activities. tracked.
6. d  The tag, which is valid for five years, comes in
seven different colours, violet, orange, yellow,
 Pakistan recently test-fired a nuclear-
green, pink, blue, black.
capable surface-to-surface ballistic
missile ‘Shaheen-1’.  Each colour is assigned to a particular category
of vehicles.
 It can strike targets up to 650 kilometres,
bringing many Indian cities under its range.  People residing within a range of 10 km
from the toll plaza, can avail a
 The missile is capable of delivering all
concession on toll to be paid via FASTag.
types of warheads (conventional as well as
nuclear).  In such a case, they need to submit the
required documents, proof of residence at the
7. d
bank, nearest POS location to validate the
 The Prime Minister’s Overarching Scheme for residential address is within 10 km of a
Holistic Nutrition or POSHAN Abhiyaan, is particular toll plaza.
a Ministry of Women and Child Development’s
 Once the address is verified, they can avail a
flagship programme to improve nutritional
concession on the toll paid via FASTag
outcomes for children, pregnant women
assigned to the particular vehicle.
and lactating mothers.
9. b
 NITI Aayog has played a critical role in
shaping the POSHAN Abhiyaan.  The MADAD Portal is an online grievances
monitoring system launched by the Ministry of
 NITI Aayog has been entrusted with the
External Affairs (MEA) on February 21, 2015.
task of closely monitoring the POSHAN
Abhiyaan and undertaking periodic  The e-portal provides Indian citizens living
evaluations. abroad an online platform in which they can
file consular grievances to the Indian
 NITI Aayog is required to submit
government.
implementation status reports of POSHAN
Abhiyaan every six months to the PMO.  Rail MADAD (Mobile Application
for Desired Assistance During travel) is an app
 The task of implementation of POSHAN
launched by Ministry of Railways to expedite
Abhiyaan is to be carried out through the
and streamline passenger grievance redressal.
‘Technical Support Unit’ (TSU) established at
NITI Aayog. 10. c
8. c  Bacteriophages are viruses that can kill the
bacteria.
 Ministry of Road Transport and Highways has
announced that FASTags will
become mandatory for all vehicles, private
and commercial, from December 1, 2019. 23-11-2019
 A FASTag is a reloadable tag that 1) Consider the following statements with
automatically deducts toll charges and respect to Cartosat-3
allows a vehicle to pass through a toll gate 1. It is a third-generation advanced Space-
without stopping for the payment. observation satellite with high-
resolution imaging capability.
 It uses radio frequency identification (RFID)
technology to make cashless payments through 2. It is the first dedicated Indian satellite
a prepaid account linked to it. aimed at studying celestial sources in X-

www.shankariasacademy.com | www.iasparliament.com
77

ray, optical and UV spectral bands 5) Consider the following statements with
simultaneously. respect to Jan Jagrukta Abhiyaan
3. It will be launched using PSLV C-47 1. It is to sensitize and mobilise the
rocket. community on measures for prevention
and control of Vector Borne Diseases
Which of the statement(s) given above is/are
(VBDs).
correct?
2. It is an initiative of NITI Aayog along
a. 3 only
with WHO.
b. 1 and 2 only
Which of the statement(s) given above is/are
c. 1 and 3 only correct?
d. 1, 2 and 3 a. 1 only
b. 2 only
2) The Jing Kieng Jri or Living Root c. Both 1 and 2
Bridges are aerial bridges built by weaving and
d. Neither 1 nor 2
manipulating the roots of the Indian rubber
tree. These are found in?
a. Assam 6) Consider the following statements with
respect to One Stop Centre (OSC) Scheme
b. Nagaland
1. It supportswomen above 18 years of age
c. Meghalaya
affected by violence, irrespective of
d. Karnataka caste, class, religion, region, sexual
orientation or marital status.
2. The Central Government will provide
3) Budapest Convention sometimes seen in the 100% financial assistance to the State
news recently is associated with? Government/UT Administrations under
a. Cyber Crime the Scheme.
b. Protection of Ozone Layer Which of the statement(s) given above is/are
correct?
c. Ballast Water Management
a. 1 only
d. Civil Liability for oil pollution and damage
b. 2 only
c. Both 1 and 2
4) Consider the following statements with
respect to Botulinum d. Neither 1 nor 2
1. It is a natural toxin produced by
bacteria known as Clostridium botulin. 7) Consider the following statements with
2. It is found only in soil and can also be respect to Central Asian Flyway (CAF)
artificially prepared. 1. It is a traditional migration route across
Which of the statement(s) given above is/are the Russia, Kazakhstan, Uzbekistan,
correct? Turkmenistan, Afghanistan, and ends
in Pakistan.
a. 1 only
2. Bonn Convention provides a global
b. 2 only platform for the conservation and
c. Both 1 and 2 sustainable use of migratory animals
and their habitats.
d. Neither 1 nor 2

www.shankariasacademy.com | www.iasparliament.com
78

Which of the statement(s) given above is/are a. 1 only


correct?
b. 2 only
a. 1 only
c. Both 1 and 2
b. 2 only
d. Neither 1 nor 2
c. Both 1 and 2
d. Neither 1 nor 2
Answers
1. a
8) Ganga Sagar and Mousuni Islandswhich was
in news recently are a part of?
a. The continental shelf of Bay of Bengal, West
Bengal
b. St. Mary's Island, Karnataka
c. Willingdon Island, Kochi
d. Diu Island

9) Consider the following statements with


respect to National Credit Guarantee Trustee
Company
1. It is a common trustee company to
manage and operate various credit
guarantee trust funds.
2. It has been set up by the Department of
Financial Services, Ministry of Finance.
Which of the statement(s) given above is/are
correct?

a. 1 only
b. 2 only
c. Both 1 and 2
d. Neither 1 nor 2
 ISRO will launch Cartosat-3, and 13 other
foreign satellites, aboard a PSLV (Polar
10) Consider the following statements with Satellite Launch Vehicle) C-47.
respect to Anti-defection law
 The main purpose of the mission is to place the
1. It made legislators liable to be penalised 1,625-kg Cartosat-3 satellite, the third
for their conduct both inside and generation of Earth-observation remote
outside the legislature. sensing satellites that ISRO has been
2. It made the Presiding Officer of the launching and using 1988.
legislature the sole arbiter of defection
 These satellites provide high-resolution
proceedings. imagery of the Earth that are used for
Which of the statement(s) given above is/are applications like 3-D mapping, disaster
correct? management, agriculture and water

www.shankariasacademy.com | www.iasparliament.com
79

management, recording of changes in land use, 4. a


and urban and rural infrastructure planning,
Avian Botulism
and even border surveillance.
 Over 17,000 birds were found dead in the past
 The PSLV-C47 rocket is also carrying 13
few days in the Sambhar Lake in Rajasthan.
commercial nano-satellites from the United
States (U.S).  It is not the first time that where deaths due to
2. c botulism have been recorded. It is reported in
Lake Michigan in 2007 and 2008 and in
 The Jing Kieng Jri or Living Root Bridges — Hawaii in 2008.
aerial bridges built by weaving and
manipulating the roots of the Indian rubber  The Indian Veterinary Research Institute
tree — have been serving as connectors for (IVRI), Bareilly, states that climatic conditions
generations in Meghalaya. were ultimately responsible for having
triggered the mass die-off.
 Spanning between 15 and 250 feet and built
over centuries, the bridges, primarily a means  It is occurred due to avian botulism, a
to cross streams and rivers, have also become disease caused by bacteria that affects nerves.
world-famous tourist attractions.  Botulinum is a natural toxin produced by
 Now, new research investigates these bacteria known as Clostridium botulin.
structures and proposes to integrate them in  It produces the toxin when it starts
modern architecture around the world, and reproducing and attack the neurons which
potentially help make cities more leads to muscle paralysis.
environment-friendly.
 It is commonly found in the soil, river, and
3. a sea water.
 It is the sole legally binding multilateral treaty  There are around eight types — A, B, C1, C2, D,
that coordinates cybercrime investigations E, F, and G — of botulinum toxin and they are
between nation-states and criminalises certain distinguishable when diagnosed.
cybercrime conduct.
 Botulinum affects both humans and animals,
 The convention is led by Council of Europe, but the type of the toxin varies — botulinum C
which is distinct from European Union. in birds and A, B and E in humans.
 Both members and non-members of Council of 5. a
Europe are parties to the convention.
 Jan JagruktaAbhiyaan(Peoples
 Totally 64 nations are party to this convention Movement)is a special campaign in Delhi to
and 3 countries have signed but yet to ratify sensitize and mobilise the community on
the convention. measures for prevention and control of
 India, Russia and Chinaare not members to Vector Borne Diseases (VBDs) like
this convention. Malaria, Dengue and Chikungunya.

 The 2nd additional protocol to the convention  It is an initiative of Ministry of Health and
is under preparation and the member Family Welfare.
countries recently met at Strasbourg. 6. b
 Russia recently passed a resolution in United  The One Stop Centre (OSC)
Nation General Assembly (UNGA), to create a Scheme provides a range of services including
new treaty that can coordinate and share data police facilitation, medical aid, psycho-social
to prevent cybercrime. counselling, legal counselling and temporary
shelter to women affected by violence in an
 India voted in favour of Russia’s resolution.
integrated manner under one roof.
 It supports all women including girls
below 18 years of age affected by violence,

www.shankariasacademy.com | www.iasparliament.com
80

irrespective of caste, class, religion, region,  Studies over the decades have recorded that
sexual orientation or marital status, both in the sea level in the area rose from 2.26 mm
private and public spaces. a year in 2002 to 12 mm in 2014 and by
another 2.9 mm in 2019.
 The Centres will be integrated with a Women
Helpline to facilitate access.  Ganga Sagar and Mousuni Islands are in
the Ganges delta, lying on thecontinental
 The Scheme will be funded through
shelf of Bay of Bengalto the south of
NirbhayaFund .
Kolkata, West Bengal.
 The Central Government will provide
 Ganga Sagaris the largest island in the
100% financial assistance to the State
Sundarbans archipelago.
Government /UT Administrations under
the Scheme.  It is one of the most famous Hindu pilgrimage
centres in India.
7. b
 Every year on MakarSankranti pilgrims from
 A joint study by Russian and Indian scientists
all over India, gather at Gangasagar for a holy
shows that Migratory birds from Siberia and
dip at the confluence of the river Ganga and
northern Eurasia could be falling prey to
the Bay of Bengal.
power lines and turbines in India.
 Gangasagar Fair is the second biggest
 The Central Asian Flyway (CAF) covers a
congregation of Hindu pilgrims after the
large continental area of Eurasia between the
KumbhaMela.
Arctic and Indian Oceans and the associated
island chains.  Mousuni, which lies a few km south of Ganga
Sagar, is similarly exposed to the sea.
 It traditionally migrated 5,500 km south
across the Russian Federation, Kazakhstan,
Uzbekistan, Turkmenistan, Afghanistan, and
Pakistan, before arriving at the wintering
grounds in Keoladeo Ghana National
Park in Rajasthan.
 Even Indian government has launched a
national action plan conservation of migratory
birds along the Central Asian Flyway.
 India provides a critical stopover as well as a
wintering habitat to more than 370 species of
migratory birds.
 CMS or Bonn Convention(Convention on
the Conservation of Migratory Species of Wild
Animals) provides a global platform for the
conservation and sustainable use of migratory
animals and their habitats.
 CMS brings together the States through which
migratory animals pass, the Range States, and
lays the legal foundation for internationally
coordinated conservation measures
throughout a migratory range.
8. a
 The island at the confluence of the
Ganga and the Bay of Bengal is losing the
battle to sea level rise, coastal erosion, tidal
surges and violent cyclones.

www.shankariasacademy.com | www.iasparliament.com
81

9. c 1. This allows the two countriesto directly


share information on North Korea’s
Credit Guarantee Fund
nuclear and missile activities.
 National Credit Guarantee Trustee Company 2. South Korea decided to quit GSOMIA
Ltd is a common trustee company to manage pact with Japan recently.
and operate various credit guarantee trust
funds. Which of the statement(s) given above is/are
correct?
 It has been set up by the Department of
Financial Services, Ministry of Finance. a. 1 only

 It operates Fund of Funds for Start-ups (FFS) b. 2 only


under Start up India Programme. c. Both 1 & 2
 Under Startup India initiative, all applications d. Neither 1 nor 2
can be submitted to the Department for
Promotion of Industry and Internal Trade.
 Collateral-free loans have been made available 3) Consider the following statements with
to MSMEs under the Credit Guarantee Scheme respect toSumatran Rhinos
in Aspirational Districts. 1. They are the smallest of the living
10. c rhinoceroses and the only Asian rhino
with two horns.
 It made legislators liable to be penalised for
their conduct both inside and outside the 2. They arelisted asCritically
legislature. Endangeredin the IUCN Red List.
3. The New Delhi Declarationwas signed to
 It made the Presiding Officer of the legislature
secure the future of Asian rhinos.
the sole arbiter of defection proceedings.
Which of the statement(s) given above is/are
correct?
25-11-2019 a. 1 and 2 only
1) Article 74 of the constitution of India b. 2 and 3 only
provides that “there shall be a Council of
Ministers with the Prime Minister as its head to c. 1 and 3 only
advice the President”. Which rule of the d. 1,2 and 3 only
Government of India (Transaction of Business)
Rules 1961, states that “the Prime Minister
may, permit or condone a departure from 4) Consider the following with respect
these rules, to the extent he deems necessary”? to MahamastakabhishekaMahotsav
a. Rule 2 1. The festival honors Lord Bahubali, a
b. Rule 12 revered Buddhist monk.
c. Rule 8 2. Gomateshwara statue is one of the
largest free-standing statues in the
d. Rule 10 world made up of Grey stone.
Which of the statement(s) given above is/are
2) Consider the following statements with correct?
respect to General Security of Military a. 1 only
Information Agreement (GSOMIA) between
Japan and South Korea b. 2 only
c. Both 1 and 2
d. Neither 1 nor 2

www.shankariasacademy.com | www.iasparliament.com
82

c. 2, 3 and 4 only
5) Consider the following statements with d. 1, 2, 3 and 4
respect to Golden Rice
1. It is a Biofortifiedrice variety.
8) Consider the following statements with
2. It contains beta-carotene which gives respect to Consumer Expenditure Survey
the golden color. (CES)
3. It is intended to fight against Vitamin A 1. It is traditionally a quinquennial
deficiency. (recurring every five years) survey.
Which of the statement(s) given above is/are 2. It will be conducted by National Sample
correct? Survey Office (NSSO).
a. 1 and 2 only Which of the statement(s) given above is/are
correct?
b. 1 and 3 only
a. 1 only
c. 1,2 and 3 only
b. 2 only
d. None of the above
c. Both 1 and 2
d. Neither 1 nor 2
6) Which of the following best describes the
term “Sea Ranching” sometimes seen in the
news recently?
9) Consider the following statements with
a. Cultivation of marine organisms for food and respect to Exchange-Traded Fund (ETF)
other products in the open ocean
1. It is a fund that are publicly listed and
b. Fishes are reared in a controlled laboratory traded on a stock exchange.
environment and let into the sea
2. The price of the ETF is based on the net
c. Mineral retrieval process that takes place on the asset value of the underlying stocks.
ocean floor
3. It is available only for stocks and are not
d. Submersion of oil drilling equipments fully in available for currencies and
deep ocean waters commodities.
Which of the statement(s) given above is/are
correct?
7) Consider the following statements with
respect to Cochin Forest Cane Turtle a. 1 only
1. It is endemic to the Western Ghats in b. 1 and 2 only
the states of Kerala, Karnataka, and
c. 2 and 3 only
Tamil Nadu.
d. 1, 2 and 3
2. It is extremely rare and cannot be easily
spotted.
3. It is herbivorous, with the diet including 10) Consider the following statements with
mostly Sugar Canes. respect to Industrial Security Annex
(ISA) Pact
4. It is listed as critically endangered
under the IUCN Red List. 1. It is a military pact recently signed
between India and U.S.A.
Which of the statements given above are
correct? 2. It is a part of the Communications,
Compatibility and Security Agreement
a. 1 and 2 only
(COMCASA).
b. 1, 2 and 4 only

www.shankariasacademy.com | www.iasparliament.com
83

Which of the statement(s) given above is/are  South Korea, decided to suspend its
correct? plans to quit an intelligence sharing pact
with Japan amid pressure from the US.
a. 1 only
b. 2 only  Significantly, both South Korea and Japan
are US allies.
c. Both 1 and 2
 But in recent years, the relationship
d. Neither 1 nor 2 between both countries has
deteriorated, given the territorial dispute
over the Dokdo islandsknown as
Answers Takeshima in Japan.
1. b  While South Korea controls them, the islands
are claimed by Japan.
 The Centre invoked Rule 12 of the
Transaction of Business Rules, 1961 to  Furthermore, the two countries have differing
revoke President’s Rule in Maharashtra. views on Imperial Japan’s treatment of
Koreans, especially the forced labourers and
 It empowered it to revokethe “comfort women” or “sex slaves”.
proclamation of President’s rule without
the prior approval of the Union Cabinet.  In July, Japan imposed export controls on
three chemicals that South Korea uses in its
 Titled “Departure from Rules”, Rule important semiconductor industry.
12 says, “The Prime Minister may, in case or
classes of cases permit or condone a departure  In August, Japan decided to remove South
from these rules, to the extent he deems Korea from its “white list”, a fast track trade
necessary.” list of trusted partners.
 The Cabinet can subsequently give post-facto  This is considered to be retaliation for South
approval for any decision taken under Rule 12. Korea’s decision to leave the intelligence pact,
a decision that was made in August.
 The last big decision taken through the
invocation of Rule 12 was in, 3. d
o re-organisation of the state of Jammu  The Sumatran rhino is now officially
and Kashmirinto the Union extinct in Malaysia, with the death of the
Territories of Jammu and Kashmir, and last known specimen.
Ladakh on October 31.
 The Sumatran rhino once roamed across Asia,
 Rule 12 is usually not used to arrive at major but has now almost disappeared from the wild,
decisions by the government. with fewer than 100 animals believed to exist.
 However, it has been used in matters such as  The species is listed as Critically
withdrawal of an office memorandum or Endangered in IUCN Red List.
signing of MoUs in the past.
 They are the smallest of the living
2. a rhinoceroses and the onlyAsian rhino
with two horns.
 The General Security of Military
Information Agreement, commonly  India signed ‘The New Delhi Declaration
referred to as GSOMIA, allowed Japan and on Asian Rhinos 2019’ with Bhutan,
South Korea to directly share information on Nepal, Indonesia and Malaysia to increase
North Korea’s nuclear and missile activities. the population of three species of Asian
rhinos.
 The agreement is the first intelligence-sharing
agreement between the two nations since  The Asian species include the greater one-
Korea’s liberation from Imperial Japan in horned rhino, the Javan rhino and the
1945. Sumatran rhino.

www.shankariasacademy.com | www.iasparliament.com
84

4. d  In an effort to replenish natural fish stock in


the sea off the coast of Tamil Nadu, the
 It is a ritual celebrated once in 12 years
Fisheries Department has planned to ranch
for Lord Bahubali in Shravanabelagola,
seeds of black tiger shrimps, Indian prawn
Karnataka.
and cobia at a cost of Rs. 10.5 crore.
 The festival 7. a
honorsLordGommateshwara, also known
asLordBahubali, a revered figure among  The Cochin Forest Cane Turtle,
the Jains. Vijayachelys silvatica (Family Geoemydidae),
is a cryptic species in a monotypic
 Lord Bahubali is the son of the first Jain genus endemic to the semi-evergreen and
Tirthankara. evergreen forests of the Western Ghats in
 The statue was carved under the order of south-western India.
Chamundaraya, who was the Ganga  It is the smallest turtle species in India.
Dynasty's minister and commander in 981
AD.  Vijayachelys silvatica is omnivorous, with the
diet including insects, molluscs, earthworms,
 It is one of the largest free-standing statues in millipedes, and fruits.
the world made up ofGranite.
 The turtle that was discovered in 1912 and was
5. c
again spotted only in 1982 is extremely rare
 Bangladesh is on the verge of becoming and cannot be easily spotted.
the first countryto approve
 The 10-cm-long turtle blended in colour and
plantation of Golden Rice.
shape with the dry leaves that clothed the
 It is said to be answer to Vitamin A forest floor.
deficiency, which in Bangladesh, over 21% of
the children are suffering.
 Golden rice is a Genetically modified,
Biofortified crop. Biofortification increases
the nutritional value in crops.
 Rice is naturally low in the pigment beta-
carotene.
 It contains ‘beta-carotene’ (provitamin A),
which is converted into vitamin A as
needed by the body and gives the grain its
goldencolor.
 It is developed by the Philippines-
based ‘International Rice Research
Institute’ (IRRI).
6. b
 Sea Ranching or Ocean Ranching is an  The cane turtle is an anomaly. It scuttles
internationally accepted concept, where fish through the forest, but it is not a tortoise.
that are commercially important and need
replenishment are reared in a controlled  Although the shape of its legs and webbed feet
laboratory environment and let into the sea are reminiscent of its aquatic relatives, it
when they are of a certain size. doesn’t like water and stays clear of
streams.
 The expectation is that they would develop so
that local fishers can catch them.  It is listed as “Endangered” in the IUCN Red
List.

www.shankariasacademy.com | www.iasparliament.com
85

8. c fund management fee of an ETF is much lower


than that of a normal mutual fund scheme.
 The CES is traditionally
a quinquennial (recurring every five  So, investors save on that aspect as well while
years) survey conducted by the government’s getting the benefit of a professional fund
National Sample Survey Office (NSSO) that is manager.
designed to collect information on the
What is the difference between an ETF and an
consumption spending patterns of households
Index Fund?
across the country, both urban and rural.
 While an index fund is also designed to mirror
 The data gathered in this exercise reveals the
a particular index, there are some differences
average expenditure on goods (food and non-
between an ETF and an index fund. First, an
food) and services and helps generate
index fund is just like any other mutual fund
estimates of household Monthly Per Capita
wherein the net asset value (NAV) of the fund
Consumer Expenditure (MPCE) as well as the
is based on the closing price of the underlying
distribution of households and persons over
securities.
the MPCE classes.
 In the case of an ETF, however, the NAV is
 The estimates of monthly per capita
continuously linked to the current market
consumption spending are vital in gauging the
price of the underlying stocks, which also
demand dynamics of the economy as well as
makes it possible to buy or sell the ETF unit
for understanding the shifting priorities in
throughout the day just like an ordinary stock.
terms of baskets of goods and services, and in
But, since an ETF is traded like a stock, one
assessing living standards and growth trends
needs a demat account to buy or sell an ETF
across multiple strata.
whereas an index fund can be bought directly
9. b from an asset management company without
having a demat account.
Exchange-Traded Fund (ETF)
Are ETFs available only for stocks?
 An ETF is a fund that comprises a group of
stocks that are listed on an exchange and can  No. An ETF can be created for any kind of
be simply traded like any other listed security. asset or security that has an index or a liquid
market for the underlying securities of the
 Typically, an ETF mirrors a particular index, ETF.
which means the group of stocks in the ETF
would be similar to those in the index that it is  Globally, there are many ETFs on bonds,
benchmarked to. So, for instance, an ETF currencies and commodities as well.
mirroring the Sensex would have the same 30
stocks that the Sensex has.  In India, gold ETFs are quite popular and have
physical gold as the underlying security.
 In other words, if one buys an ETF mirroring
10. d
the Nifty, he’s indirectly buying all the stocks
that are there in the Nifty without going  The Industrial Security Annex (ISA) is a
through the trouble of buying each stock part of the General Security of Military
individually. Information Agreement (GSOMIA),
What are the benefits of investing through which India had signed with the U.S. many
ETFs? years ago.

 The biggest benefit is liquidity. Since an ETF is  The ISA is crucial for U.S. companies bidding
publicly listed and traded on a stock exchange, for big-ticket Indian deals to partner Indian
liquidity is not an issue. private companies.

 An investor can know the price of each unit of  The next 2+2 dialogue between India and the
the ETF and take an informed decision. U.S. is likely to be held on December 18 in
Washington, D.C., during which the two
 The price of the ETF is based on the net asset countries are expected to sign
value of the underlying stocks. Second, the the Industrial Security Annex (ISA) and

www.shankariasacademy.com | www.iasparliament.com
86

review the steps being taken to operationalise b. Only the owner will be allowed to drive the
the Communications Compatibility and vehicle and no others are permitted to drive that
Security Agreement (COMCASA). vehicle
 But the Basic Exchange and Cooperation c. The insurance firm will pay the claim amount in
Agreement for Geo-spatial Cooperation full only if the named driver was at the wheel
(BECA), which is under discussion, is unlikely
d. None of the above
to be concluded.

4) Consider the following statements with


26-11-2019 respect to Sabka Vishwas Scheme
1) Consider the following statements with 1. The aim of the scheme is to help
respect to Integrated Development of Wildlife taxpayers in clearing the disputes under
Habitats (IDWH) Scheme legacy service and central excise tax
1. It is a centrally sponsored scheme. which are subsumed in Goods and
Service Tax.
2. Under the scheme, financial assistance
is provided to State/UTs for protection 2. The main components of the Scheme
and conservation of wildlife and its are dispute resolution and amnesty.
habitats in Protected Areas as well as 3. It was launched by the Ministry of
outside Protected Areas. Finance.
3. Currently, 21 critically endangered Which of the statement(s) given above is/are
species have been identified under this correct?
programme.
a. 2 only
Which of the statement(s) given above is/are
correct? b. 1 and 2 only
a. 3 only c. 1 and 3 only

b. 1 and 3 only d. 1, 2 and 3


c. 2 and 3 only
d. 1, 2 and 3 5) Consider the following statements with
respect to Atal Beemit Vyakti Kalyan Yojana
1. It is a welfare measure being
2) “Gamosa” is a traditional textile which implemented by the Ministry of
recently gets Geographical Identity (GI) tag is Finance.
famous in which of the following states?
2. It offers cash compensation to insured
a. Assam persons when they are rendered
b. Meghalaya unemployed.

c. Tamil Nadu Which of the statement(s) given above


is/are incorrect?
d. Arunachal Pradesh
a. 1 only
b. 2 only
3) Which one of the following best describes
the term “Named Driver Policy” sometimes c. Both 1 and 2
seen in the news recently? d. Neither 1 nor 2
a. Four wheeler licenses will be issued to a person
only after he/she is certified as a trained two
wheeler driver

www.shankariasacademy.com | www.iasparliament.com
87

6) Consider the following statements with 2. The tunnel provides a winter link to
respect to Archaeological Survey of India residents of Lahaul and Spiti.
(ASI)
3. National Highways Authority of India
1. It functions as an attached office of the (NHAI) along with Border Roads
Department of Culture. Organisation (BRO) is working on this
project.
2. It takes up the conservation project only
in India. Which of the statement(s) given above is/are
correct?
Which of the statement(s) given above is/are
correct? a. 1 only
a. 1 only b. 2 only
b. 2 only c. 3 only
c. Both 1 and 2 d. None
d. Neither 1 nor 2
10) Consider the following statements with
respect to X-59 QueSST
7) Consider the following statements with
respect to Bodos 1. It is a new supersonic demonstration
piloted aircraft to fly faster than sound
1. They are the single largest tribal
without producing the annoying, sonic
community in Tripura.
booms.
2. They are recognized as a plains tribe in
2. It is being indigenously developed by
the Sixth Schedule of the Indian
NASA.
Constitution.
Which of the statement(s) given above is/are
Which of the statement(s) given above is/are
correct?
correct?
a. 1 only
a. 1 only
b. 2 only
b. 2 only
c. Both 1 and 2
c. Both 1 and 2
d. Neither 1 nor 2
d. Neither 1 nor 2

8) Sangai Festival which was in news recently Answers


is celebrated in 1. d
a. Mizoram  The Government of India provides financial
b. Meghalaya and technical assistance to the State/UT
Governments for activities aimed at wildlife
c. Assam conservation through the Centrally Sponsored
d. Manipur Scheme viz. ‘Integrated Development of
Wildlife Habitats’.
The scheme has following three components:
9) Consider the following statements with
respect to Rohtang Tunnel 1. Support to Protected Areas (National Parks,
Wildlife Sanctuaries, Conservation Reserves
1. It will cut through the mighty Pir Panjal and Community Reserves)
range of Himalayas located in the
Ladakh Union Territory. 2. Protection of Wildlife outside Protected Areas
and mitigation of Human Wildlife conflict.

www.shankariasacademy.com | www.iasparliament.com
88

3. Recovery programmes for saving critically  Gamosa is one of the unique identities of
endangered species and habitats. Assamese society. It is a symbol of Assamese
culture.
Scope
 This is a handloom product which has high
 Under IDWH, the financial assistance is
esteem and wide usage in Assamese culture.
provided to State/UT Governments for
protection and conservation of wildlife and its  In earlier times, Gamosa was also known
habitats in Protected Areas (PAs) as well as “Phali”.
as outside PAs and also for the recovery
programmes of the critically  Furthermore, as it was used to wipe face that is
endangered species. why it was also known as “Mukhosa”.

 The implementation of the schemes would be  The Chokuwa rice of Assam has been
done through the respective States in described as a “unique gift of nature”.
designated Tiger Reserves, Protected Areas  This class of rice is not known in any other
and Elephant Reserves. parts of the world.
 Presently, 21 critically endangered  Chokuwa rice is a special class of semi
species have been identified under this glutinous winter rice (Sali rice in vernacular)
programme. of Assam which is in cultivation from time
2. a immemorial.

 Assam’s traditionally woven “Gamosa” with  This group of rice is characterized by low
its distinctive red border and floral motifs, and amylase content (12-17%) of the grains.
a semi-glutinous winter rice
 Chokuwa rice varieties are tall, photo-sensitive
variety “Chokuwa” have earned the coveted
and long duration varieties with low yield
geographical indications (GI) recognition as
potential.
proof of their unique geographical origins,
thereby getting legal protection to prevent  However, this class of semi-glutinous rice
their unauthorized use. differs from other Sali rice varieties in its plant
and grain characteristics.
 Recently, the
multipurpose Assamese gamosa, a
ubiquitous, white cotton towel, has also been
assigned a new function — conservation of rare
freshwater turtles.
 Few cultural symbols are as utilitarian as the
white handmade cotton gamosa, with its
characteristic red border of woven motifs.
 It is valued as a gift for visitors, used as a scarf,
anti-dust mask, wrapped around the head as a
 The Muga silk was the state’s first product to turban.
get the tag in 2007.
 Conservationists are now banking on this
 In 2015, the pungent Karbi Anglong ginger and cultural icon to carry forward the message of
the juicy Tezpur litchi got the recognition. turtle conservation, with gamosas woven with
turtle images.
 The GI Registry journal described the Gamosa
as a handloom product, which is one of the 3. c
unique identities of Assamese society.
 IRDAI panel has recently suggested the
 Gamosa is a traditional textile of Assam. It Named Driver Policy and use of
bears great significance for the people of telematics in insurance sector.
Assam.

www.shankariasacademy.com | www.iasparliament.com
89

 A Named Driver Policy is a type of  In all these cases, there would be no other
insurance policy that, in short, provides liability of interest, fine or penalty. There is
coverage for only the drivers listed on the also a complete amnesty from prosecution.
policy.
5. a
 (i.e) If you have a named driver insurance
 Atal Bimit Vyakti Kalyan Yojana is a
policy, the only drivers covered the event of a
welfare measure being implemented by
crash are those specifically listed on the policy.
the Employee's State Insurance (ESI)
 If you allow your friend — unlisted on the Corporation, Ministry of Labour.
named driver policy — borrow your vehicle
 It offers cash compensation to insured persons
and they get into an accident, you would have
when they are rendered unemployed.
no coverage.
 It will be applicable for employees covered
 Use of telematics or tracking devices will
under the Employees’ State Insurance
monitor the driving habits such as
(ESI) Act, 1948.
acceleration, and braking and will provide
feedback to the driver. Employee’s State Insurance (ESI) Corporation
 Auto owners only pay the premium aligned to  It is self-financing social security and
their driving profile and thus avoid paying for health insurance scheme for Indian
coverage based on one-size-fits-all system. workers.
 Telematics will eventually pave the way for a  It is autonomous corporation by
‘Pay As You Drive’ and ‘Pay How You Drive’ statutory creation under Ministry of
model. Labour and Employment, Government of
India.
4. d
 It is managed by Employees’ State
 The aim of Sabka Vishwas (Legacy Dispute
Insurance Corporation (ESIC) according
Resolution) Scheme, 2019 is to help taxpayers,
to rules and regulations stipulated there in the
including small taxpayers, in clearing the
ESI Act 1948.
baggage of disputes under legacy taxes (Service
Tax and Central Excise), which are subsumed 6. a
in Goods and Service Tax.
 Recently, Ministry of Culture told in Lok
 It was launched by the Ministry of Finance. Sabha that a total 651 Hindu temples in India
are designated as Centrally Protected
 The main components of the Scheme are
Monuments under the Archaeological
dispute resolution and amnesty.
Survey of India (ASI).
 The dispute resolution component is
 Archaeological and historical pursuits in India
aimed at liquidating the legacy cases of Central
started with the efforts of Sir William Jones,
Excise and Service Tax that are subsumed in
who put together a group of antiquarians to
GST and are pending in litigation at various
form the Asiatic Society in 1784 in
forums.
Calcuta.
 The amnesty component of the Scheme
 ASI is the successor of The Asiatic Society
offers an opportunity to the taxpayers to pay
of India. It was established in 1861.
the outstanding tax and be free of any other
consequence under the law.  It functions as an attached office of the
Department of Culture.
 The most attractive aspect of the Scheme is
that it provides substantial relief in the tax  The major activities of the Archaeological
dues for all categories of cases as well as full Survey of India are:
waiver of interest, fine and penalty.
1. Survey of archaeological remains and
excavations

www.shankariasacademy.com | www.iasparliament.com
90

2. Maintenance and conservation of centrally started agitating for the creation of


protected monuments sites and remains the Bodoland state.
3. Chemical preservation of monuments and 8. d
antiquarian remains
 Government of Manipur, Spearheaded by
4. Architectural survey of monuments Tourism Department, celebrates the
Manipur Sangai Festival.
5. Development of epigraphical research and
numismatic studies  Celebrated each year from 21st to 30th of
6. Setting up and re-organisation of site November, this is one of the most popular
museums festivals of India.

7. Expedition abroad  Theme - To project & showcase the tourism


potential and promote “DESTINATION
8. Training in Archaeology MANIPUR”.
9. Publication of technical report and research  The Festival strives to showcase and promote
works. the best of what the state has to offer to the
 The ASI has taken up the conservation world in terms of art and culture, handloom,
project of Ta Prohm, Cambodia under the handicrafts & fine arts, indigenous sports,
ITEC programme of Ministry of External cuisines & music, eco & adventure sports as
Affairs. well as the scenic natural beauty of the land.

7. b  Moreover, the Festival has assumed great


significance in recent years as it celebrates
 Recently, the central government extended the ethnic diversity of Manipur Society.
ban on the insurgent group National
Democratic Front of Bodoland  Participation of various Indian States, foreign
(NDFB) by 5 more years for its involvement traders and cultural troupes in the Festival
in a series of violent activities including makes it grander in scale and style.
killings and extortion, and for joining hands 9. b
with anti-India forces.
 Recently, Defence Minister informed that
 National Democratic Front of Bodoland efforts would be made to make the Rohtang
(NDFB), an organisation that is known to be Tunnel operational by May 2020.
involved in attacks, killings, and extortions.
 The Rohtang Tunnel, when complete, will cut
 Bodos are the single largest tribal through the mighty Pir Panjal range located
community in Assam, making up over 5-6 in the state of Himachal Pradesh.
per cent of the state’s population.
 It will provide a winter link to residents of
 The four districts in Assam, Kokrajhar, Baksa, Lahaul and Spiti village of Himachal
Udalguri and Chirang, that constitute the Bodo Pradesh.
Territorial Area District (BTAD), are home to
several ethnic groups.  It will reduce the distance between Manali and
tribal district Lahual-Spiti's administrative
 They are recognized as a plains tribe in centre Keylong by around 45 kilometres.
the Sixth Schedule of the Indian
Constitution.  Border Roads Organisation (BRO) have
been working on the project.
 The Bodos felt increasingly alienated with the
unwillingness of both the central and state 10. a
governments in resolving the issue. This in  The X-59 QueSST, (short for Quiet
turn intensified the Bodo movement. SuperSonic Technology), a new supersonic
 No longer did they demand a union territory demonstration piloted aircraft to enable
Instead, the All Bodo Students’ Union (ABSU) the next generation of commercial supersonic
aircraft.

www.shankariasacademy.com | www.iasparliament.com
91

 US-based aerospace firm Lockheed Martin 2) Consider the following statements with
Skunk Works has partnered respect to Transgender Persons (Protection of
with NASA to enable the next generation of Rights) Bill
commercial supersonic aircraft.
1. It defines a transgender person as one
 It is being developed by Lockheed Martin whose gender does not match the
Skunk Works for NASA. gender assigned at birth.

 It has a 4k display instead of a front-facing 2. It provides every transgender person a


window. right to reside and be included in his
household.
3. It seeks to establish a National Council
for Transgender persons (NCT) chaired
by the Union Minister for Social Justice.
Which of the statements given above are
correct?
a. 1 and 2 only
b. 1 and 3 only
c. 2 and 3 only
d. 1, 2 and 3

3) Emissions Gap Report, 2019 was released


recently by which of the following?
a. UN Environment Programme
27-11-2019 b. International Energy Agency
1) Consider the following statements with c. UN Framework Convention on Climate Change
respect to Scheme for setting up of Plastic
d. None of the above
Parks
1. Under the scheme, the government
provides grant funding up to 90 per cent 4) Consider the following statements with
of the project cost. respect to BRUIE Robot sometimes seen in the
news recently
2. Plastic parks are being developed by
State Governments with the support of 1. It is an underwater robot developed for
Ministry of Chemicals & Fertilizers. underwater exploration in extra-
terrestrial and icy waters.
Which of the statement(s) given above is/are
correct? 2. It was developed by National
Aeronautics and Space Administration
a. 1 only
(NASA).
b. 2 only
Which of the statement(s) given above is/are
c. Both 1 and 2 correct?
d. Neither 1 nor 2 a. 1 only
b. 2 only
c. Both 1 and 2
d. Neither 1 nor 2

www.shankariasacademy.com | www.iasparliament.com
92

5) With respect to “ElectroSkin” sometimes Which of the statement(s) given above is/are
seen in the news recently, consider the correct?
following statements:
a. 1 only
1. It is a new fundamental building block
b. 2 only
for a range of soft next-generation
robots. c. Both 1 and 2
2. It is an important step towards soft d. Neither 1 nor 2
robots that can be easily transported,
deployed and even worn.
Which of the statement(s) given above is/are 8) Consider the following statements with
correct? respect to CLSS Awas Portal (CLAP)

a. 1 only 1. It aims to provides a transparent and


robust real-time web-based monitoring
b. 2 only system for credit-linked Subsidy
Services.
c. Both 1 and 2
2. It was launched by Ministry of Finance.
d. Neither 1 nor 2
Which of the statement(s) given above is/are
correct?
6) Consider the following statements with
a. 1 only
respect to Global Exhibition on Services (GES)
b. 2 only
1. It is an attempt towards escalating the
Indian services bar in the global arena c. Both 1 and 2
by exploring 12 Champion Services
Sectors. d. Neither 1 nor 2

2. GES 2019 SEPC is looking to promote


eSports. 9) ‘She Shuttle’ which was in news recently was
3. It is organised by Ministry of Commerce launched in?
and Industry in partnership with a. Delhi
Services Export Promotion Council
(SEPC) and Confederation of Indian b. Mumbai
Industry (CII) only. c. Pune
Which of the statement(s) given above is/are d. Hyderabad
correct?
a. 1 and 2 only
10) Consider the following statements with
b. 2 and 3 only respect to International Commission for the
c. 1, 2 and 3 only Conservation of Atlantic Tunas (ICCAT)

d. None of the above 1. It is an intergovernmental organization


responsible for the management and
conservation of tuna and tuna-like
7) Consider the following statements with species in the Atlantic Ocean and
respect to Prosperity & Inclusion City Seal and adjacent seas.
Awards (PICSA) Index 2. India is a member to this organisation.
1. It provides a new measure of economic Which of the statement(s) given above is/are
productivity that goes beyond GDP. correct?
2. It was commissioned and compiled by a. 1 only
Basque institutions and D&L Partners.
b. 2 only

www.shankariasacademy.com | www.iasparliament.com
93

c. Both 1 and 2 of: (i) Union Minister for Social Justice


(Chairperson); (ii) Minister of State for
d. Neither 1 nor 2
Social Justice (Vice- Chairperson); (iii)
Secretary of the Ministry of Social Justice; (iv)
one representative from ministries including
Answers Health, Home Affairs, and Human Resources
1. b Development.

Plastic Parks  Every transgender person shall have a right


to reside and be included in his
 A plastic park is an industrial zone devoted to household.
plastic enterprises and its allies’ industries.
3. a
 It includes a whole range of companies
required by the plastics processing community  Global emissions of greenhouse gases, which
such as are responsible for global warming, are
growing rather than declining as they should.
1. Material and machinery suppliers,
 Even if all countries meet the unconditional
2. Plastics processing companies, commitments they made at the Paris
3. Plastic recycling companies including waste conference in 2015, GHG emissions will be way
management system. more than what they should be.

 These plastic parks are being developed by  India is among the few countries that are on
State Governments with the support of track to meeting their Paris targets (called
Department of Chemicals & Petrochemical of ‘nationally determined contributions’ or
Ministry of Chemicals & Fertilizers. NDCs).

 The government has approved setting up of 6  This is the central message of the United
plastic parks in Assam, Madhya Pradesh, Nations Environment Programme’s
Odisha, Jharkhand and Tamil Nadu. ‘Emissions Gap Report, 2019’, released
recently.
 Under the scheme, the government provides
grant funding up to 50 per cent of the project  ‘Emissions gap’, as defined by the report itself,
cost. refers to “the difference between where we’re
likely to be and where we need to be.”
 The remaining project cost is to be funded by
State Government beneficiary industries and 4. c
by a loan from financial institutions.  NASA recently displayed a new underwater
2. d robot for a mission that could one day search
for life in ocean worlds beyond Earth.
 The Bill defines a transgender person as one
whose gender does not match the  The robot, known as Buoyant Rover for
gender assigned at birth. Under-Ice Exploration (BRUIE), is being
developed for underwater exploration in
 It includes trans-men and trans-women, extra-terrestrial, icy waters.
persons with intersex variations, gender-
queers, and persons with socio-cultural  It could be used to explore Jupiter’s moon
identities, such as kinnar and hijra. Europa or Saturn’s moon Enceladus.

 Intersex variations is defined to mean a person 5. c


who at birth shows variation in his or her  "ElectroSkin" is a new fundamental
primary sexual characteristics, external building block for a range of soft next-
genitalia, chromosomes, or hormones from the generation robots.
normative standard of male or female body.
 It is an important step towards soft robots that
 It seeks to establish a National Council for can be easily transported, deployed and even
Transgender persons (NCT) will consist worn.

www.shankariasacademy.com | www.iasparliament.com
94

 The combination of electrical artificial muscles  The eSports industry is expected to grow
and electrical gripping replicated the rapidly and in 2017 worldwide revenue
movements of animals like slugs and snails, generated in eSports market amounted to USD
and where they can go, so could our robots. 655 million.
 The thin and light robots have been developed 7. c
using a new way of embedding artificial
 The first-ever non-commercial,
muscles and electrical adhesion into soft
Prosperity & Inclusion City Seal and
materials.
Awards (PICSA)
 Traditional robots are rigid and incompliant Index was commissioned by Basque
whereas soft robots are compliant and can institutions and compiled by D&L
stretch and twist to adapt to their Partners.
environments.
 It provides a new measure of economic
 Until now, soft robots have separated their productivity that goes beyond GDP.
movement abilities from their capabilities to
 It provides a holistic account of how well
grip the surface they move on.
people are doing in an economy and which
 Taking inspiration from biological skins and have the most empowered populations.
soft organisms like snails and slugs,
 It marks the first time that the world’s major
researchers have successfully demonstrated a
cities have been ranked not just by the size and
new robotic skin that crawls across a surface
health of their economy, but for their efforts to
by alternately contracting embedded artificial
build inclusive and prosperous environments
muscles and gripping the surface using
for all its citizens.
electrical charges.
 Bangalore stood at 83rd rank, while Delhi
 In the future, "ElectroSkin" robots could crawl
and Mumbai ranked 101 and 107 among 113
up walls and across ceiling to clean them,
cities in world.
explore difficult to reach environments
including collapsed buildings, and be used in a  Zurich, as the number one, scores strongly
range wearable second-skin devices. across all measures, on quality of life, work,
housing, leisure, safety, and education.
 This new advance could create new thin and
light robots for environmental monitoring and  The Swiss higher education system attains an
deployment in hazardous environments, robot especially high score.
grippers for delicate objects and new wearable
technologies.  Vienna, the Austrian capital in 2nd place,
scores close to top marks on healthcare.
6. c
 Copenhagen, Luxembourg and Helsinki
 The 5th edition of GES 2019 is being held complete the top five.
from 26-28th November 2019 in Bengaluru,
Karnataka. 8. a

 It is an attempt towards escalating the Indian  Ministry of Housing and Urban


services bar in the global arena by exploring Affairs launched the Credit-linked
12 Champion Services Sectors, Subsidy Services Awas Portal (CLAP) of
encompassing participation from 74 countries. Pradhan Mantri Awas Yojana-Housing for All
(Urban).
 It is organised by Department of Commerce,
Ministry of Commerce and Industry in  It aims to provide a transparent and
association with Services Export Promotion robust real-time web-based monitoring
Council (SEPC) and Confederation of Indian system for the beneficiaries of the Credit-
Industry (CII). linked Subsidy Services (CLSS).

 In GES 2019 SEPC is also looking to  It will help in addressing the grievances of
promote eSports. beneficiaries in a much comprehensive and
organized manner and using the portal, a

www.shankariasacademy.com | www.iasparliament.com
95

beneficiary can track his application status in 28-11-2019


real-time.
1) Consider the following statements with
9. d respect to Madden Julian Oscillation (MJO)
 In a move to improve measures for the 1. Unlike ENSO, which is stationary, the
safety and security of women, an MJO is a westward moving disturbance
exclusive bus service for women of clouds, rainfall, winds, and pressure.
passengers was launched in Hyderabad.
2. It traverses the planet in the polar
 'She Shuttle' is equipped with a well- region similar to the jet streams and
monitored surveillance system with CCTV returns to its initial starting point in 30
cameras, a lady security guard and a to 60 days, on average.
smartphone application for every passenger
with a panic button. Which of the statement(s) given above is/are
correct?
10. a
a. 1 only
 Recently, the world’s major fishing nations
b. 2 only
have agreed “modest” quota cuts for the under-
pressure Atlantic Bigeye tuna. c. Both 1 and 2
 International Commission for the d. Neither 1 nor 2
Conservation of Atlantic Tunas
(ICCAT) is an intergovernmental organization
responsible for the management and 2) Researchers have recently discovered a new
conservation of tuna and tuna-like species in species of non-venomous snake Trachischium
the Atlantic Ocean and adjacent seas. apteii, which belongs to the fossorial group
that live mostly underground in which of the
 India is a not member to this organisation. following regions?
 ICCAT is one of many regional fisheries a. Rajasthan
management organizations where countries
and fishing entities come together to discuss b. Western Ghats
and develop international fisheries c. Arunachal Pradesh
management measures.
d. Andaman & Nicobar Islands

3) With respect to 15th Finance Commission,


consider the following statements:
1. The term of 15th Finance Commission
was extended for two years recently.
2. Its recommendations will be applicable
for six years and not the conventional
five-year period.
Which of the statement(s) given above is/are
correct?
a. 1 only
b. 2 only
c. Both 1 and 2
d. Neither 1 nor 2

www.shankariasacademy.com | www.iasparliament.com
96

4) Consider the following statements with 7) Consider the following statements with
respect to The Special Protection Group respect to Dividend Distribution Tax (DDT)
(Amendment) Bill, 2019
1. It is tax to be paid by shareholders when
1. It will provide security only for the theyget their dividends from
Prime Minister and his/her immediate thecompanies profit.
family residing with him at his official
2. It is a part of income tax liability of a
residence, as well as any former prime
company.
minister and members of his immediate
family. Which of the statement(s) given above is/are
correct?
2. According to the bill, the SPG will
provide security to former Prime a. 1 only
Ministers and their immediate family
members for the rest of their lives. b. 2 only

Which of the statement(s) given above is/are c. Both 1 and 2


correct? d. Neither 1 nor 2
a. 1 only
b. 2 only 8) Consider the following statements with
c. Both 1 and 2 respect to FICCI(Federation of Indian
Chambers of Commerce and Industry)
d. Neither 1 nor 2
1. It is agovernment owned, not-for-profit
organisation, which articulates the
views and concerns of industry.
5) Tally Valley Wildlife Sanctuary is located in
which of the following states? 2. It serves its members from the Indian
private and public corporate sectors and
a. Karnataka
multinational companies.
b. Telangana
3. Along with the Ministry of HRD and the
c. Arunachal Pradesh Ministry of Commerce and Industry,
FICCI organises Higher Education
d. Andaman and Nicobar Islands
Summit.
Which of the statement(s) given above is/are
6) Him Vijay Exercise which was in news correct?
recently is?
a. 1 and 2 only
a. An inter-service military exercise involving the
b. 2 and 3 only
participation of the Indian Army, Air Force, Navy
and the Coast Guard c. 1 and 3 only
b. To help to fight against Air Pollution in Delhi d. 1, 2 and 3
c. To validate operational capabilities,test mobility
and coordination of Indian Army in difficult
9) Consider the following statements with
terrain
respect to India Trade Promotion
d. To address the mass death of migratory birds in Organisation (ITPO)
Sambhar Lake
1. It is the premier trade promotion
agency of the Ministry of Commerce &
Industry committed to showcase
excellence achieved by the country in
trade and commerce.

www.shankariasacademy.com | www.iasparliament.com
97

2. It is conferred withCategory-1 Mini- Arunachal Pradesh, named Trachischium


Ratna, Central Public Sector Enterprise apteii.
(CPSE).
 It was found under fallen logs inside a thickly
Which of the statement(s) given above is/are forested area of the Tally Valley Wildlife
correct? Sanctuary near the town of Ziro
in Arunachal Pradesh.
a. 1 only
b. 2 only  The newly discovered species belongs to a
group of fossorial snakes that live mostly
c. Both 1 and 2 underground, and surface mainly during or
d. Neither 1 nor 2 after a heavy monsoon shower.
 Trachischium species are commonly called
slender snakes, and are currently known by
10) Consider the following statements with seven species that are distributed across the
respect to Superbugs Himalayas, and the Indo-Burma and Indo-
1. It refers to microorganisms that develop China regions.
antimicrobial resistance
2. It is associated only with Bacteria.
Which of the statement(s) given above
is/are not correct?
a. 1 only
b. 2 only
c. 3 only
d. None of the above

3. b
Answers
 The Union Cabinet has recently approved the
1. d
extension of the term of the 15th Finance
 Both El Nino and MJO are ocean-atmosphere Commission, which will now submit two
interactions that occur in the equatorial region reports.
and influence a number of weather events
 The term had been extended till October 30,
worldwide, including the Indian monsoon.
2020 (11 months).
 El Nino, the unusual warming of sea-surface
 The first report, for financial year 2020-21, will
Pacific waters off the South American coast, is
be submitted in the coming months before the
a stationary system.
Union Budget, and the second report for the
 The MJO, on the other hand, is a moving period 2021-26 will be presented by October
system of wind, cloud and pressure that 30, 2020.
brings rain as it circles around the equator.
 This will effectively mean the 15th Finance
 The MJO is an eastward Commission’s recommendations will
moving disturbance of clouds, rainfall, winds, be applicable for six years and not the
and pressure that traverses the planet in conventional five-year period.
the tropics and returns to its initial starting
 It is not unprecedented for the term of a
point in 30 to 60 days, on average.
Commission to be extended when there is a
2. c restructuring of States, or if the terms of
reference for the Commission have been
 Researchers have discovered a new species significantly expanded.
of non-venomous burrowing snake in

www.shankariasacademy.com | www.iasparliament.com
98

 It happened before in the 14th Finance 6. c


Commission when Andhra Pradesh and
 Exercise ‘Him Vijay'is the Indian Army's
Telangana were split.
biggest ever mountain combat exercise.
 15th Finance Commission has seen its terms of
 It is to test mobility, communication and
reference expanded and the reorganisation of
coordination of such huge body of fast-
J&K.
moving troops in difficult terrain.
 The 9th Finance Commission under the
 In this exercise, the capability of newly-
chairmanship of N.K.P. Salve also saw its
conceived Integrated Battle Groups
recommendation period extended to six years
(IBGs) was tested.
from the conventional five.
4. a  The 'Him Vijay' will include troop
mobilisation, mountain assault and air assault.
 Under the Act, the SPG provides security to the
7. d
Prime Minister and his immediate family
members.  Indian corporates are now demanding to
abolish the Dividend Distribution
 It also provides security to former Prime
Tax (DDT) on the pretext of double taxation.
Ministers and their immediate family
members for a period of one year from the  A dividend is a return given by a company to
date on which they cease to hold the office. its shareholders out of the profits earned by
the company in a particular year.
 Beyond this period, the SPG security is
provided based on the level of threat as  Dividend constitutes income in the hands of
decided by the central government. the shareholders which ideally should be
subject to income tax.
 The Bill amends this provision to state that the
SPG will provide security to the Prime  However, the income tax laws in India
Minister, and members of his immediate provide for an exemption of the
family residing with him at his official dividend income received from Indian
residence. companies by the investors by levying a tax
called the Dividend Distribution Tax (DDT) on
 It will also provide security to any former
the company paying the dividend.
Prime Ministers, and his immediate family
members residing with him at the residence  DDT is to be paid by ‘companies’ who
allotted to him. distribute their profits to their
shareholders in the form of dividends.
 This security will be provided for a period of
five years from the date on which he ceases  The reason why DDT exists if that the profits
to hold the office of Prime Minister. earned by companies must be retained within
the company to fund its future operations and
 The Act provides that if the SPG security is
growth.
withdrawn from a former Prime Minister, it
will also be withdrawn from his immediate  It is to be paid within 14 days of declaration,
family members, unless the level of threat distribution or payment of dividend whichever
faced by the immediate family member is the earliest.
warrants such security.
 It is applicable only to domestic
 The Bill removes this condition to state that if companies in India.
the SPG security is withdrawn from a
former Prime Minister, it will also be 8. b
withdrawn from his immediate family  Recently, President of India addressed in
members. the 15th FICCI Higher Education
5. c Summit.
 It provides exposure to Global Best Practices
for education, deliberations on policy

www.shankariasacademy.com | www.iasparliament.com
99

roadmap, explore partnerships 10. b


opportunities,and augment their brand value
by taking advantage of multiple promotional  A new research in the US has found a high
opportunities. incidence of Superbugs in Walmart’s pork
products.
 This Summit is being organised by
the FICCI in collaboration with the Ministry  Microorganisms that develop antimicrobial
of Human Resource Development and resistance are referred to as “Superbugs”.
the Ministry of Commerce and Industry.  Antimicrobial resistance happens
 Established in 1927, FICCI is the largest and when microorganisms (such as bacteria,
oldest apex business organisation in India. fungi, viruses, and parasites) change
when they are exposed to antimicrobial drugs
 It is anon-government, not-for-profit (such as antibiotics, antifungals, antivirals,
organisation, FICCI is the voice of India's antimalarials, and anthelmintics).
business and industry.
 Often, superbugs are bacteria that have
 Itarticulates the views and concerns of become resistant to antibiotics. They can also
industry. be fungi.
 It serves its members from the Indian
private and public corporate sectors and
multinational companies,reaching out to 29-11-2019
over 2,50,000 companies. 1. Consider the following statements with
9. c respect to North Atlantic Treaty Organization
(NATO)
 The Ministry of Commerce and Industryis set
to revamp the PragatiMaidan, a space for 1. All decisions are taken by consensus of
exhibitors from India and abroad. all member countries.
2. If the stability is threatened, NATO
 India Trade Promotion Organisation
would even defend non-NATO
(ITPO), is the premier trade
members.
promotionagency of the Ministry of
Commerce &Industry. Which of the statement(s) given above is/are
correct?
 It is committed to showcase excellence
achieved by the country in diverse fields a. 1 only
especially trade and commerce.
b. 2 only
 It provides a wide spectrum of services to trade c. Both 1 and 2
and industry and acts as a catalyst for growth
of India's trade. d. Neither 1 nor 2
 It approves holding of international trade fairs
in India and regulates holding of various 2. Consider the following statements with
expositions in India. respect to Union Territory of Daman & Diu
 It has an extensive infrastructure as well as 1. Daman and Diu are separated by the
marketing and information facilities that are Gulf of Kutch.
availed by both exporters and importers.
2. Both Daman and Diu remained as a part
 It is also providing assistance to State of the Portuguese dominions in India.
Governments in setting up Regional Trade
Promotion Centres (RTPC) in various State's Which of the statement(s) given above is/are
Capital and major cities. not correct?
a. 1 only
b. 2 only

www.shankariasacademy.com | www.iasparliament.com
100

c. Both 1 and 2 Which of the statement(s) given above is/are


correct?
d. Neither 1 nor 2
a. 1 and 2 only
b. 2 and 3 only
3. Consider the following statements with
respect to Green Highways Policy 2015 c. 1 and 3 only
1. It aims to promote greening of Highway d. 1,2 and 3
corridors with participation of only
government institutions.
2. The monitoring mechanism is in place Answers
by using ISRO’s Bhuvan and GAGAN 1. c
satellite systems.
 The North Atlantic Treaty Organization
Which of the statement(s) given above is/are (NATO) is set to meet in London in December
correct? to celebrate its 70th anniversary.
a. 1 only  The member countries consult and take
b. 2 only decisions on security issues at all levels and in
a variety of fields.
c. 3 only
 A NATO decision is the expression of
d. None of the above the collective will of all 29 member
countries since all decisions are taken by
consensus.
4. ‘Phishing’ which was in news recently is
related to,  It is an intergovernmental military alliance
started to provide collective security against
a. Fraudulent use of phone to solicit personal
the Soviet Union.
information, often targets users of Voice over IP
(VoIP) services like Skype and calls.  If the stability is threatened, NATO
would defend non-members.
b. A scam where a fraudster installs malicious
code on a personal computer or server.  On August 28, 2014, NATO announced it had
c. An attack that try to lure victims via text photos proving that Russia invaded Ukraine.
messages (SMS)  Although Ukraine is not a member, it had
d. A fraudulent use of electronic communications, worked with NATO over the years.
especially emails to deceive and take advantage of  Russia's invasion of Ukraine threatened nearby
users. NATO members.
 In the 1990s, NATO stopped further bloodshed
5. Consider the following statements with from occurring in Bosnia and Kosovo.
respect to Ashram Schools
 Since 2003, NATO has helped to ensure that
1. These are residential schools which Afghanistan is no longer a safe haven for
impart education to the children international terrorist groups.
belonging to Scheduled Tribes (STs).
 NATO has also helped to prevent piracy off the
2. The scheme provides for 100% central Horn of Africa.
financing for the construction of
2. a
ashram schools in naxal areas.
3. The Ministry of Tribal Affairs has the  A Bill has been introduced in Lok Sabha
responsibility for running and overall to merge two Union Territories, Daman
maintenance of the schools. and Diu and Dadra and Nagar
Haveli, into one.

www.shankariasacademy.com | www.iasparliament.com
101

 Daman and Diu are separated by the  This was to reduce the impact of air pollution
‘Gulf of Khambhat’, located on India’s and dust by planting trees and shrubs along
western coast near Gujarat. the National Highways.
 For over four centuries, both Daman and  They act as natural sink for air pollutants and
Diu remained part of the Portuguese arrest soil erosion at the embankment slopes.
dominions in India, and were ruled from
Goa.  There is a strong monitoring mechanism
in place by using ISRO’s Bhuvan and
GAGAN satellite systems.
 Every planted tree is counted and auditing is
done. The agencies performing good is being
awarded.
4. d
 Google says it warned 500 users in
India of government-backed Phishing
attacks.
 Phishing - A fraudulent use of electronic
communications, especially emails to
deceive and take advantage of users.
 The goal is to trick the email recipient into
believing that the message is something they
want or need, a request from their bank, for
instance, or a note from someone in their
company and to click a link or download an
attachment.
 A majority of internet scams are considered to
be Phishing scams.
 Pharming - scam where a fraudster
 Dadra and Nagar Haveli are also located installs malicious code on a personal
on India’s western coast near Gujarat, and computer or server.
were colonial possessions of
Portugal before they became part of  This code then redirects any clicks you make
independent India. on a website to another fraudulent Website
without your consent or knowledge.
3. b
 Vishing - Fraudsters use the phone,
 Union Minister for Road Transport and
phone calls to solicit your personal
Highways gave the details of the Green
information.
Highways Policy-2015 in Lok Sabha
recently.  It often targets users of Voice over IP (VoIP)
services like Skype.
 Green Highways (Plantation, Transplantation,
Beautification & Maintenance) Policy 2015  Smishing - It uses cell phone text
was launched to promote greening of messages (SMS) to lure consumers in. Often
Highway corridors with participation of the text will contain an URL or phone number.
the community, farmers, private sector,
NGOs, and government institutions. 5. a

 The vision of the policy is to provide dignified  Ministry of Tribal Affairs has decided to
employment to local people and communities. provide financial support to State
Governments to set up 97 more Ashram
Schools.

www.shankariasacademy.com | www.iasparliament.com
102

 It is under the Special Central Assistance 3. Consider the following statements regarding
to Tribal Sub-Plan (SCA to TSS). NuGen Mobility Summit
 The scheme provides for 100% central 1. It is the largest automotive technology
financing for the construction of summit organised by International
ashram schools for girls and for ashram Center of Automotive Technology
schools for boys in naxal areas. (ICAT).

 The Standing Committee on Social Justice and 2. It covers relevant topics of alternate fuel
Empowerment recommended providing 100% systems and E-mobility.
central financing for all ashram schools. 3. ICAT works under the Ministry of Road
 The ‘State government’ has Transport and Highways.
the responsibility for running and Which of the above statement(s) is/are
overall maintenance of the schools, correct?
including appointment of teachers.
a. 1 and 2 only
b. 1 and 3 only
30-11-2019
c. 2 and 3 only
1. Consider the following statements regarding
d. 1,2 and 3
Yuwaah Youth Skilling Initiative
1. It aims to provide relevant skills to
adolescent girls and boys for their 4. Consider the following statements with
productive lives and future of work. respect to United Nations Framework
Convention on Climate Change (UNFCCC)
2. It was launched by World Bank in India.
1. Conference of Parties (COP) is the
Which of the above statement(s) is/are
supreme decision-making body of the
correct?
UNFCCC.
a. 1 only
2. COP25 to UNFCC will take place under
b. 2 only the Presidency of the Government of
Chile, to be held in Madrid, Spain.
c. Both 1 and 2
Which of the statement(s) given above is/are
d. Neither 1 nor 2
correct?
a. 1 only
2. Consider the following statements regarding
b. 2 only
Anti-Tank Guided Missile (ATGM)
c. Both 1 and 2
1. NAG is 4th generation indigenous
ATGM with a range of 4km. d. Neither 1 nor 2
2. SPIKE is 5th generation ATGM
imported from Israel with target
5. Consider the following statements with
precision of 4km.
respect to SHe-Box
Which of the above statement(s) is/are not
1. It has been developed by NITI Aayog
correct?
along with UN Women with the
a. 1 only objective to facilitate registration of
complaints related to sexual
b. 2 only
harassment of women at workplace.
c. Both 1 and 2
2. It provide a single window access to
d. Neither 1 nor 2 every woman only in organised sector.

www.shankariasacademy.com | www.iasparliament.com
103

Which of the statement(s) given above is/are 2. It hears and disposes of appeals against
not correct? orders passed by the Pension Fund
Regulatory and Development Authority
a. 1 only
(PFRDA) under the PFRDA Act, 2013.
b. 2 only
Which of the above statement(s) is/are
c. Both 1 and 2 correct?
d. Neither 1 nor 2 a. 1 only
b. 2 only
6. ‘Surya Kiran’ is a military exercise between c. Both 1 and 2
a. India – Indonesia d. Neither 1 nor 2
b. India – Russia
c. India – Nepal 10. Consider the following statements
regarding BOD & COD
d. India - Sri Lanka
1. The higher the Biological Oxygen
Demand, the lesser the amount of
7. 'Chagos Islands' which was in news recently dissolved oxygen available for higher
is located in organsims.
a. Oceania 2. Chemical oxygen Demand is slightly a
better measure than Biological oxygen
b. Philippine Sea Demand.
c. South Pacific 3. COD measures the amount of oxygen in
d. Indian Ocean parts per million required to oxidise
only in non-biodegradable
compounds in the water sample.
8. Consider the following statements with Which of the above statement(s) is/are
respect to Hallmarking correct?
1. In India, at present Gold is the only a. 1 only
precious metal brought under the
purview of Hallmarking. b. 1 and 2 only
2. Bureau of Indian Standards (BIS) under c. 2 and 3 only
the Ministry Of Science and Technology d. 1 and 3 only
provides Hallmark certification.
Which of the statement(s) given above is/are
correct? Answers
a. 1 only 1. a
b. 2 only  YuWaah is a multi-stakeholder alliance by
c. Both 1 and 2 UNICEF which aims to facilitate youth to gain
relevant skills for productive lives and the
d. Neither 1 nor 2 future of work.
 The target age group of YuWaah includes
9. Consider the following statements Securities adolescent girls and boys.
Appellate tribunal (SAT)  It intends to create platforms to guide youth to
1. It has all the powers of a Civil Court. market opportunities and facilitate integration
of career guidance in school education.

www.shankariasacademy.com | www.iasparliament.com
104

2. c  It provides a single window access to


every woman, irrespective of her work
 NAG is 3rd generation ATGM whereas SPIKE
status, whether working in organised or
is 4th generation ATGM.
unorganised, private or public sector, to
3. a facilitate the registration of complaint related
to sexual harassment.
 NuGen Mobility Summit-2019 is the first in a
series of the conference organized by 6. c
International Center of Automotive
 Joint military exercise ‘Surya Kiran –
Technology (ICAT), Manesar in Haryana.
XIV’ between India and Nepal will be
 It is the largest automotive technology event conducted at Salijhandi, Rupendehi district of
covering relevant topics of alternate fuel Nepal from 3rd to 16th December 2019.
systems and E-Mobility.
 It is an annual event which is conducted
 In the three-day conference, Automotive alternatively in Nepal and India.
technology experts from 15 countries including
 The aim of this exercise is to increase
India will be presenting more than 120
interoperability in jungle warfare and counter
technical research papers.
terrorist operations in mountainous terrain,
 ICAT, Manesar is a division of NATRIP humanitarian assistance and disaster relief,
Implementation Society (NATIS) under the medical and environmental conservation
Ministry of Heavy Industries and Public including aviation aspects.
Enterprises. 7. d
4. c
 Recently, Mauritius called the UK an “illegal
 The Conference of Parties (COP) is colonial occupier” after it ignored a UN
the supreme decision-making body of mandated deadline to return the Chagos
the UNFCCC. Islands, to Mauritius.

 All States that are Parties to the  The United Nations had given UK six months
Convention are represented at the COP, to process the transfer, a move the UK and the
US have bitterly resisted.
 The Parties review the implementation of the
Convention and other legal instruments that
the COP adopts and take decisions necessary
to promote the effective implementation of the
Convention.
 Massive protests triggered by social unrest
over economic issues have forced Chile to
cancel its hosting of COP25.
 So, the COP25 will take place under the
Presidency of the Government of
Chile, to be held in Madrid, Spain.
 The logistical support will be provided by the
Government of Spain.
5. c
 Chagos Archipelago or Chagos
 SHe-Box (Sexual Harassment electronic Box) Islands are a group of seven atolls comprising
is an online Complaint Management System more than 60 individual tropical
for lodging complaints related to workplace islands located in the Indian Ocean.
sexual harassment.
 Mauritius has argued that the Chagos Islands
 It is an initiative of Ministry of Women and has been a part of its territory since at least the
Child Development.

www.shankariasacademy.com | www.iasparliament.com
105

18th century, till the UK broke the archipelago 10. b


away from Mauritius in 1965.
 Biochemical oxygen demand (BOD), the
 After Mauritius gained independence from the amount of dissolved oxygen used by
UK in 1968, the United Kingdom refused to microorganisms in the biological process of
return the Chagos Islands to Mauritius. metabolizing organic matter in water.
 The largest island on the Chagos Islands  The more organic matter there is (e.g., in
archipelago, Diego Garcia, is where the US sewage and polluted bodies of water), the
and the UK operate a large military base. greater the BOD.
8. d  The greater the BOD, the lower the amount of
dissolved oxygen available for higher animals
 Union Government has announced that
such as fishes.
Notification for mandatory Hallmarking
of Gold Jewellery & Artefacts across the  The BOD is therefore a reliable gauge of the
nation to be issued on 15th January, 2020. organic pollution of a body of water.
 Hallmarking is the accurate determination and  COD measures the amount of oxygen in parts
official recording of proportionate content of per million required to oxidise biodegradable
precious metal in precious metal articles. and non-biodegradable compounds in the
water sample.
 In India, at present 2 precious
metals namely gold and silver have
been brought under the purview of
Hallmarking.
 Bureau of Indian Standards (BIS) is the
National Standard Body of India, under the he
aegis of Ministry of Consumer Affairs,
Food & Public Distribution.
 It is established under the BIS Act
2016 for the harmonious development of the
activities of standardization, marking and
quality certification of goods.
9. c
 Securities Appellate Tribunal is a statutory
body established under the Securities and
Exchange Board of India Act, 1992.
 It hears and disposes of appeals against orders
passed by
1. SEBI under the SEBI Act, 1992
2. Pension Fund Regulatory and Development
Authority (PFRDA) under the PFRDA Act,
2013 and
3. Insurance Regulatory Development Authority
of India (IRDAI) under the Insurance Act,
1938, the General Insurance Business
(Nationalization) Act, 1972 and the Insurance
Regulatory and Development Authority Act,
1999.

www.shankariasacademy.com | www.iasparliament.com

You might also like